Download as pdf or txt
Download as pdf or txt
You are on page 1of 118

PRESTORMINGTM 2022

TEST – 4 ECONOMICS I
EXPLANATION
QUESTION NO PAGE NO
Q.1) ............................................................................................................................................. 4
Q.2) ............................................................................................................................................. 5
Q.3) ............................................................................................................................................. 6
Q.4) ............................................................................................................................................. 7
Q.5) ............................................................................................................................................. 8
Q.6) ............................................................................................................................................. 9
Q.7) ........................................................................................................................................... 11
Q.8) ........................................................................................................................................... 13
Q.9) ........................................................................................................................................... 13
Q.10) ........................................................................................................................................ 15
Q.11) ........................................................................................................................................ 15
Q.12) ........................................................................................................................................ 16
Q.13) ........................................................................................................................................ 17
Q.14) ........................................................................................................................................ 18
Q.15) ........................................................................................................................................ 19
Q.16) ........................................................................................................................................ 20
Q.17) ........................................................................................................................................ 21
Q.18) ........................................................................................................................................ 23
Q.19) ........................................................................................................................................ 24
Q.20) ........................................................................................................................................ 25
Q.21) ........................................................................................................................................ 26
Q.22) ........................................................................................................................................ 27
Q.23) ........................................................................................................................................ 28
Q.24) ........................................................................................................................................ 29
Q.25) ........................................................................................................................................ 30
Q.26) ........................................................................................................................................ 31
Q.27) ........................................................................................................................................ 32
Q.28) ........................................................................................................................................ 33
Q.29) ........................................................................................................................................ 34
Q.30) ........................................................................................................................................ 35
Q.31) ........................................................................................................................................ 36
Q.32) ........................................................................................................................................ 37
Q.33) ........................................................................................................................................ 38
Q.34) ........................................................................................................................................ 39
Q.35) ........................................................................................................................................ 41
Q.36) ........................................................................................................................................ 42
Q.37) ........................................................................................................................................ 43
Q.38) ........................................................................................................................................ 44
Q.39) ........................................................................................................................................ 46
Q.40) ........................................................................................................................................ 47
Q.41) ........................................................................................................................................ 48
Q.42) ........................................................................................................................................ 49
Q.43) ........................................................................................................................................ 50
Q.44) ........................................................................................................................................ 51
Q.45) ........................................................................................................................................ 52
Q.46) ........................................................................................................................................ 53
Q.47) ........................................................................................................................................ 55
Q.48) ........................................................................................................................................ 57
Q.49) ........................................................................................................................................ 59
Q.50) ........................................................................................................................................ 61
Q.51) ........................................................................................................................................ 62
Q.52) ........................................................................................................................................ 63
Q.53) ........................................................................................................................................ 65
Q.54) ........................................................................................................................................ 67
Q.55) ........................................................................................................................................ 68
Q.56) ........................................................................................................................................ 69
Q.57) ........................................................................................................................................ 70
Q.58) ........................................................................................................................................ 72
Q.59) ........................................................................................................................................ 73
Q.60) ........................................................................................................................................ 74
Q.61) ........................................................................................................................................ 75
Q.62) ........................................................................................................................................ 77
Q.63) ........................................................................................................................................ 77
Q.64) ........................................................................................................................................ 78
Q.65) . ....................................................................................................................................... 80
Q.66) ........................................................................................................................................ 81
Q.67) ........................................................................................................................................ 82
Q.68) ........................................................................................................................................ 83
Q.69) ........................................................................................................................................ 83
Q.70) ........................................................................................................................................ 84
Q.71) ........................................................................................................................................ 85
Q.72) ........................................................................................................................................ 86
Q.73) ........................................................................................................................................ 87
Q.74) ........................................................................................................................................ 88
Q.75) ........................................................................................................................................ 89
Q.76) ........................................................................................................................................ 90
Q.77) ........................................................................................................................................ 91
Q.78) ........................................................................................................................................ 92
Q.79) ........................................................................................................................................ 93
Q.80) ........................................................................................................................................ 94
Q.81) ........................................................................................................................................ 95
Q.82) ........................................................................................................................................ 96
Q.83) ........................................................................................................................................ 98
Q.84) ...................................................................................................................................... 100
Q.85) ...................................................................................................................................... 102
Q.86) ...................................................................................................................................... 104
Q.87) ...................................................................................................................................... 105
Q.88) ...................................................................................................................................... 106
Q.89) ...................................................................................................................................... 107
Q.90) ...................................................................................................................................... 108
Q.91) ...................................................................................................................................... 108
Q.92) ...................................................................................................................................... 110
Q.93) ...................................................................................................................................... 110
Q.94) ...................................................................................................................................... 111
Q.95) ...................................................................................................................................... 112
Q.96) ...................................................................................................................................... 113
Q.97) ...................................................................................................................................... 114
Q.98) ...................................................................................................................................... 116
Q.99) ...................................................................................................................................... 117
Q.100) ...................................................................................................................................... 118
Q.1) Consider the following statements about the “Incremental Capital Output Ratio” (ICOR):

1. An improvement in the value of ICOR is indicative of better utilisation of resources in an economy.


2. When the value of ICOR is high in an economy, it indicates that even a minor increase in
investment can raise the output more than proportionately.

Which of the statements given above is/are correct?

(a) 1 only
(b) 2 only
(c) Both 1 and 2
(d) Neither 1 nor 2

EXPLANATION:

The incremental capital-output ratio (ICOR) is the amount of capital required to produce one unit of
output.
The higher the ICOR, the less efficient we are in the use of capital.
So in case of improvement of ICOR (improvement = getter better in a more positive way), means better
utilisation of resources.
So, Statement 1 is correct.
ICOR can be calculated as:

ICOR = Annual Investment/Annual Increase in GDP

Therefore, when ICOR(Fraction) is high , it means that Numerator (i.e.) Capital(Investment required to
raise the output in an economy ) is high already, So any further minor increase would only
proportionately increase the Output and it does not increase more than the proportion. (Which is an
extreme case)
So, Statement 2 is not correct.

ADDITIONAL INFORMATION:

Incremental Capital Output Ratio (ICOR):

ICOR • The incremental capital output ratio (ICOR) is a frequently used tool that
explains the relationship between the level of investment made in the economy
and the subsequent increase in the gross domestic product (GDP).
• ICOR indicates the additional unit of capital or investment needed to produce an
additional unit of output.
Significance • The incremental capital output ratio (ICOR) explains the relationship between
the level of investment made in the economy and the consequent increase in
GDP.
• ICOR is a metric that assesses the marginal amount of investment capital
necessary for a country or other entity to generate the next unit of production.
• A lower ICOR is preferred as it indicates a country's production is more efficient.
• Some critics of ICOR have suggested that the use of ICOR is limited as it favors
developing countries that can increase infrastructure and technology use as
opposed to developed countries, which are operating at the highest level
possible.
Indian • As we look at the Indian performance in the last five years, two facts stand out.
Scenario One is a decline in the investment rate and the second is a rise in ICOR; both of
which can only lead to a lower growth rate.
• As growth was coming down sharply initially, the investment rate was falling
only slowly, implying a rising ICOR.
• India’s investment rate reached a peak in 2007-08 at 38.0% of GDP.
• With an ICOR of 4, it was not surprising that a high growth rate of close to 9.4%
was achieved.
• One sees a steady decline in the investment rate since then.
• Therefore, It is necessary to improve Public as well as Private Investments ;
reforms to simplify procedures, speed up the delivery system and enlarge
competition must be pursued vigorously.
• All viable “stalled” projects must be brought to completion.
• Financial bottlenecks need to be cleared. The banking system is under stress.
The non-performing loans of the system have risen and are rising.

Q.2) Which one of the following statements is correct?

(a) High per capita income is indicative of high standards of living for all the people living in an
economy
(b) A high value of the Gini Coefficient suggests a relatively even distribution of income among the
people living in an economy
(c) High Gross Domestic Product need not necessarily mean high prosperity in an economy
(d) A low value of the Gini Coefficient is indicative of high standards of living for all the people living in
an economy

EXPLANATION

High per capita income is indicative of high standards of living, whereby it doesn't account for income
inequality. Per capita income is calculated from countries national income which denotes the growth
of the economy, Whereas the standard of living is derived from development. It does not ensure a high
standard of living for all the people living in an economy.
So, Option (a) is not correct.

Gini Coefficient is often used as a gauge of economic inequality, measuring income distribution among
the population. The coefficient ranges from 0 to 1, with 0 representing perfect equality and 1
representing perfect inequality. so the high value of the Gini Coefficient suggests a relatively uneven
distribution of income among people. The Low value of the Gini Coefficient suggests a relatively even
distribution of income and is indicative of high standards of living for most of the population. . The Gini
Coefficient of 0 is indicative of high standards of living for all the people that is perfect equality.
So, Options (b) and (d) are not correct.

Gross Domestic Product is the total value of all the final goods and services produced within the
boundary of the country during a given period. High Gross Domestic Product need not necessarily
mean high prosperity in an economy since GDP represent the growth of a country. It is used to
estimate the size of an economy and its growth rate.
So, Option (c) is correct.

ADDITIONAL INFORMATION

Per Capita Income • It is the measure of the amount of money earned per
person in a country
• It is used to evaluate the standard of living
and quality of life of the population
• Calculation
1. By dividing the country's national income by its
population
• Limitations are its inability to account for
1. Inflation
2. Income disparity
3. Poverty
4. Wealth
5. Savings
Gini Coefficient • The Gini index, or Gini coefficient, is a measure of the
distribution of income across a population
• Gini Coefficient Ranges
1. The coefficient ranges from 0 to 1
2. 0 representing perfect equality
3. 1 representing perfect inequality
4. Values over 1 are theoretically possible due
to negative income
Gross Domestic Product • It is the monetary value of all finished goods and
services made within a country during a specific
period.
• Calculation using three methods
1. Expenditures Method
2. Production Method
3. Incomes Method
• Types of GDP
1. Nominal GDP
2. Real GDP

Q.3) Consider the following statements about currency in India:

1. Reserve Bank of India prints currency notes of all denominations in India while the Government of
India mints coins of all denominations.
2. Coins of denominations such as Rs. 75 and Rs. 100 have been minted by the Government of India
from time to time and issued as legal tender.

Which of the statements given above is/are correct?

(a) 1 only
(b) 2 only
(c) Both 1 and 2
(d) Neither 1 nor 2

EXPLANATION

The Reserve Bank of India (RBI) prints and manages currency in India, The RBI is permitted to print
currency up to 10,000 rupee notes except for 1 rupee note. The Indian government is solely
responsible for minting coins as well as 1 rupee notes. Although the government handles minting
coins, the Reserve Bank issues them for circulation.
So, Statement 1 is not correct.
Coins of denominations such as Rs.75 and Rs. 100 are commemorative coins. Commemorative coins
are not used for circulation, coins that are all under circulation can be used as legal tender. Presently,
coins of 50 paise, ₹ 1/-, 2/-, 5/- and 10/- denominations of various sizes, themes and designs are in
circulation.
So, Statement 2 is not correct.

ADDITIONAL INFORMATION

Currency in India
Bank of issue • Reserve Bank of India prints and manages currency in India
• Indian government regulates what denominations to
circulate
Currencies • The RBI is permitted to print currency up to 10,000 rupee
notes
• The only exemption is 1 rupee notes
Coins • The Indian government is solely responsible for minting coins
• 1 rupee note is also released by the government
• Coins are minted at the four India Government Mints
1. Mumbai
2. Alipore(Kolkata)
3. Saifabad(Hyderabad)
4. Cherlapally (Hyderabad)
5. NOIDA (UP)
Coins in circulation • Coins up to 50 paise are called small coins
• Coins of Rupee one and above are called Rupee Coins
• Coins that are all under circulation can be used as legal
tender
• Coins of 50 paise, ₹ 1/-, 2/-, 5/- and 10/- are under
circulation

Q.4) Which one of the following statements is correct?

(a) Treasury Bills tend to be riskier than Commercial Papers


(b) Commercial Papers tend to be less rewarding than Treasury Bills
(c) Certificate of Deposits are secured short term instruments issued by the banks
(d) Treasury Bills are often referred to as risk-free gilt-edged instruments

EXPLANATION
Treasury Bills are issued by RBI on behalf of the Central Government and Commercial Papers are
issued by high-rated companies. Therefore Treasury bills carry practically no risk of default. So,
Option (a) is not correct.
Since Commercial Papers are riskier than Treasury bills they provide higher rewards than
Treasury Bills to attract lenders. So, Option (b) is not correct.
Certificate of Deposits is an unsecured money market instrument. Banks issue short term
Certificate of Deposits that is for a period of 7 days to 1 year. Financial Institutions issue medium
term Certificate of Deposits that is for a period of 1 year to 3 years. So, Option (c) is not correct.
Since Treasury Bills are issued by Government entities, they carry practically no risk of default
and, hence, are called risk-free gilt-edged instruments. So, Option (d) is correct.
ADDITIONAL INFORMATION
Instruments of Money Market
Money • Money markets are short-term debt (a year or less than a year)
Market products.
• Characterized by a high degree of safety and relatively low level of
return.
• It is regulated by the Reserve Bank of India.
Classification • In India, the Money market is classified into
1. Organized Money market
2. Unorganized Money market
3. Non-Banking Financial Companies
Instruments • Treasury Bills
of Organized • Certificate of Deposits
Money • Commercial Papers
Market • Call and Notice Money
• Promissory Note
• Commercial Bills or Bills of Exchange
• Banker’s Acceptance
• Repurchase Agreements
• Inter-bank term market
Treasury • T-bills are issued by RBI on behalf of the Government of India.
Bills • Issued in 3 tenors 91 days, 182 days, and 364 days.
• Issued at discount and redeemed at face value at maturity.
• They are risk-free gilt-edged instruments (government-issued).
Certificate of • They are unsecured negotiable promissory notes.
Deposits • Issued by Commercial banks (7 days to1 year) and Financial
Institutions (1 year to 3 years).
Commercial • They are an unsecured short-term instrument.
Papers • Issued by a listed company.
• They offer high return rates.
Call and • Under the call money market, funds are transacted on an overnight
Notice basis.
Money • Under notice money market, funds are transacted for the period
between 2 and 14 days.
• It is for uncollateralized lending and borrowing of funds.
Commercial • They are unsecured short-term debt.
Bills • Issued by large banks or corporations.
Repurchase • Repurchase Agreements (Repo) are collateralized short-term borrowing
Agreements and lending through sale/purchase operations in debt instruments.
• Holder of securities sells them to an investor with an agreement to
repurchase at a predetermined date and rate.
Q.5) Which of the following is not a likely outcome of making capital account freely convertible?

(a) It is likely to result in greater investments into India


(b) It will help overcome India's current account deficit
(c) It may cause volatility in the Balance of Payments
(d) India will be more exposed to financial crises happening around the world

EXPLANATION

Capital account convertibility means the freedom to convert home currency into foreign currency
regarding the transaction in the capital account. Full capital account convertibility opens up the
country’s markets to global players including investors, businesses, and trade partners which helps
overcome India's current account deficit.
So, Option (a) is not correct.
India's Current account deficit happens due to a negative Trade balance and Net Invisibles. Though
Trade balance can be rectified by increasing export through Stable and Mature Markets, the Net
Invisibles cannot be rectified. So it will not help to overcome India's Current Account Deficit
So, Option (b) is correct.
Capital account convertibility causes easy access to foreign capital and opens up the country's market
that causing Changes in import and export. It brings in foreign investment, hence causing volatility in
the Balance of payments.
So, Option (c) is not correct.
A fully convertible capital account opens up the county's market to global players which will result in
exposure to financial crises happening around the world.
So, Option (d) is not correct.

ADDITIONAL INFORMATION

Capital Account Convertibility


capital account freely convertible refers to the freedom to convert home currency into foreign currency
regarding transactions in the capital account.
Benefits • Market returns
• Reduction in transaction cost due to free rupee
convertibility
• Improvement in savings and investments

Advantages of freely convertible capital • Sign of Stable and Mature Markets


account convertibility • Increased Liquidity in Financial Markets
• Improved Employment and Business
Opportunities
• Onshore Rupee Market Development
• Easy Access to Foreign Capital
• Better Access to a Variety of Goods and Services
• Progress in Multiple Industry Sectors
• Improved Financial System

Disadvantages • High Volatility


• Foreign Debt Burden

Q.6) Consider the following statements about the “Gross Value Added” (GVA):

1. It refers to the total value of all final goods and services produced in an economy during an
accounting period calculated at market price.
2. GVA is calculated by considering the value-added during each point of a production process and
then finally summing all such value additions.
3. GVA includes the net taxation at the point of production but excludes the net taxation at the point
of sale.

Which of the statements given above is/are correct?

(a) 2 only
(b) 2 and 3 only
(c) 1 and 2 only
(d) 1, 2 and 3
EXPLANATION :

Gross value added (GVA) is defined as the value of output less the value of intermediate
consumption.

It is used to measure the output or contribution of a particular sector.

When such GVAs from all sectors (∑ GVA) are added together and adding taxes (product) and
reducing subsidies (product), we can get the GDP (at market price).

 Earlier, India had been measuring GVA at ‘factor cost’ till the new methodology was adopted in
2105 in which GVA at ‘basic prices’ became the primary measure of economic output.

o GVA at basic prices will include production taxes and exclude production subsidies.

o GVA at factor cost included no taxes and excluded no subsidies.

 The base year has also been shifted to 2011-12 from the earlier 2004-05.

So, Statement 1 is not correct.

GDP at Market Prices = ∑ GVA at basic prices + product taxes – product subsidies.

In this context, when GVA from all sectors are added together and necessary adjustment for taxes and
subsidies are made, we will get the GDP for the economy.
GVA is for a particular sector
∑GVA is for the economy
GDP is for the economy
So, Statement 2 is correct.
GVA at factor cost + (Production taxes less Production subsidies) = GVA at basic prices
GDP at market prices = GVA at basic prices + Product taxes- Product subsidies

Therefore, since we consider only GVA at basic prices it includes production taxes and excludes
product taxes.
So, Statement 3 is correct.
ADDITIONAL INFORMATION:
Gross Value Added:

GVA • Gross value added (GVA) is defined as the value of output less the value of
intermediate consumption.
• Value added represents the contribution of labour and capital to the production
process.
• When the value of taxes on products (less subsidies on products) is added, the
sum of value added for all resident units gives the value of gross domestic
product (GDP).
• Thus, Gross Domestic Product (GDP) of any nation represents the sum total of
gross value added (GVA) (i.e, without discounting for capital consumption or
depreciation) in all the sectors of that economy during the said year after
adjusting for taxes and subsidies.
Calculation • In India, GDP is estimated by Central Statistical Office (CSO).
• In the revision of National Accounts statistics done by Central Statistical
Organization (CSO) in January 2015, it was decided that sector-wise wise
estimates of Gross Value Added (GVA) will now be given at basic prices instead
of factor cost.
• In simple terms, for any commodity the basic price is the amount receivable by
the producer from the purchaser for a unit of a product minus any tax on the
product plus any subsidy on the product.
• However, GVA at basic prices will include production taxes and exclude
production subsidies available on the commodity.
• On the other hand, GVA at factor cost includes no taxes and excludes no
subsidies and GDP at market prices include both production and product taxes
and excludes both production and product subsidies.
• The relationship between GVA at Factor Cost and GVA at Basic Prices and GDP
at market prices and GVA at basic prices is shown below:
o GVA at factor cost + (Production taxes less Production subsidies) = GVA
at basic prices
o GDP at market prices = GVA at basic prices + Product taxes- Product
subsidies
• Production taxes or production subsidies are paid or received with relation to
production and are independent of the volume of actual production.
• Some examples of production taxes are land revenues, stamps and registration
fees and tax on profession.
• Some production subsidies include subsidies to Railways, input subsidies to
farmers, subsidies to village and small industries, administrative subsidies to
corporations or cooperatives, etc. Product taxes or subsidies are paid or received
on per unit of product.
• Some examples of product taxes are excise tax, sales tax, service tax and import
and export duties. Product subsidies include food, petroleum and fertilizer
subsidies, interest subsidies given to farmers, households, etc. through banks.
• The concept of GVA at basic prices follows from the United Nation's System of
National Accounts (SNA) introduced in 1993 and carried forward in an identical
fashion in SNA 2008 as a part of revision of compilation and classification
systems.
• This has been adopted by CSO in its base revision carried out in January 2015.

Q.7) Consider the following statements about Gender Development Index (GDI) and Gender Inequality
Index (GII):
1. While the GDI is published by United Nations Development Programme, GII is published by
World Economic Forum.
2. GDI measures gender inequalities in three important aspects of human development -
reproductive health, empowerment and economic status.
3. GII measures gender gaps in human development by accounting for disparities for women and
men in three basic dimensions - health, knowledge and living standards

Which of the statements given above is/are not correct?

(a) 1 only
(b) 2 and 3 only
(c) 1 and 2 only
(d) 1, 2 and 3
EXPLANATION

GDI and GII both are published by United Nations Development Programme (UNDP)
So, Statement 1 is not correct.
The GDI measures gender gaps in human development achievements by accounting for disparities
between women and men in three basic dimensions of human development health, knowledge and
living standards.
So, Statement 2 is not correct.
The GII measures gender inequality in three important aspects of human development reproductive
health, empowerment, and economic status
So, Statement 3 is not correct.

ADDITIONAL INFORMATION

Index Published Dimensions Characters


by
Gender UNDP • Three dimensions • It is a direct measure of the gender
Development 1. Health gap showing the female Human
Index (GDI) 2. Knowledge Development Index(HDI) as a
3. Living standard percentage of the male HDI
Gender UNDP • Three important • The GII is built on the same
Inequality aspects framework as the Inequality Adjusted
Index (GII) 1. Reproductive Human Development Index (IHDI) to
health better expose differences in the
2. Empowerment distribution of achievements between
3. Economic status women and men

GDI

GII
Q.8) Which one of the following statements is correct regarding the capital expenditure of the
Government of India?

(a) Capital expenditure includes the grants given to states also, as long as they are used for asset
creation
(b) Every capital expenditure has to result in the creation of asset
(c) Expenditure towards the implementation of every scheme forms part of the capital budget
(d) Government can borrow within permissible limits as long as it is used for capital expenditure

EXPLANATION:

The government of India’s capital expenditure will have only provisions for the loans or advances to the
state government, but that will not include grants which is a revenue expenditure, even it is used for
asset creation. So, option (a) is not correct.
The expenditure that occurred is not necessarily resulting in asset creation because the loans borrowed
for the purpose also include the repayment of interests for loans plus the asset creation. So, option (b)
is not correct.
The scheme implementation includes both revenue and capital expenses. For example, in the
MGNREGA act, the salaries given to the workers will fall under the revenue expenditure. So, option (c)
is not correct.
Under the fiscal measures, the FRBM act restricts the borrowings of the government up to a certain
limit to make the fiscal deficit under the control. So, option (d) is correct.

ADDITIONAL INFORMATION:

CAPITAL EXPENDITURE
BENEFITS • Much needed for the economic slowdown due to coronavirus
and the fall in employment ratio.
• It results in effective value creation and generates cash flows.
• Leads to the expansion of industries and services.
• Acts as a macroeconomic stabilizer to countercyclical fiscal
policy.
CONCERNS • Bureaucratic procedural hurdles result in high maintenance
costs and time lag.
• This will have the time lag effect because of the capacity
available in the industry, inventory, work in progress
• The quality control and the costs of revenue expenditure are
not monitored.

Q.9) Which one of the following results in greater money multiplication in an economy?

(a) A higher reserve ratio in the banking system


(b) A push towards increasing consumption
(c) Increase in demand deposits in the banking system
(d) Increase in term deposits in the banking system
EXPLANATION :

High Reserve Ratio would decrease the money available with the banks to lend and thus the money
multiplier.
So, Option (a) is not correct.

Increased Consumption decreases the savings habit amongst the people and thus decreases the
money multiplier.
So, Option (b) is not correct.

Increase in Deposits would in any way increase Money Multiplier in an economy as banks would have
lot of money available to lend whereas among the two (Demand and Term) Term deposits would
greatly increase the Money multiplier for the simple reason that banks could hold those deposits for
longer time than demand deposits and thus increasing MM.

So, Option (c) is not correct & Option (d) is correct.

ADDITIONAL INFORMATION:

Money
Multiplier MM MM = Change in Total Money Supply/ Change in monetary base

• The Money Multiplier refers to how an initial deposit can lead to a bigger
final increase in the total money supply.
• For example, if the commercial banks gain deposits of 1lakh and this leads
to a final money supply of 10 lakhs. The money multiplier is 10.
• The money multiplier is a key element of the banking system.
1. There is an initial increase in bank deposits (monetary base)
2. The bank holds a fraction of this deposit in reserves and then lends out the
rest.
3. This bank loan will, in turn, be re-deposited in banks allowing a further
increase in bank lending and a further increase in the money supply.

Reserve Ratio The reserve ratio is the % of deposits that banks keep in liquid reserves.
Money Multiplier= 1/ Reserve Ratio

In theory, we can predict the size of the money multiplier by knowing the reserve
ratio.

• If you had a reserve ratio of 5%. You would expect a money multiplier of
1/0.05 = 20
• This is because if you have deposits of 1lakh and a reserve ratio of 5%. You
can effectively lend out 20 lakhs.
Currency • The currency deposit ratio shows the amount of currency that people hold
Deposit Ratio as a proportion of aggregate deposits.
• An increase in cash deposit ratio leads to a decrease in money multiplier.
An increase in deposit rates will induce depositors to deposit more, thereby
leading to a decrease in Cash to Aggregate Deposit ratio. This will in turn
lead to a rise in Money Multiplier.
Q.10) Which one of the following statements is not correct?

(a) Money market includes transactions in both debt securities and shares
(b) Capital market generates money for long term investment
(c) Money market helps companies to get cash for meeting operational requirements
(d) Capital market includes transactions in the primary market also

EXPLANATION

The money market includes transactions in debt securities not includes shares that come under the
capital market. The money market includes commercial banks, Regional Rural Banks, and Bill
markets.
So, Option (a) is not correct.

Capital market is the market for borrowing and lending of medium and long terms funds say above 3
years. It helps to generate bulk funds for the government and industries.
So, Option (b) is correct.

The money market is for borrowing and lending of funds for short terms, it helps companies to get cash
for meeting operational requirements.
So, Option (c) is correct.

Capital markets include security markets under which new and old issue markets occur. The new
issue market is also known as the Primary market. The old issue market is also known as the
Secondary market. The primary market includes the sale and purchase of new securities.
So, Option (d) is correct.

ADDITIONAL INFORMATION

Money market • The market for borrowing and lending of short term funds
• Includes
1. Commercial banks
2. Regional Rural Banks
3. Bill markets
Composition of • Money market
money market 1. Organized
a. Banking sector
b. Submarkets
2. Unorganized
a. Moneylenders
b. Merchant cum money lenders
Capital market • Security market
1. Government securities market
2. Industrial Securities Market
• Development financial institution
• Financial intermediaries

Q.11) Which one of the following statements best conveys the concept of “Aggregate Demand”?
(a) It is the sum of demand of all households in an economy
(b) It is the sum of demand for all consumer goods and services, fixed capital goods, change in
the stock of unused raw materials, half produced goods and unsold goods, valuables,
government spending and net exports
(c) It is the sum of demand for all consumer goods and services, fixed capital goods, change in the
stock of unused raw materials, half produced goods and unsold goods, valuables, government
spending, exports and imports
(d) It is the sum of demand for all goods and services within an economy

ADDITIONAL INFORMATION:
Aggregate Demand :

Definition • Aggregate demand measures the total amount of demand for all finished
goods and services produced in an economy.
• Aggregate demand is expressed as the total amount of money spent on those
goods and services at a specific price level and point in time.
• Aggregate demand consists of all consumer goods, capital goods (factories and
equipment), exports, imports, and government spending.
Calculation • Aggregate demand over the long term equals gross domestic product (GDP)
because the two metrics are calculated in the same way.
• GDP represents the total amount of goods and services produced in an
economy while aggregate demand is the demand or desire for those goods.
• As a result of the same calculation methods, the aggregate demand and GDP
increase or decrease together.
• Aggregate demand consists of all consumer goods, capital goods (factories and
equipment), exports, imports, and government spending programs.
Formula • The equation for aggregate demand adds the amount of consumer spending,
private investment, government spending, and the net of exports and imports.
The formula is shown as follows:
Aggregate Demand= C+I+G+Nx
where:
• C=Consumer spending on goods and services
• I= Private investment and corporate spending on non-
final capital goods (factories, equipment, etc.)
• G = Government Spending
• Nx= Net Exports

Q.12) The car industry and Airlines markets in India are examples of

(a) Oligopoly

(b) Monopoly

(c) Monopolistic Competition

(d) Perfect Competition

EXPLANATION:

Under the market structures of the economy, the term “OLIGOPOLY” refers to a small number of
producers working, either explicitly or tacitly, to restrict output and/or fix prices, to achieve
above normal market returns. They are interdependent. Examples of this are oil and gas, airlines,
mass media, automobiles, and telecom. So, option (a) is correct.
A monopoly is a dominant position of an industry or a sector by one company, to the point of
excluding all other viable competitors. Monopolies are often discouraged in free-market nations.
They are seen as leading to price-gouging and deteriorating quality due to the lack of choices for
consumers. They also can concentrate wealth, power, and influence in the hands of one or a few
individuals. Some of the examples are Google, patents, trademarks, railways, etc. So, option (b)
is not correct.
Monopolistic competition characterizes an industry in which many firms offer products or
services that are similar (but not perfect) substitutes. Barriers to entry and exit in a monopolistic
competitive industry are low, and the decisions of anyone firm do not directly affect those of its
competitors. The clothing sector, restaurants, consumer electronics, etc are some of the
examples. So, option (c) is not correct.

The perfect competition occurs when all companies sell identical products, market share does not
influence price, companies can enter or exit without barrier, buyers have perfect or full
information, and companies cannot determine prices. In other words, it is a market that is
entirely influenced by market forces. It is the opposite of imperfect competition, which is a more
accurate reflection of a current market structure. Some of the examples are agriculture, online
shopping, etc. So, option (d) is not correct.

Q.13) Global Gender Gap Index is published by which one of the following organizations?

(a) United Nations Development Programme


(b) United Nations Educational, Scientific and Cultural Organization
(c) World Economic Forum
(d) UN Women

EXPLANATION

World Economic Forum is responsible for the publication of the Global Gender Gap Index. India had
ranked 112th among 153 countries in the Global Gender Gap Index 2020. To track progress on
relative gaps between women and men on dimensions like health and survival, education
attainment, political empowerment and economic participation. Its score range from 1 to 0, 1
representing equality and 0 representing inequality.
So, Option (c) is correct.

ADDITIONAL INFORMATION

Reports Published by

World Development Report World Bank

Global Economic Prospect (GEP) Report World Bank

Global Competitiveness Report World Economic Forum

Global Gender Gap Index World Economic Forum

Global Environment Performance Index World Economic Forum

World Economic Outlook International Monetary Fund (IMF)


Gender Parity Index United Nations Educational, Scientific and Cultural
Organization (UNESCO)

World Wildlife Crime Report United Nations Office on Drugs and Crime

World of Work Report International Labour Organization (ILO)

Human Development Report United Nations Development Programme (UNDP)

Gender Inequality Index United Nations Development Programme (UNDP)

Global Food Price Index Food & Agriculture Organisation (FAO)

Global Innovation Index World Intellectual Property Organisation (WIPO)

Red List International Union for Conservation of Nature


(IUCN)

World Press Freedom Index Reporters Without Borders

Corruption Perception Index Transparency International

International Intellectual Property Index US Chamber of Commerce

Q.14) Which one of the following statements best describes the “Fiscal Deficit” of the Government of
India?

(a) It is the total borrowings of the Government of India


(b) It is the difference between the total expenditure and total receipts after excluding the
borrowings during a budgeting period
(c) It is the shortfall that the government faces in meeting its capital expenditures during a budgeting
period
(d) It is the total borrowings of all governments in India

EXPLANATION:

The total borrowings include all the debt-creating values for the long period where India’s debt to GDP
ratio is 70% since 1991, but the fiscal deficit is measured only for that particular year and not
calculated for all debts. The fiscal deficit of the Government of India consists only of estimates of
central government and not the state government values.
The fiscal deficit refers to the shortfall of its income compared to its spending of the government of
India. The shortfall faced by them to meet its capital expenses is called a capital deficit and also only
the debt-creating nature of capital assets will fall under the fiscal deficit. So, option (b) is correct.

ADDITIONAL INFORMATION:

FISCAL DEFICIT
IMPACTS • It might reduce the sovereign bond rating of the
country.
• It will affect the interests of both the government
and the Indian business.
• It will make it difficult to raise funds abroad and
attract foreign investments.
• It will crowd out the private investment and
possibly end in inflation.
MEASURES • The FRBM(fiscal responsibility and budget
management) act 2003, has targeted to control
fiscal deficit of3% of GDP.
• The committee of NK SINGH suggested to control
the deficit of 3% in March 2020, 2.8% in 2020-21,
and 2.5% in 2023.
• Recently, the central government aimed to achieve
6.8% of the fiscal deficit in 2021-22 and 4.5% in
2025-26.

Q.15) Which one of the following will be considered during the calculation of GDP in an open economy?
(a) The value of goods that are subsequently used as input material in further production during the
same accounting year
(b) All final goods consumed by residents during an accounting period
(c) All services including the ones for self-consumption rendered during the accounting period
(d) Half-finished goods produced during the accounting period

EXPLANATION :

The given statement is about intermediate goods and GDP takes into account only finished goods as
that would lead to double counting.
So, Statement 1 is not correct.
It is the value of all final goods and services produced by the normal residents as well as non-
residents in the domestic territory of the country.
Whatever is produced in India, whether by an Indian or foreign national is part of Indian GDP.
So, Statement 2 is not correct.

The output of domestic and personal services produced for own consumption within households is not
included, although housing services produced for own consumption by owner-occupiers and services
produced on own account by employing paid domestic staff are included.
So, statements 3 is not correct.

Half finished goods produced during the accounting period will be considered in the calculation of GDP.
So, statement 4 is correct.

ADDITIONAL INFORMATION :
Gross Domestic Product:

Definition • The gross domestic product (GDP) of a nation is an estimate of the total value
of all the goods and services it produced during a specific period, usually a
quarter or a year.
Significance • Its greatest use is as a point of comparison: Did the nation's economy grow or
contract compared to the previous period measured?
Measurement 1. GDP can be calculated by adding up all of the money spent by consumers,
businesses, and government in a given period (Expenditure Method)
2. It may also be calculated by adding up all of the money received by all the
participants in the economy. (Income Method)
In either case, the number is an estimate of "nominal GDP."
Once adjusted to remove any effects due to inflation, "real GDP" is revealed.
3. Or by Product/ Value added method.
Expenditure • Thus, a country’s GDP is the total of consumer spending (C) plus business
Method investment (I) and government spending (G), plus net exports, which is total
exports minus total imports (X – M).
• GDP= C + I + G + X – M
Income • The flip side of spending is income. Thus, an estimate of GDP may reflect the
Method total amount of income paid to everyone in the country.
• This calculation includes all of the factors of production that make up an
economy.
• It includes the wages paid to labor, the rent earned by land, the return on
capital in the form of interest, and the entrepreneur’s profits.
• All of these make up the national income.
Product or • Gross value added of firm, i (GV Ai) ≡ Gross value of the output produced by
Value Added the firm i (Qi) –
Method • Value of intermediate goods used by the firm (Zi)
• If there are N firms in the economy, each assigned with a serial number from 1
to N, then
• GDP ≡ Sum total of the gross value added of all the firms in the economy ≡
GVA1 + GVA2
• + · · · + GVAN

Q.16) The Indian rupee is an example of which one of the following?

(a) Fiat money


(b) Convertible representative paper money
(c) Full-bodied money
(d) All of the above

EXPLANATION :

The currency notes that are issued by Reserve Bank of India are ‘fiat’ paper money, that is, they are
issued by the fiat i.e. order) of the Government.
So, Statement 1 is correct.

Under Convertible representative paper money the paper currency issued by the Central Bank was fully
backed by the reserves of gold and silver of equal value kept by it.

Indian money is not convertible into gold or other precious metals.

Thus, when paper money is inconvertible, the issuing authority (RBI) is not responsible to convert the
paper notes into gold or gold coins.
So, Statement 2 is not correct.

Any coins whose face value and intrinsic value are equal, is known as a full bodied.

Gold sovereign of Britain or the silver rupee of India were full bodied money because if they were not
used as money, then gold or silver could be sold in the market at the same price at which that coin
could be used.

But these days the face value of the coins is different from their intrinsic value.
So, Statement 3 is not correct.
ADDITIONAL INFORMATION :

Types of Money :

Convertible • The term paper money only applies to Government notes and the notes issued
Paper Money by the Central Bank of the country.
• In early times when notes were introduced they were backed by an exactly
equal amount in gold or silver kept in reserve” by the issuing authority.
• Such notes could be exchanged for gold or silver coins when demanded and did
nothing more than representing metallic coins.
• Such paper money or notes are therefore called representative paper money.
• Therefore, this paper currency system was called “Full Reserve System”.
• But with the passage of time it was thought that a cent per cent reserve against
paper currency issued was not needed and instead only proportion of 30 to 50
per cent was enough to convert the notes presented for conversion into gold.
• Paper money now-a-days is not wholly backed by gold or silver.
• Representative paper money is now not found anywhere in the world.

Inconvertible • Under the inconvertible paper money system, money is not convertible into
Paper Money gold or other precious metals.
• Thus, when paper money is inconvertible, the issuing authority is not
responsible to convert the paper notes into gold or gold coins.
• The currency notes that are issued by Reserve Bank of India are ‘fiat’ paper
money, that is, they are issued by the fiat i.e. order) of the Government.
• As they are legal tender, they are generally acceptable in exchange for goods
and services and for payment of debt.
• It may be noted that ‘promises to pay’ written on the currency notes are not
‘promises to pay’ something else.
Standard or • Any money whose face value (exchange value) is equal to its intrinsic value (the
full-bodied worth of the metallic content of money) is called standard money. Full-bodied
money money requires the fulfillment of two conditions:
• Money can be shifted from monetary to non-monetary uses without any cost;
and
• (b) The metal can be coined into money without limit and without cost. There is
free coinage of standard money and the mint is open to the public. It is
unlimited legal tender and money of account of a country. Standard coins are
generally made of gold and silver.
Token coins • These refer to small coins of various denominations, which are issued to
facilitate day-to-day requirements of the people.
• All Indian coins, like those of Rs 10, 5, 2 or 1, are token coins since their value
as money is more than value of metal contained in them.
Q.17) Consider the following statements about the “Mini-Light Emitting Diodes” (Mini-LEDs):

1. Mini-LEDs allow the backlight to be divided into many more Local Dimming zones.
2. They are very small LEDs (0.2 mm or even smaller) that collectively produce the backlight on LCD
panels.
3. They are very much costlier to make due to complicated procedures than the OLED displays.
4. Micro-LEDs panels are an extension of OLED panels, on a more precise scale

Which of the statements given above are correct?


(a) 1, 2 and 3 only
(b) 1, 2 and 4 only
(c) 1, 3 and 4 only
(d) 2, 3 and 4 only

EXPLANATION

One individual diode can be less than a tenth of a centimetre. This super small ‘mini’ size allows the
backlight to be divided into many more Local Dimming zones than the regular LCDs. So, Statement 1
is correct.
Mini-LEDs are essentially very small LEDs that are around 0.2 millimetres in size or a bit smaller and
collectively produce the backlight on LCD panels. Micro-LEDs are even smaller in size. So, Statement
2 is correct.
Mini-LEDs bring the contrast of OLED panels and are easier to make than OLED or Micro-LED panels.
They are also cheaper than OLED displays. So, Statement 3 is not correct.
Micro-LED panels mean smaller individual LED panels in an OLED screen. Therefore micro-LEDs are
an extension of OLED panels, on a more precise scale. So, Statement 4 is correct.

ADDITIONAL INFORMATION

Mini-LEDs or Mini-Light Emitting Diodes

LCD • A traditional LCD (Liquid Crystal Display) display consists of a single backlight
Display that throws light on your screen.
• However, the contrast is controlled by something called an LCD matrix.
• The matrix selectively blocks out certain parts of the backlight to produce
contrast on the individual pixels.
• The single backlight ends up throwing some extra light to the true pitch of the
colour. This is called bleeding.
OLED • It has an individual pixel with its own lighting.
Display • This allows the screen to have granular control that adjusts which pixel is
supposed to show bright light and which one to show less light.
• OLEDs typically have a better contrast ratio compared to traditional LCDs.
Micro-LEDs • Micro-LED panels mean smaller individual LED panels in an OLED screen.
• It has a super-precise construction.
• So it is much costlier of all and cannot be used in our homes in near future.
Mini-LEDs • The backlight behind the LCD matrix could be divided into further parts is called
Local Dimming.
• This would significantly improve the contrast ratio on our LCD panel and can be
seen in premium LCDs.
• LCD with Local Dimming can have anywhere between dozens and a few hundred
backlight zones to improve contrast.
• Mini-LED displays will have a thousand backlight zones, allowing a lot more
granular control over that same contrast.
Importance • Give contrast as OLEDs but cheaper than that.
of Mini- • Easy to make than OLEDs and Micro-LEDs.
LEDs • Can be used to construct small displays like a notebook, tablet and
smartphones.
In Future • Mini-LEDs are on the expensive side now and would be affordable in near future.
• Today implemented in TVs and even laptops like the MSI Creator 17.
• Apple iPads and MacBooks of the future would be updated with this technology.

Q.18) Consider the following statements about the “World Database on Protected Areas”:

1. It contains the database on both terrestrial and marine protected areas.


2. It is a joint project of IUCN and the World Wide Fund for Nature.
3. Marine Protection Atlas has been built based on this database.

Which of the statements given above is/are correct?

(a) 1 only
(b) 1 and 3 only
(c) 2 and 3 only
(d) 1, 2 and 3

EXPLANATION:

The World Database on Protected Areas (WDPA) is the most comprehensive global database on
terrestrial and marine protected areas. It is a joint project between the United Nations Environment
Programme (UNEP) and the International Union for Conservation of Nature (IUCN), managed by UNEP
World Conservation Monitoring Centre (UNEP-WCMC) in collaboration with governments, non-
governmental organizations, academia, and industry. The WDPA is updated every month and the
protected planet report is related to this. So, Statement 1 is correct and 2 is not correct.
The Marine Protection Atlas (MPAtlas) released by the Marine Conservation Institute uses data reported
to the World Database on Protected Areas (WDPA) by applying science-based frameworks to
independently categorize and report MPAs and MPA zones. So, Statement 3 is correct.

ADDITIONAL INFORMATION:

CATEGORIES OF PROTECTED AREAS IN INDIA


SANCTUARY • An area of adequate natural,flora,fauna,ecological or
zoological importance.
• certain rights are permitted for people inside, as long as
they don’t harm the wildlife.
• Such activities are livestock grazing, collecting minor
forest produce, etc.
NATIONAL PARK • It is constituted for the protection of wildlife or its
environment, which has a higher degree of protection. No
human activities are allowed inside the parks.
COMMUNITY RESERVES • They are declared by the state government in any private
or community land.
• It is an area where the individuals or community, eager to
conserve the wildlife habitat.
CONSERVATION RESERVES • They are declared by the state governments.
• The aim is to protect landscapes, seascapes, flora, fauna,
and their protection.
• They act as a buffer zone between the parks, sanctuaries,
and protected forests.
Q.19) Phillips Curve describes the relationship between unemployment and inflation. Which of the
following is the relationship established by the curve?

1. Lower the inflation, higher the unemployment.

2. Higher the inflation, lower the unemployment.

3. Higher the inflation, higher the unemployment.

4. Lower the inflation, lower the unemployment.

Select the correct answer using the code given below.

(a) 1 only
(b) 3 only
(c) 1 and 2 only
(d) 3 and 4 only

EXPLANATION:

The Phillips curve is an economic concept developed by A. W. Phillips stating that inflation and
unemployment have a stable and inverse relationship. The theory claims that with economic
growth comes inflation, which in turn should lead to more jobs and less unemployment. Higher
inflation is associated with lower unemployment and vice versa, but the effects of this on the
economy are very short as it fails to explain the situation of stagflation in which both inflation and
unemployment will be at the alarmingly high. So, option (c) is correct.

ADDITIONAL INFORMATION:

INFLATION

BENEFITS • Deflation will harm the economy and can lead to lower
consumer spending and lower growth as it will delay their
purchasing power.
• Moderate inflation will help us to achieve and adjustable
to wages, to keep prices relative and stable, reduce the
real value of debt.
• It helps the labour market to reach equilibrium at a faster
pace.
CONCERNS • The effect of inflation is not evenly distributed on the
overall economy, there are chances for hidden costs.
• High inflation often leads to lower growth as the
consumer’s purchasing ability will reduce and be less
stable because it is not conducive for better budget
planning, which in turn creates lower investments and
uncertainities.
• It will reduce the country’s competitiveness at the global
level.
Q.20) The standard of living of the people of a country as a whole is determined mainly by

(a) The consumption expenditure of the people

(b) The Nation Income of the country

(c) The investment expenditure of the people

(d) The total expenditure on capital goods

EXPLANATION

Standard of living is the material well-being of the average person in a given population. It is typically
measured using GDP per capita. It could be determined mainly by the consumption expenditure of the
people, which reflects the expected development rate. consumption expenditure reflects the ability to
access the basic need of the people.
So, Option (a) is correct.
The improvement in the national income of the country will result in improvement in the growth of the
economy, it does not necessarily improve the development of the country. standard of living is related
to the development of the country to provide basic services necessary for the people.
So, Option (b) is not correct.
Investment expenditure of the people necessarily does not need to increase the standard of living of the
country as a whole. The standard of living is more dependent on the investment expenditure of the
government
So, Option (c) is not correct.
Total expenditure of capital goods does not increase the expenditure of people on consumer goods,
which are the basic needs of people to improve the standard of living
So, Option (d) is not correct.
ADDITIONAL INFORMATION

Standard of Living
Definition • Standard of living is the material well-being of the average
person in a given population.
• It is typically measured using the gross domestic product
(GDP) per capita.

Focus area • Income,


• Gross Domestic Product (GDP)
• Life expectancy
• Economic opportunity.

Features • Standards of living are usually higher in developed countries.

Q.21) Which of the following are included under factor cost?

1. Cost on Capital

2. Indirect taxes

3. Raw materials cost

4. Labour wages

5. Rent for Land

Select the correct answer using the code given below.

(a) 3, 4 and 5 only


(a) 1, 2, 3 and 4 only
(b) 1, 3, 4 and 5 only
(c) 1, 2, 3, 4 and 5

EXPLANATION

Factor cost includes the total cost incurred in deploying all factors of production, It is calculated based
on the cost incurred to produce the goods and services available in the market. The factors of
production include land, labor, entrepreneurship, and capital. It does not include Indirect taxes.
So, Option (c) is correct.
ADDITIONAL INFORMATION

Factor cost
Factor cost is the cost related to a factor of production. Factors of production are the inputs needed for
creating a good or service.
Factors of production • There are four factors of production
1. Land
2. Labor
3. Entrepreneurship
4. Capital
Land as a Factor • It takes on various forms like
1. Agricultural land
2. Commercial real estate
3. Resources available from a particular piece of
land
Labor as a Factor • The effort expended by an individual to bring a
product or service to the market

Capital as a Factor • It facilitates the processes used in the production


• It enables entrepreneurs and company owners to
purchase capital goods or land
Entrepreneurship as a Factor • It combines all the other factors of production into a
product or service for the consumer market

Q.22) An economy faces a low growth rate despite the high rate of capital formation. Which one of the
following is the reason for it?

(a) High Capital Output Ratio


(b) Low Capital Output Ratio
(c) Population explosion
(d) High dependence on imports

EXPLANATION

Incremental Capital Output Ratio (ICOR) is the additional capital required to increase one unit of
output. A lower ICOR value indicates the efficiency of the country’s production. When a country
fails to grow despite its high capital formation it means the country’s production is not efficient that
is it has a higher Capital Output Ratio. So, Option (a) is correct.

ADDITIONAL INFORMATION

Economic • Macro-economic shows us that capital investment is essential to drive


Growth economic growth

Incremental • ICOR refers to a unit of capital needed to increase one unit of output
Capital Output (growth).
Ratio • Lower ICOR indicates a country’s production is more efficient.
• Higher ICOR indicates a country’s production is less efficient.
Limitations • It favours developing countries that can increase their capital formation
by improving infrastructure and technology as opposed to developed
countries.
Higher ICOR • Following are some of the reasons that increase an ICOR value
o Delay in completion of projects
o Lack of complementary investment in related sectors
o Non-availability if critical inputs
India’s • In India, ICOR has increased from 3.8 in 2016-2017 to 4.9 in 2018 -
Situation 2019 and deteriorated further due to COVID to 6.9 in 2019-2020.

Q.23) Which one of the following parameters was recently added into the recent editions of the Human
Development Report, released by the United Nations Development Program?

(a) Per capita Wealth


(b) Crime Rate of a country
(c) Insolvency rate of a country
(d) Per-capita carbon emissions

OPTION ELIMINATION STRATEGY


Per capita income is already considered, therefore option (a) can be eliminated
Option (d) is more relevant as it projects the overall human development better than the other two.

EXPLANATION:

HDI is measured with three parameters namely life expectancy, education and per capita income. This
year's index has incorporated new parameters namely CO2 emission and Material footprint and
introduced Planetary pressures–adjusted Human Development Index (PHDI). to reflect the impact
caused by each country’s per-capita carbon emissions and its material footprint, which measures the
amount of fossil fuels, metals and other resources used to make the goods and services it consumes
So, Option (d) is correct.

ADDITIONAL INFORMATION:
Human Development Report
Human  The Human Development Report is an annual Human Development
Development Index Index report published by the Human Development Report Office of
the United Nations Development Programme
 Calculation of the index combines four major indicators: life
expectancy for health, expected years of schooling, mean of years of
schooling for education and Gross National Income per capita for
the standard of living. Every year UNDP ranks countries based on
the HDI report released in their annual report.
India's Status 2020
 India dropped two ranks in the United Nations’ Human Development
Index this year, standing at 131 out of 189 countries.
 However, if the Index were adjusted to assess the planetary pressures
caused by each nation’s development, India would move up eight places
in the ranking, according to the report.
 The absolute value of the index has gone up for India to 0.645 in 2019
(the year under consideration) compared to 0.642 the year before,
reflecting overall better performance.
 In all four HDI indicators, India’s performance has either improved in
2019 compared to 2018 or remained the same. Life expectancy at birth
has improved to 69.7 years compared to 69.4 years the year before. GNI
per capita at $6,681 in 2019 was higher than $6,427 in 2018. Expected
years of schooling and mean year of schooling in 2019 remained the
same as the previous year at 12.2 and 6.5 respectively

Q.24) Consider the following statements:

1. There is no minimum land area requirement for setting up a Special Economic Zone (SEZ) for
Information Technology enabled Services.

2. Both the Central and State Governments can allot land for SEZs.

3. All existing notified SEZ shall be deemed to be a multi-sector Special Economic Zone.

4. Among the States, Maharashtra has the highest number of SEZs in operation.

Which of the statements given above is/are correct?

(a) 1 and 3 only


(b) 2 and 4 only
(c) 1, 3 and 4 only
(d) 1, 2 and 3 only

EXPLANATION:
There shall be no minimum land area requirement for setting up an SEZ for IT/ITES, Biotech or
Health (other than a hospital) services but the minimum built-up processing area requirement shall
be applicable as per SEZ (3rd Amendment) Rules, 2019
So, Statement 1 is correct.
Under the Special Economic Zone Act, 2005, SEZ can be set up either jointly or severally by the
Central Government, State Government, or any person (including a private or public limited
company, partnership or proprietorship). Even a foreign company can also set up SEZ in India.
So, Statement 2 is correct.
The central government has notified that all notified and existing Special Economic Zones (SEZs)
shall be deemed to be multi-sector economic zones. This amendment to the SEZ rules of 2006
ensures units from two or more sectors can start operations in any, including trading and
warehousing.
So, Statement 3 is correct.
Tamil Nadu has the highest number of operational SEZs (40), followed by Karnataka (31), and
Maharashtra (30).
So, Statement 4 is not correct.
ADDITIONAL INFORMATION:
Special Economic Zones

Meaning  A special economic zone (SEZ) is an area in a country that is subject to


different economic regulations than other regions within the same
country. The SEZ economic regulations tend to be conducive to—and
attract—foreign direct investment (FDI).

Objectives
 The main objectives of the SEZ Scheme are the generation of additional
economic activity, the promotion of exports of goods and services, the
promotion of investment from domestic and foreign sources, the
creation of employment opportunities along the development of
infrastructure facilities.
Benefits

 companies may be offered tax holidays, where upon establishing


themselves in a zone, they are granted a period of lower taxation.
 The creation of special economic zones by the host country may be
motivated by the desire to attract foreign direct investment (FDI).
 The benefits a company gains by being in a special economic zone may
mean that it can produce and trade goods at a lower price, aimed at
being globally competitive.
Administration  All laws of India are applicable in SEZs unless specifically exempted as
per the SEZ Act/ Rules. Each Zone is headed by a Development
Commissioner and is administered as per the SEZ Act, 2005 and SEZ
Rules, 2006. Units may be set up in the SEZ for manufacturing, trading
or for service activity.

Q.25) Consider the following statements about the “Anti-Microbial Resistance”

1. It is the phenomenon by which bacteria and fungi evolve and become resistant to presently
available medical treatment.
2. It can spread from person to person or between people and animals, including from food of animal
origin.
3. The Chennai Declaration calls for urgent initiatives to formulate an effective national policy to
control the rising antimicrobial resistance, including a ban on the over-the-counter sale of
antibiotics.

Which of the statements given above is/are correct?

(a) 1 only
(b) 1 and 2 only
(c) 2 and 3 only
(d) 1, 2 and 3

EXPLANATION

Antimicrobial Resistance (AMR) occurs when bacteria, viruses, fungi, and parasites change over time
and no longer respond to already available medicines making infections harder to treat. So,
Statement 1 is correct.
AMR can occur naturally over time and it can spread from person to person or between people and
animals, including from food of animal origin. The main cause of spread is misuse and overuse of
antimicrobials, lack of access to clean water for both animals and plants. So, Statement 2 is
correct.
“The Chennai Declaration” is the result of the first ever joint meeting of medical societies in India
addressing antibiotic resistance. It calls for urgent initiatives to formulate an effective national policy
to control the rising antimicrobial resistance, including a ban on the over-the-counter sale of
antibiotics, and to bring about changes in the medical education curriculum to include training in
antibiotic usage and infection control. So, Statement 3 is correct.

ADDITIONAL INFORMATION

Antimicrobial Resistance

Antimicrobials • It includes antibiotics, antivirals, antifungals, and antiparasitics that are used
to prevent and treat infections in humans, animals, and plants.
Antimicrobial • It occurs when bacteria, viruses, fungi, and parasites change over time and no
Resistance longer respond to medicines making infections harder to treat and increasing
the risk of disease spread, severe illness, and death.
• As a result of this antimicrobial medicines become ineffective and unable to
treat the disease and increase the risk.
Global Threat • Antimicrobial resistance (AMR) is a global health and development threat.
• WHO has declared that AMR is one of the top 10 global public health threats
facing humanity.
• It requires urgent multisectoral action to achieve the Sustainable
Development Goals (SDGs).
Global action • WHO formulated a global action plan with the following strategies.
plan on • To improve awareness.
antimicrobial • To strengthen surveillance and research.
resistance • To reduce the incidence of infection.
• To optimize the use of antimicrobial medicines.
• To ensure sustainable investment in countering antimicrobial resistance.
Global • WHO launched GLASS in 2015.
Antimicrobial • It provides a standardized approach to the collection, analysis,
Resistance interpretation, and sharing of data by countries, territories, and areas, and
Surveillance monitors the status of existing and new national surveillance systems, with
System emphasis on representativeness and quality of data collection.
(GLASS)
Global • It is a joint initiative of WHO and the Drugs for Neglected
Antibiotic Diseases Initiative (DNDi).
Research and • It encourages research and development through public-private partnerships.
Development • It aims to develop and deliver five new treatments within 2025 that target
Partnership drug-resistant bacteria identified by WHO which poses the greatest threat.
(GARDP)
Indian • Indian government launched National Programme on AMR Containment
Initiative during the 12th five-year plan. Its objective is
• Establish a laboratory-based AMR surveillance.
• Strengthen infection control practices and promote rational use of
antimicrobials through Antimicrobial stewardship activities.
• Generate awareness and rational use of antimicrobials.
Red Line • India’s idea is to mark a red line on antibiotic usage to curb their over-the-
Campaign counter sale.
• It can be used globally against the threat of superbugs.

Q.26) With reference to foreign trade, which one of the following is mainly considered as indicative of the
structure and level of development of an economy?

(a) Volume of trade

(b) Composition of trade

(c) Direction of trade

(d) Terms of trade

EXPLANATION

The volume of trade relates to the size of transactions involved in the trade. It calculates the share
of imports and exports in GDP. So, Option (a) is not correct.

The composition of trade gives a detailed description of commodities of import and commodities of
export of a country. Developing countries export raw materials, agricultural products and
intermediate goods; developed countries export finished goods, machines, equipment and
technique. So, Option (b) is correct.

The direction of trade means a study of the countries to whom the exports are made and from
whom the imports are made. So, Option (c) is not correct.

Terms of trade (TOT) represent the ratio between a country's export prices and its import prices. So,
Option (d) is not correct.

ADDITIONAL INFORMATION

Trade • Exchange of goods and services in an economy


• Can be a domestic trade or a foreign trade

Trade & • Trade has a positive and significant impact on economic growth
Economic • One percentage rise in the average trade to GDP ratio leads to an
growth increase in the average GDP per capita growth by about one-half
(0.47) percentage point.

India’s
trade

Q.27) Consider the following statements regarding the concept of the “Circular Carbon Economy”:

1. It was recently endorsed by the G20 as a way to promote economic growth.


2. It is transforming an industrial waste product into a valuable new resource, reducing the impact of
carbon emissions on our planet.
3. It can eliminate almost half of those remaining greenhouse gas emissions by capturing and reusing
them.

Which of the statements given above is/are correct?

(a) 1 only
(b) 1 and 2 only
(c) 2 and 3 only
(d) 1, 2 and 3

EXPLANATION

Recently, G20 Energy Ministers endorsed the Circular Carbon Economy framework (CCE) in the Riyadh
Summit aiming for Sustainable Development. Circular Carbon Economy contributes to economic
growth by reusing and recycling the resources. So, Statement1 is correct.
The circular Carbon Economy framework (CCE) works on four main principles that are to reduce,
reuse, recycle and remove carbon emissions from our planet. So, Statement 2 is correct.
The circular carbon economy can eliminate almost half of those remaining greenhouse gas emissions
by capturing and reusing a projected 9.3 billion metric tonnes of CO2 by 2050. So, Statement 3 is
correct.

ADDITIONAL INFORMATION

Circular Carbon Economy

CCE • The Circular Carbon Economy framework (CCE) aims to provide sustainable
Objective development by providing clean and affordable energy to all.
Salient • It aims to provide inclusive development that addresses CO2 emissions.
Features • It paves the path to achieving the goals of climate change.
• It enables the countries to share the technologies among them to make the best
use of available resources.
Principles • It encompasses the 4R approach to managing carbon emissions.
1) Reduce - By using energy-efficient technologies and also making use of
renewables, hydropower, nuclear, and bioenergy.
2) Reuse – By employing methods that can reuse carbon in feedstocks,
fuels, and other uses.
3) Recycle – By transforming CO2 into other useful products like fertilizer,
cement, or other synthetic fuels.
4) Remove – By capturing and storing CO2 by direct air capture and
increasing afforestation activities.
Endorsement • Recently, G20 endorse the circular carbon economy to enable a climate-friendly
approach.
• Saudi Arabia and Aramco have also adopted the CCE framework to reduce
carbon footprints.

Q.28) Consider the following parameters about the Indian Economy in a particular year.

1. Consumption in the Economy is 100 lakhs


2. Investment in the Economy is 50 Lakhs
3. Government expenditure is 20 lakhs
4. Export from India is 20 lakhs
5. Import from foreign countries is 10 lakhs

By considering the above-given values, what is the Gross National Product of India in that year?

(a) 200 lakhs

(b) 180 Lakhs

(c) 100 Lakhs

(d) 80 Lakhs

EXPLANATION

Gross National Product (GNP) is the total value of all finished goods and services produced by a
country’s citizens in a given financial year, irrespective of their location.
GNP = C + I + G + X + Z
Where C is Consumption,
I is Investment,
G is government Expenditure,
X is net Exports,
Z is net Income.
GNP = 100+50+20+(20-10)
GNP = 180 Lakhs.

So, Option (b) is correct.

ADDITIONAL INFORMATION

Gross National Product


GNP • The total value of the total output or production of final
goods and services produced by the nationals of a
country in a given period.
• GNP includes the income earned by Indian Nationals
only.
Formula to calculate GNP • GNP = GDP + (X-M)
• GNP = C + I + G + X + Z
Importance of GNP • Act as one of the important economic indicators
• Economists rely on this data to solve national problems
• Helps in analyzing the balance of payments

Q.29) Consider the following statements:

1. If Per Capita Income rises, it means that on average, people are getting more goods and services.

2. It is necessary that when National Income rises, Per Capita Income also rises.

Which of the statements given above is/are correct?

(a) 1 only

(b) 2 only

(c) Both 1 and 2

(d) Neither 1 nor 2

EXPLANATION

If per capita income rises, it means that on average there is more income available to spend on
incurring goods and services. Thus people are getting more goods and services with a rise in Per Capita
Income.
So, Statement 1 is correct.
Per capita income doesn't need to rise when National Income rises. Because per capita income also
depends on the population of the country. Per capita income will not rise along with a rise in national
income unless there is also a percentage change in population.
So, Statement 2 is not correct.
ADDITIONAL INFORMATION

Per Capita Income • It is a measure of the amount of money earned per person
in a nation or geographic region
Calculation • Dividing the overall income of the population by the total
number of people.
Uses • It helps to determine the average per-person income
• To evaluate the standard of living for a population
Limitation • It doesn't always provide an accurate representation of
the standard of living
• It does not reflect inflation in an economy
• Not convenient for international comparisons
• It does not exclude children while measuring, who is a
dependant population
• It does not include individuals saving or wealth

Q.30) With reference to the methods of estimating National Income, the performances of different sectors
of an economy was revealed by

(a) Production method

(b) Income distribution method

(c) Expenditure method

(d) All the three methods

EXPLANATION

In the Production method to compile the national income the gross value of final goods and services
produced in a country is calculated. This method is used to estimate the performances of different
sectors of an economy. It is easy to count and multiple the products produced in different sectors,
hence helping in estimating their performance.
So, Option (a) is correct.
In Income distribution methods incomes of factors of production is compiled. Here the source of income
could not be compiled, so could not estimate the performance of different sectors of an economy.
So, Option (b) is not correct.
In the Expenditure method, national income is measured as a flow of expenditure. Here expenditure
incurred from a particular sector could not be estimated differently. So, could not estimate the
performance of different sectors of an economy.
So, Option (c) is not correct.

ADDITIONAL INFORMATION

Estimation of national income


National income • NNP at factor cost calculates national income
based on the cost incurred to produce the goods
and services.
Method of measuring National Income • Measured by three different methods
1. Product or output method
2. Income method
3. Expenditure or consumption method
Product method • The money value of all final goods and services
produced in an economy during a year
• Final goods refer to goods that are directly
consumed
Income method • It is calculated by compiling income of factors of
production
• National income = Total wages +total rent + Total
interest + Total profit
Consumption method • National income is measured as a flow of
expenditure.
• GDP =C+I+G
• C- Consumption expenditure of consumer
• I - Consumption expenditure of investor
• G - Government consumption

Q.31) The National Statistical Office coordinates the statistical activities in the country. Its activities
include the compilation of

1. National Accounts

2. Index of Industrial Production

3. Consumer Price Indices

4. Gender Statistics

5. Economic Census

Select the correct answer using the code given below.

(a) 1, 3 and 5 only


(b) 1 and 3 only
(c) 2, 4 and 5 only
(d) 1, 2, 3, 4 and 5

EXPLANATION:

The National statistical office releases various reports such as National account statistics, the
conduct of annual survey of industries and economic census, Index of Industrial production as well
as the consumer price index, human development and gender statistics coordinates the statistical
work in respect of the Ministries/Departments of the Government of India and State Statistical
Bureaus (SSBs), advises the Ministries/Departments of the Government of India on statistical
methodology and statistical analysis of data. So, option (d) is correct.

ADDITIONAL INFORMATION:

NATIONAL STATISTICAL OFFICE

RECENT CHANGES • The government of India under the ministry of


statistics and program implementation has now
merged the CSO(central statistical office) and
NSSO(national sample survey organization).

OBJECTIVES • It aims to have greater synergies amongst the


available statistical manpower and improve data
quality not only for survey data but also for
administrative statistics.
• To monitor India’s progress in attaining the
Sustainable Development Goals (SDGs) by 2030,
MoSPI has formulated a National Indicator
Framework which requires significant data to flow
from the field formations and requires extensive
use of technology. This requires active and
continuous involvement of State Governments,
and the field presence of NSS in the States, with
their knowledge and experience of official
statistics system, can play a proactive role in this
process, in addition to their existing functions.
• The internal restructuring of MoSPI is to
strengthen the national statistical system while
maintaining its autonomy.

Q.32) Consider the following statements about the “National Statistical Commission”:

1. It was set up based on the recommendation of the Rangarajan Commission.

2. It has a full-time Chairperson and four part-time Members.

3. The maximum tenure of a member is five years.

4. Chief Executive Officer of the NITI Aayog is the ex-officio member of the Commission.

Which of the statements given above is/are correct?

(a) 1 and 4 only


(b) 2 and 3 only
(c) 1, 2 and 4 only
(d) 1, 2, 3 and 4

EXPLANATION:

The National Statistical commission was set up in 2005 with a mandate to evolve policies,
priorities, and standards in statistical matters by a government resolution, based on the
recommendations of the Statistical commission headed by the Rangarajan. Recently, the ministry
has drafted the statistical bill for public comments to make it a statutory body. So, Statement 1 is
correct.

The Commission has a part-time Chairperson, four part-time Members, and an ex-officio Member.
The Chief Statistician of India, the post created specifically as the Head of the National Statistical
Office is the Secretary of the Commission. He is also the Secretary to the Government of India in
the Ministry of Statistics and Programme Implementation. So, Statement 2 is not correct.

The Chairperson and the Members also enjoy relative security of tenure as once they assume
office, they can be removed only by the President after the Supreme Court of India has on inquiry
held by the procedure Article 145 of the Constitution of India reported that they ought to be
removed. So, Statement 3 is not correct.

Apart from the chairperson and members, NITI AAYOG’s chief executive officer is also one of the ex-
officio members. So, Statement 4 is correct.

Q.33) With reference to the Indian economy, the Household Financial Savings constitute

1. Currency

2. Bank deposits

3. Debt securities

4. Mutual funds

5. Insurance

6. Investments in small savings schemes

Select the correct answer using the code given below.

(a) 1, 2 and 6 only


(b) 3, 4 and 5 only
(c) 1, 2, 3, 4 and 5 only
(d) 1, 2, 3, 4, 5 and 6

EXPLANATION:

The investment in financial saving comprises currency, net deposits, shares and bonds, net claims on
government in the form of small savings, investment in Central & State government securities, life
insurance funds and provident & pension funds, debt securities, mutual funds, etc. The above all will
be referred to as gross household savings but if you exclude the loans borrowed we can get Net
household financial savings. So, option (d) is correct.

ADDITIONAL INFORMATION:

HOUSEHOLD FINANCIAL SAVINGS


COMPRISES OF • The household sector comprises, apart from
individuals, all non-government, non-corporate
enterprises like sole proprietorships and
partnerships owned and/or controlled by
individuals and non-profit institutions which
furnish educational, health, cultural, recreational,
and other social and community services to
households.
• The saving of the household sector is taken as the
sum of its investments in various instruments of
financial saving and the form of physical assets.
Since direct annual data on household saving as
an excess of income over current expenditure of
households and household enterprises are not
available, the saving of the household sector is
worked out by following the residual method.

CURRENT LEVEL • The preliminary estimate of household financial


savings is placed at 8.2 percent of GDP in 2020-
21,

Q.34) Consider the following statements about the Multidimensional Poverty Index (MPI):

1. MPI uses income as the key parameter in measuring poverty while considering various deprivations
experienced by people in their daily lives including poor health, insufficient education and a low
standard of living.
2. MPI uses 10 indicators in three dimensions with each indicator contributing equally to the
calculation of the deprivation score.
Which of the statements given above is/are correct?

(a) 1 only
(b) 2 only
(c) Both 1 and 2
(d) Neither 1 nor 2

EXPLANATION

The MPI value summarizes information on multiple deprivations into a single number. It is
calculated by multiplying the poverty headcount by the intensity of poverty. It measures the
deprivation of people in health, education, and standard of living. The national multidimensional
poverty index is released by NITI Aayog.
So, Statement 1 is not correct.

MPI uses 10 indicators in three dimensions but each indicator does not contribute equally to the
calculation instead it depends on the weightage given to each indicator. Indicators like Years of
schooling and child enrollment 1/6 weightage each, Child mortality and nutrition 1/6 weightage
each, Electricity, drinking water, sanitation, cooking fuel and assets 1/18 weightage each.
So, Statement 2 is not correct.

ADDITIONAL INFORMATION

Multidimensional Poverty Index (MPI)

MPI is based on the idea that poverty is not unidimensional not just depends on income and
one individual may lack several basic needs like education.
Published by • The United Nations Development Programme
(UNDP)
• The Oxford Poverty & Human Development
Initiative (OPHI)
• Launched in 2010
Indicators and Dimensions • Health
1. Nutrition
2. Child mortality
• Education
1. Years of schooling
2. School Attendance
• Living Standards
1. Cooking oil
2. Sanitation
3. Drinking water
4. Electricity
5. Housing
6. Assets
Q.35) Who had penned the book ‘the Planned Economy of India’ in 1934?

(a) Subash Chandra Bose


(b) Jawaharlal Nehru
(c) Visvesvaraya
(d) MN Roy

EXPLANATION:

In 1934, Sir M. Visvesvaraya had published a book titled “Planned Economy in India”, in which he
presented a constructive draft of the development of India in the next ten years. His core idea was
to lay out a plan to shift labour from agriculture to industries and double up National income in ten
years.
So, Option (c) is correct.

ADDITIONAL INFORMATION:
Plan proposals before Independence
National Planning
Committee This is the first attempt to develop a national plan for India in 1938 when
Congress President Subhash Chandra Bose had set up a National Planning
Committee with Jawaharlal Nehru as its president. However, the reports of
the committee could not be prepared properly.

Bombay Plan This plan envisaged doubling the per capita income in 15 years and tripling
the national income during this period. Nehru did not officially accept the
plan, yet many of the ideas of the plan were inculcated in other plans which
came later.

Eight Industrialists of Bombay viz. Mr JRD Tata, GD Birla, Purshottamdas


Thakurdas, Lala Shriram, Kasturbhai Lalbhai, AD Shroff, Ardeshir Dalal, &
John Mathai working together prepared “A Brief Memorandum Outlining a
Plan of Economic Development for India” in 1944. This is known as the
“Bombay Plan”.
People’s Plan The nationalization of all agriculture and production was the main feature of
this plan. This plan was based on Marxist socialism and drafted by M N Roy on
behalf of the Indian federation of Lahore.
Gandhian Plan This plan was drafted by Sriman Nayaran, principal of Wardha Commercial
College. It emphasized the economic decentralization with primacy to rural
development by developing the cottage industries.
Sarvodaya Plan Sarvodaya Plan (1950) was drafted by Jaiprakash Narayan. This plan itself was
inspired by the Gandhian Plan and the Sarvodaya Idea of Vinoba Bhave. This
plan emphasized agriculture and small & cottage industries. It also suggested
the freedom from foreign technology and stressed land reforms and
decentralized participatory planning

Q.36) With reference to the concept of economic growth, an increase in the per capita real income implies
that

(a) the rate of increase in real national income should be lesser than the growth rate of the population

(b) the rate of increase in real national income should be higher than the per capita consumption

(c) the rate of increase in real national income should be lesser than the per capita consumption

(d) the rate of increase in real national income should be higher than the growth rate of the
population

EXPLANATION :

Per capita income for a nation is calculated by dividing the country's national income by its
population.

Per Capita Real Income = Real National Income


Population

Note : Real national income is nominal or money national income (output) adjusted for
inflation. It is also national income at 'at constant prices.
So, the above formula implies that, for increase in per capita national income, the rate of
increase in real national income (Numerator) must be greater than the rate of increase in
Population (denominator).
So, option (d) is correct.

ADDITIONAL INFORMATION :
Per capita Income :

Definition • Per capita income is a measure of the amount of money earned per person in a
nation or geographic region.
• Per capita income can be used to determine the average per-person income for
an area and to evaluate the standard of living and quality of life of the
population.
• Per capita income for a nation is calculated by dividing the country's national
income by its population.
Significance • Per capita income is also useful in assessing an area's affordability. It can be
used in conjunction with data on real estate prices, for instance, to help
determine if average homes are out of reach for the average family.
• Businesses can also use per capita income when considered opening a store in a
town or region. If a town's population has a high per capita income, the company
might have a better chance at generating revenue from selling their goods since
the people would have more spending money versus a town with a low per capita
income.
• Per capita income counts each man, woman, and child, even newborn babies, as
a member of the population.
• This stands in contrast to other common measurements of an area's prosperity,
such as household income, which counts all people residing under one roof as a
household, and family income, which counts as a family those related by birth,
marriage, or adoption who live under the same roof.
Limitations • Since per capita income uses the overall income of a population and divides it by
the total number of people, it doesn't always provide an accurate representation
of the standard of living.
• In other words, the data can be skewed, whereby it doesn't account for income
inequality.
• Per capita income doesn't reflect inflation in an economy, which is the rate at
which prices rise over time.
• The cost of living differences can be inaccurate when making international
comparisons since exchange rates are not included in the calculation.
• Per capita income doesn't include an individual’s savings or wealth.
• Per capita includes children in the total population, but children don't earn any
income.
• The welfare of the people isn't necessarily captured with per capita income. For
example, the quality of work conditions, the number of hours worked, education
level, and health benefits are not included in per capita income calculations.

Q.37) Which of the following factors are considered as the important determinants of the economic
development of a country?

1. Capital formation

2. Capital-Output ratio

3. Occupational Structure

4. Growth of Population

Select the correct answer using the code given below.

(a) 1 and 4 only


(b) 2 and 3 only
(c) 1, 2 and 3 only
(d) 1, 2, 3 and 4

EXPLANATION

Economic development is determined by various economic and non-economic factors. Economic


factors such as Population, Natural Resources, Capital formation and capital accumulation,
Capital-Output Ratio, Investment Pattern, Occupational Structure, Technological Advancement
and Development Planning favours growth in the economy which in turn provides for economic
development. So, Option (d) is correct.
ADDITIONAL INFORMATION

Economic Development

Economic • Economic development is a process of raising the rate of capital


Developme formation that is both physical capital and human capital.
nt • It is concerned with structural changes in the economy.
• It is influenced by several factors like economic, political, social, and
administrative and so on.
• It depends on both economic and non-economic factors.
Classificat Economic factors Non-economic factors
ion
• Population • Education
• Natural Resources • Changes in Social and
• Capital formation and Institutional Factors
capital accumulation • Proper Maintenance of
• Capital-Output Ratio Law and Order
• Investment Pattern • Administrative Efficiency
• Occupational Structure • Cultural set-up
• Technological • Political Legal
Advancement Environment
• Development Planning
Capital • It is a process by which savings of an economy is used in investment of
Formation capital formation by infrastructural and technological development.
• Income is not entirely spent on consumption and used in investment
which triggers growth in an economy.
Savings ratio
Capital- • Rate of growth of GNP =
Capital output ratio
Output • Thus to achieve a higher rate of growth a country has to
Ratio 1. Raise the rate of investment
2. Generate necessary forces for reducing the capital-output ratio
Occupatio • To attain a high rate of economic development, inter-sectoral transfer of
nal the workforce is very much necessary.
Structure • The rate of economic development and the level of per capita income
increase as more and more workforce shift from the primary sector to
the secondary and tertiary sector.
Growth of • Population growth can both contribute and act as a hurdle to economic
Population growth.
• Contribute by providing labour and entrepreneurship.
• A galloping population may increase demand in the economy affecting
the balance of trade and may also trigger the unemployment problem.

Q.38) Consider the following statements:

1. Changing structure of GDP in India has been accompanied by changing structure of the workforce.

2. There has been a clear shift in the workforce from primary to secondary and tertiary sectors in the
Indian Economy since 1951.

Which of the statements given above is/are correct?

(a) 1 only
(b) 2 only
(c) Both 1 and 2
(d) Neither 1 nor 2

EXPLANATION

In India after independence primary sector contributed 52% of GDP growth with 64.6% of the
workforce participating in it, the secondary sector contributed for only 9% of GDP growth with
10.4% of the workforce participating in it, the tertiary sector contributed 34.6% of GDP growth
with 20.8% of the workforce participating in it. But as of now, there has been a drastic change
in the contribution made by each sector to GDP growth with the tertiary sector contributing to
67% but with just 26.8% of the workforce participating in it, still keeping the workforce of the
primary sector at maximum with 48.9%. So, Statement 1 is not correct.

There is only a gradual shift in the workforce since Independence. Since 1951, the primary
sector has been the major contributor to the workforce, after which the tertiary sector
contributed to a higher workforce than the secondary sector and this trend can be observed
even today. So, Statement 2 is not correct.

ADDITIONAL INFORMATION
Q.39) Consider the following statements about the “Patent Act of 1970”:

1. It was enacted to mainly remove the monopolies over pharmaceutical drugs.


2. It allowed generic manufacturers in India to grow and as a result, life-saving drugs were available
at affordable prices.
3. But with the advent of the TRIPS agreement in 1995, the production of generic medicines by the
developing countries was banned.

Which of the statements given above is/are correct?

(a) 1 only
(b) 1 and 2 only
(c) 2 and 3 only
(d) 1, 2 and 3

EXPLANATION

Indian Patents and Design act of 1911 granted the patent to both product and process invention in the
pharmaceutical sector which lead to monopolies of that sector and the product price was high. To
overcome this, methods or process in the pharmaceutical sector alone was made patentable in Patent
Act 1970. So, Statement 1 is correct.
As stated above Patent Act 1970 was enacted which laid the foundation for India’s generic drug
industry as a result life-saving drugs were available at affordable prices. So, Statement 2 is correct.
TRIPS agreement gives protection to developing nations for accessing essential medicines at affordable
cost by the use of generic drugs. So, Statement 3 is not correct.

ADDITIONAL INFORMATION

Patent in India

Patent • A patent is a form of preservation of intellectual property.


• It is an exclusive right granted for an invention,
• It can be for a product or a process that provides, in general, a new way of
doing something, or offers a new technical solution to a problem.
Criteria for • It should be novel.
an Invention • Must involve an inventive step (technical advancement)
• Capable of industrial application
Term • 20 years from the date of filling
• For a product - the patentee has a right to prevent others from making, using,
offering for sale, selling, or importing the patented product in India.
• For a process - the patentee has the right to prevent others from using the
Rights process, using the product directly obtained by the process, offering for sale,
granted selling, or importing the product in India directly obtained by the process.

• Patents (Amendment) Act, 2005


1) Product patent was extended to all fields of technology.
2) Exclusive Marketing Rights (EMRs) have been repealed, and the
Patents Act compulsory license has been introduced.
1970 • Patents (Amendment) Rules, 2021
1) Patent Fees for Educational Institutions Reduced by 80%
2) Expedited Examination system
i. Initially provided only for Startups
ii. Now extended to 8 more categories
• Elongating the term of a granted patent in a jurisdiction that is about to expire,
Evergreening in order to retain royalties from them, by taking out new patents.
• Indian Patent Act does not allow evergreening of patents for inventions
of Patent involving new forms of a known substance unless it differs significantly in
properties about efficacy.
• CL is the grant of permission by the government to entities to use,
Compulsory manufacture, import, or sell a patented invention without the patent owner's
Licencing consent.
(CL) • Patents Act in India deals with CL.
• CL is permitted under the WTO’s TRIPS (IPR) Agreement provided conditions
such as ‘national emergencies, other circumstances of extreme urgency and
anti-competitive practices’ are fulfilled.

Q.40) If a country is placed under the Financial Action Task Forces’ Greylist, then what does it indicate?

(a) It indicates a country where the business risk is high and the country is not suitable for a
conducive business environment
(b) It indicates a country is at higher risk of money laundering and terrorism financing
(c) With this notification, the country is not allowed to permit the Foreign Direct Investment
(d) With this declaration, the international aid agencies will automatically stop their aids to that
country

EXPLANATION:

If any country supports money laundering and terror financing, they will come under the Greylist
(high-level monitoring) category of the Financial Action Task Force. When the FATF places a
jurisdiction under increased monitoring, it means the country has committed to resolving swiftly the
identified strategic deficiencies within agreed timeframes and is subject to increased monitoring.
Although no official consequences follow, these countries are restricted access to international loans,
and also the listed countries' financial relations with other countries will get affected due to FATF
rankings such as foreign direct investment and portfolio inflows will hit. So, option (b) is correct.

The FATF also consists of BLACK LIST (High-level jurisdictions subject to call for action) in that the
countries are considered non-cooperative about encouraging money laundering and terrorist activities.
Once the country is included in this list, it will get economic sanctions and prohibitive measures from
FATF like FDI is not allowed inside the country and also NGO’S or any International aid agencies can’t
able to further aid to the country. Currently, only two countries are in this black list are Iran and North
Korea.

ADDITIONAL INFORMATION:

FINANCIAL ACTION TASK FORCE


ABOUT • FATF is an intergovernmental organization set up in 1989
during the G7 summit in Paris.
OBJECTIVES • The main objectives of the FATF are to establish norms and
standards of "legal, regulatory, and operational measures" to
fight against money laundering, terrorist financing, and other
related threats to the security and integrity of the
international financial system.
HEADQUARTER • Its secretariat is located at the OECD (organization for
economic cooperation and development) office in Paris.
MEMBERS • Currently, consists of 39 members in that INDIA is also a
member and the European Commission, Gulf Cooperation
Council.
SESSIONS • The FATF plenary is the decision-making body of it and they
meet 3times a year.
LISTS • GREYLIST
• BLACKLIST

Q.41) Consider the following statements about the concept of “Usual Status Unemployment”:

1. Its rates could be regarded as a measure of chronic unemployment during the reference year.

2. It is the more comprehensive concept as compared to underemployment on weekly basis as well as


current daily status unemployment.

3. It does not give the number of unemployed persons.

Which of the statements given above is/are correct?

(a) 1 and 2 only


(b) 2 and 3 only
(c) 1 only
(d) 1, 2 and 3

EXPLANATION

Usual Status Unemployment records persons who are unemployed for a major time during a
reference period of 365 days preceding the survey conducted by Periodic Labour Force. As it
calculates for a year it calculates the chronic unemployment that is prolonged unemployment in an
economy. So, Statement 1 is correct.

As Usual Status Unemployment captures long-term unemployment it calculates only the broad
aspects of unemployment in an economy rather comprehensively covering all factors compared to
current weekly status and current daily status. So, Statement 2 is not correct.

Usual Status Unemployment calculates the number of persons employed, unemployed during and
persons who are not in the labour force during a reference period of one year. So, Statement 3 is
not correct.

ADDITIONAL INFORMATION

Unemployment

Unemployment • It is a situation in which individuals are ready and willing to work but are
unable to find a job.
• Number of unemployed = Labour force – Workforce
Survey • National Statistical Office (NSO) launched the Periodic Labour Force
Survey (PLFS) to estimate the key employment and unemployment
indicators such as Worker Population Ratio, Labour Force Participation
Rate, Unemployment Rate.
Unemployment • Activity Status – in the following states during the reference period
Estimates o (Employed/workforce) - working or being engaged in economic
activity
o Unemployed - Not engaged in economic activity but available for
work
o Not in the labour force - Not engaged in economic activity and
also not available for work
Approaches to • Chronic or “Usual Principal Status”(US)
determine • Current Weekly Status(CWS)
Activity Status • Current Daily Status(CDS)

Differences Chronic or Usual Current Weekly Current Daily Status


Principal Status Status

The reference period The reference period The reference period is one
is one year is one week week

Employed – Major Employed - One Employed - 4 hours or


time spent on work hour a week more during a day
Employed for half-day – 1
hour to 4 hours

Unemployed – In Unemployed - In Unemployed – Not engaged


labour force but not labour force but not in work even for an hour
the major time is the spent an hour
spent on work on work

Not in the labour Not in the labour Not in the labour force –
force – Major time of force –Not working Neither had any work to do
reference period not and not available for nor available for work
working and not work at least for an
available for work hour

Q.42) Which one of the following statements best explains the term “Secular stagnation”?

(a) It is a condition when there is negligible or no economic growth in a market-based economy

(b) It is a condition where political uncertainty is combined with the economic stagnation

(c) It is a condition when stagnant economic growth, high unemployment and high inflation combine

(d) It is a condition where social unrest is combined with the economic stagnation

EXPLANATION :

In economics, secular stagnation is a condition when there is negligible or no economic growth in a


market-based economy.
In this context, the term secular means long-term.
So, option (a) is correct.

ADDITIONAL INFORMATION :
Secular • The term “secular stagnation” refers to a state of little or no economic growth –
Stagnation in other words, an environment where the economy is essentially stagnant.
• “Secular” in this context simply means “long term.”
• The term was coined by Alvin Hansen in the 1930s, during the Great
Depression, and was revived largely by Lawrence Summers.
• The primary tenets of the theory are that the lack of investing, due to an
increased tendency toward saving, and the lack of an aggressive (i.e., high
spending) government fiscal policy are the primary causes of a state of
economic stagnation, where there is little discernible economic growth.
• A sluggish economy is an economy that is experiencing little or no
macroeconomic growth.
Economic • Economic stagnation is a prolonged period of slow economic
Stagnation growth (traditionally measured in terms of the GDP growth), usually
accompanied by high unemployment.
• Under some definitions, "slow" means significantly slower than potential growth
as estimated by macroeconomists, even though the growth rate may be
nominally higher than in other countries not experiencing economic stagnation.
Stagflation • Stagflation is characterized by slow economic growth and relatively high
unemployment—or economic stagnation—which is at the same time
accompanied by rising prices (i.e. inflation).
• Stagflation can be alternatively defined as a period of inflation combined with a
decline in the gross domestic product (GDP).

Q.43) Consider the following receipts of the Government of India:

1. Tax revenue receipts


2. Non-tax revenue receipts
3. Recoveries of loans
4. Market loans

Which of the receipts given above is/are non-debt creating in nature?

(a) 3 only
(b) 1 and 2 only
(c) 1, 2 and 3 only
(d) 4 only

EXPLANATION:

The Central government receipts can broadly be divided into Non-debt and debt receipts.
The Non-debt receipts comprise tax and Non-Tax revenue, and Non-debt Capital receipts like recovery
of loans and disinvestment receipts. Debt receipts mostly comprise market borrowings and other
liabilities, which the government is obliged to repay in the future. So, option (c) is correct.

ADDITIONAL INFORMATION:

TYPES OF RECEIPTS
REVENUE • Direct Tax revenue consists of income tax and
corporate tax and Indirect tax consists of excise
and customs etc.
• Non-tax revenue consists of dividends, profits,
interest receipts, fees, fines, other fiscal, and
economic services, grants in aid, etc.
CAPITAL • Disinvestments, repayment of loans, etc.
Q.44) Which one of the following is correct about the Tax-to-GDP ratio?

(a) A rise in Tax-to-GDP ratio necessarily means that income is accruing more to the lower-income
population
(b) A rise in GDP will necessarily be followed by a rise in Tax-to-GDP ratio
(c) Lowering the tax rates can help in raising the Tax-to-GDP ratio.
(d) A rise in tax base can result in an increase in the Tax-to-GDP ratio even when the tax rate is
kept constant

EXPLANATION:

The Tax to GDP ratio is the ratio comparing the tax with the size of an economy. If it rises it does not
necessarily mean it came more from the lower income population because it depends on the population
structure of the country and their income levels, fiscal policies, etc. So, option (a) is not correct.
A rise in GDP will not necessarily be followed by the rise in tax to GDP ratio because it depends upon
the tax base whether it widens or broad and the tax-paying habits of the people of that particular
country. So, option (b) is not correct.
The reduction of a minimal amount of tax rate may increase the ratio but if the tax cuts are more it will
result in the low tax to GDP ratio. For example, the present tax rate is 15% if the government decides to
reduce that to less than 5% means the tax revenue for them will decrease compared to a minimum tax
cut of 15% to 13%. So, option (c) is not correct.
The best measure to increase the tax to GDP ratio is to widen the tax base as suggested by the
economic survey as it includes more population in the tax system and it will automatically increase the
ratio as the tax rate is the same always. So, option (d) is correct.

ADDITIONAL INFORMATION:

MEASURES TO INCREASE TAX TO GDP • Check on exceptions like transfer pricing, base
RATIO and profit erosion, etc
• Implementing the measures suggested by the Tax
Administration Reforms Commission to merge
CBDT and CBIC.
• Using PAN and following other simple laws to
increase tax buoyancy.
• Changing the attitude of the citizens towards the
nation’s duty.
• Best use of information and technology and
effective tax dispute mechanism.
• Setting up a special force for the activities of the
parallel economy.
CURRENT VALUE STATUS • Gross tax-to-GDP which was 11% in FY19, fell
to 9.9% in FY20 and marginally improved to
10.2% in FY21 (partly due to decline in GDP) and
expected to be 10.8% in FY22.

Q.45) Consider the following sources of money:

1. Profits and dividends from public sector undertakings.


2. Interest received for the loans granted.
3. Penalties, fines received by the government.
4. External Grants received by the government.

The above-mentioned sources fall under which of the following head?

(a) Tax Revenue Receipts


(b) Non-Tax Revenue Receipts
(c) Debt Capital Receipts
(d) Non-Debt Capital Receipts

OPTION ELIMINATION STRATEGY

Penalties, fines are neither tax revenue nor forms any capital, thus option (b) can be deduced

EXPLANATION:

Non-Tax Revenue is the recurring income earned by the government from sources other than taxes.
The most important receipts under this head are interest receipts (received on loans given by the
government to states, railways and others) and dividends and profits received from public sector
companies. Various services provided by the government -- police and defence, social and
community services such as medical services, and economic services such as power and railways --
also yield revenue for the government. Also, the money that is accrued by the government in the
form of penalties, fees, fines, etc., grants which are received by the government of India from its
external sources is covered.
So, Option (b) is correct.

ADDITIONAL INFORMATION:
Budget Terminologies
The Union Budget  The Union Budget is categorised into Capital Budget and Revenue
Budget.
 The Revenue Budget consists of revenue receipts of the government (tax
revenues and other revenues) and the expenditure met from these
revenues. Tax revenues comprise proceeds of taxes and other duties
levied by the Union. Other revenues are receipts of the government
mainly consisting of interest and dividends on investments made by the
government, and fees and receipts for other services rendered by the
government.
 The capital budget consists of capital receipts and payments. It also
incorporates transactions in the public account. Capital receipts are
loans raised by the government from the public which are called market
loans, borrowings by the government from the Reserve Bank and other
parties through the sale of treasury bills, loans received from foreign
bodies and governments, and recoveries of loans granted by the central
government to state and union territory governments and other parties.

Non-Debt Capital Non-debt creating capital receipts are those money receipts that are
Receipts
received by the government from the sale of old assets. These receipts are
not treated as liabilities of the government. Examples of non-debt creating
capital receipts are a recovery of loans, proceeds from the sale of public
enterprises, etc

Debt Capital Debt capital Receipts have to be repaid by the government. Market loans,
Receipts
issuance of special securities to public-sector banks, issue of securities,
short-term borrowings, treasury bills, securities against small savings, state
provident funds, relief bonds, saving bonds, gold bonds, external debt, etc,
are all examples of debt capital receipts

Q.46) With respect to the benefits of Direct tax, which of the following statements is/are correct?

1. The revenue generated from the direct tax is directly proportional to the changes in the national
wealth of the country.
2. The direct tax serves the purpose of equality and justice across all sections of the population.
3. Direct taxes can be used as an anti-inflationary tool to stabilize the price level in the market.

Select the correct answer using the code given below.


(a) 1 only

(b) 1 and 2 only

(c) 2 and 3 only

(d) 1, 2 and 3

EXPLANATION :

The revenue generated from direct tax is directly proportional to the changes in the national wealth of
the country.
In simple words, the increase in a country’s population and/or prosperity will consequently increase
the returns on direct tax.
The more wealth we acquire, the more we earn and thus have to pay more direct tax according to the
wealth earned.
And thus, the direct taxes are also very productive in nature.
So, statement 1 is correct.

The taxpayers with high income need to pay more taxes compared to the taxpayers with lesser income.
In ot/her words, the rich pay more taxes than the poor. This is, however, applicable to all the sections
of the society.
Thus, this equitable trait of the direct tax serves the purpose of equality and justice across all sections
of the population.
So, statement 2 is correct.

The increase in demand of the product and services during inflation can be decreased by increasing the
direct tax.
Doing this will force people at large to spend less money to purchase the products and services, thus,
reducing their demand and consequently the inflation rate.
Thus, direct taxes can be used as an anti-inflationary tool to stabilize the price level in the market. It
can be used to control the use and demand of products.
So, statement 3 is correct.

ADDITIONAL INFORMATION :

Direct Tax • Direct taxes are one type of taxes an individual pays that are paid straight or
directly to the government, such as income tax, poll tax, land tax, and personal
property tax.
• Such direct taxes are computed based on the ability of the taxpayer to pay,
which means that the higher their capability of paying is, the higher their taxes
are.
Types Income tax
• It is based on one’s income. A certain percentage is taken from a worker’s salary,
depending on how much he or she earns.
Transfer taxes
• The most common form of transfer taxes is the estate tax. Such a tax is levied on
the taxable portion of the property of a deceased individual, including trusts and
financial accounts.
• A gift tax is also another form wherein a certain amount is collected from people
who are transferring properties to another individual.
Property tax
• Property tax is charged on properties such as land and buildings and is used for
maintaining public services such as the police and fire departments, schools and
libraries, as well as roads.

Capital gains tax


• This tax is charged when an individual sells assets such as stocks, real estate, or
a business. The tax is computed by determining the difference between the
acquisition amount and the selling amount.
Advantages Promotes equality
• Since direct taxes are based on the ability of a person to pay, it promotes
equality among payers and citizens. Every person is charged a different amount,
depending on how much they make.
Promotes certainty
• The good thing about direct taxes is that they are determined and made final
before they are even paid. In the case of income tax, the annual tax is the same
every year as long as the salary does not change.
Promotes elasticity
• Taxes are the earnings of the government, and when they fluctuate, the earnings
also change. They can go higher or lower.
Saves time and money
• The government does not need to spend on the collection of taxes because they
are already taken right at the source of the income. Some companies
use automatic payroll deduction systems, which help save time and money.

Comparing • Direct taxes refer to taxes that are filed and paid by an individual directly to the
with government. Indirect taxes, on the other hand, are taxes that can be transferred
Indirect to another entity. Therefore, the burden of paying them can be put on another
taxes person’s shoulders.
• Direct taxes can be evaded in the absence of proper collection administration.
Indirect taxes cannot be escaped from because these are charged automatically
on goods and services.
• Direct taxes can help address inflation while indirect taxes can lead to inflation.
• Direct taxes lessen the savings of earners, but indirect taxes encourage the
opposite because they make products and services more expensive and
unaffordable.
• Direct taxes are imposed only on people that belong to various income brackets.
Indirect taxes, on the other, can be felt by everyone who buys goods and avails
services.

Q.47) Consider the following statements about the Capital Gains Tax:

1. The Capital Gains Tax in India is imposed only on immovable properties such as Land, Houses and
Farms.
2. Capital gains do not apply to an inherited property as there is no sale, only a transfer of ownership.
3. Agricultural land in rural India will not come under the category of a capital asset.

Which of the statements given above is/are correct?

(a) 1 only
(b) 1 and 2 only

(c) 2 and 3 only

(d) 1, 2 and 3

EXPLANATION:

The Capital Gains Tax in India is imposed not only on immovable properties such as Land, Houses
and Farms Land, building, house property but also on vehicles, patents, trademarks, leasehold
rights, machinery, and jewellery.
So, Statement 1 is not correct.

Capital gains do not apply to an inherited property as there is no sale, only a transfer of ownership.
The Income Tax Act has specifically exempted assets received as gifts by way of an inheritance or
will. However, if the person who inherited the asset decides to sell it, capital gains tax will be
applicable
So, Statement 2 is correct.

Agricultural land in rural areas in India is not considered a capital asset. Therefore any gains from
its sale are not taxable under the head Capital Gain
So, Statement 3 is correct

ADDITIONAL INFORMATION:
Capital Gains Tax

Meaning Any profit from the sale of a capital asset is deemed as ‘capital gains. A
capital asset is officially defined as any kind of property held by an
assessee, excluding goods held as stock-in-trade, agricultural land and
personal effects
Types  Normally if an asset is held for less than 36 months, any gain arising
from selling it is treated as a short-term capital gain (STCG) and taxed
in your hands.
 This becomes a ‘long-term’ capital gain (LTCG) if the asset is held for 36
months or more
 Shares and equity mutual funds alone enjoy a special dispensation on
capital gains tax. In their case, a holding period of 12 months or more
qualifies as ‘long-term’
Exemptions 1. Any stock, consumables or raw material, held for business or
profession

2. Personal goods such as clothes and furniture held for personal


use
3. Agricultural land in rural India
4. 6½% gold bonds (1977) or 7% gold bonds (1980) or national
defence gold bonds (1980) issued by the central government
5. Special bearer bonds (1991)
6. Gold deposit bond issued under the gold deposit scheme (1999)
or deposit certificates issued under the Gold Monetisation
Scheme, 2015
Q.48) As per the recent development in Mars Exploration, consider the following statements:

1. In the Martian Atmosphere, manmade objects can able to produce Oxygen.


2. In the Martian atmosphere, Carbon dioxide makes up 96% of the gas.
3. The oxygen could be used as liquid oxygen propellant and the rockets could use it as fuel while
coming back to Earth.

Which of the statements given above is/are correct?

(a) 1 only
(b) 1 and 2 only
(c) 2 and 3 only
(d) 1, 2 and 3

EXPLANATION:

NASA’s newest six-wheeled robot on the Martian surface converted some of the Red Planet’s thin,
carbon dioxide-rich atmosphere into oxygen. A toaster-size, experimental instrument aboard
Perseverance called the Mars Oxygen In-Situ Resource Utilization Experiment (MOXIE) accomplished
the task. MOXIE is an exploration technology investigation – as is the Mars Environmental Dynamics
Analyzer (MEDA) weather station – and is sponsored by NASA’s Space Technology Mission Directorate
(STMD) and Human Exploration and Operations Mission Directorate.
So, Statement 1 is correct.

Mars' atmosphere however is 95% carbon dioxide, 3% nitrogen, 1.6% argon, and it has traces of
oxygen, carbon monoxide, water, methane, and other gases, along with a lot of dust. Dust hanging in
the air colours Martian skies tan in photos taken from the surface. Relative to Earth, the air on Mars
is extremely thin.
So, Statement 2 is correct.

While the technology demonstration is just getting started, it could pave the way for science fiction to
become science fact – isolating and storing oxygen on Mars to help power rockets that could lift
astronauts off the planet’s surface. Such devices also might one day provide breathable air for
astronauts themselves. Rocket propellant depends on oxygen, and future explorers will depend on
producing propellant on Mars to make the trip home.
So, Statement 3 is correct

ADDITIONAL INFORMATION:
Mars missions

Perseverance  Mars 2020 is a Mars rover mission forming part of NASA's Mars
Exploration Program that includes the rover Perseverance and the small
mission
robotic, coaxial helicopter Ingenuity.

 A key objective of Perseverance’s mission on Mars is astrobiology,


including the search for signs of ancient microbial life. The rover will
characterize the planet’s geology and past climate, paving the way for
human exploration of the Red Planet, and be the first mission to collect
and cache Martian rock and regolith (broken rock and dust).

Importance
of Oxygen in To burn its fuel, a rocket must have more oxygen by weight. Getting four
mars astronauts off the Martian surface on a future mission would require
approximately 15,000 pounds (7 metric tons) of rocket fuel and 55,000
pounds (25 metric tons) of oxygen. In contrast, astronauts living and
working on Mars would require far less oxygen to breathe.

Other recent NASA: Currently, two NASA spacecraft are active on the surface of the red
Mars planet. These are InSight and Curiosity.
Missions ESA: Mars Express and Trace Gas Orbiter
ISRO: India’s Mars Orbiter Mission (MOM) or Mangalyaan was launched in
November 2013 on board a PSLV C25 rocket with the aim of studying Martian
surface and mineral composition as well as scan its atmosphere for methane
(an indicator of life on Mars).
UAESA: UAE’s Hope Mars Mission (UAE’s first-ever interplanetary mission)
CNSA: Recently, China’s spacecraft Tianwen-1 landed on Mars carrying
its first Mars rover named Zhurong. It became the third country to land on
Mars after the US and Soviet Union
Q.49) Consider the following statements:

1. The increase in demand for the product and services can be decreased by increasing the indirect
tax.
2. In India, the collection of indirect taxes will always be lower than the collection of direct taxes.

Which of the statements given above is/are correct?

(a) 1 only

(b) 2 only

(c) Both 1 and 2

(d) Neither 1 nor 2

EXPLANATION :

During Demand-pull Inflation, if the demand for the product or service has to be decreased, Taxes are
increased by the Government. This is applicable for both direct and indirect taxes.
Note : The nature of indirect tax is such that, continual increase in indirect taxes itself could trigger
inflation in the economy.
So, statement 1 is correct.

From the above graph it is true that Indirect taxes collection with respect to direct taxes were neither
always higher nor always lower in India .

So, statement 2 is not correct.

ADDITIONAL INFORMATION :

Indirect Taxes :

Definition • Indirect tax is something that a manufacturer pays to the Government of his
country.
• The burden of tax payment is on end consumer as they are the ones purchasing
the products. Unlike direct taxes, these are levied on materialistic goods.
• Excise duty, customs duty, and Value-Added Tax (VAT) are examples of Indirect
taxes.
Types Service tax:
• This tax is levied by an entity in return for the service provided by them. The
service tax is collected by the Government of India and deposited with them.
Excise duty:
• When any product or good is manufactured by a company in India, then the tax
levied on those goods is called the Excise Duty. The manufacturing company pays
the tax on the goods and in turn recover the amount from their customers.
Value Added Tax:
• Also known as VAT, this type of tax is levied on any product sold directly to
customer and are movable. VAT consists of Central Sales Tax which is paid to the
Government of India State Central Sales Tax which is paid to the respective State
Government.
Custom Duty:
• This a tax levied on the goods imported to India. Sometimes, Custome Duty is
also levied on products which are exported out of India.
Stamp Duty:
• This is a tax levied on the transfer of any immovable property in a state of India.
The state government in whose state the property is located charges this type of
tax. Stamp tax is also applicable on all legal documents too.
Entertainment Tax:
• This tax is charged by the state government and is applicable on any products or
transactions related to entertainment. Purchasing of any video games, movie
shows, sports activities, arcades, amusement parks, etc. are some of the products
on which Entertainment Tax is charged.
Securities Transaction Tax:
• This tax is levied during the trading of securities through Indian Stock Exchange.
Advantages • Convenience: Indirect taxes do not burden the taxpayer and are convenient as
they are paid only at the time of making a purchase.
• Ease of collection: Indirect taxes are easy to collect in comparison with direct
taxes.
• Collection from the poor: Those who earn less than Rs.2.5 lakh p.a. are exempt
from income tax, which means that they do not contribute to the government.
Since indirect taxes are charged at the point of sale, all individuals, regardless of
the income tax slab under which they fall, contribute towards the growth of the
economy.
• Equitable contributions: Indirect taxes are directly related to the costs of
products and services. What this essentially means that the basic necessities
attract lower rates of tax while luxury items are charged at higher tax rates,
thereby ensuring that contributions are equitable.

Dis • Indirect Tax charged sometimes are cumulative. This means that in a point-based
advantages transaction system, middlemen involved are likely to charge their own service tax
which may result in the overall price of the product increasing.
• Indirect Tax can be regressive in nature. For example, salt tax remains the same
for both poor and rich, However, if a rich person defaults the payment, then the
penalties imposed will be higher as well.
• Indirect Tax are not industry friendly. Taxes are levied on raw materials and
goods which in turn increases the cost of production, thus not allowing
industries to expand as their competitive capacity is restricted.

Q.50) Consider the following statements about the “Aries-Devasthal Faint Object Spectrograph”:

1. It is an optical spectrograph, low-cost spectroscope and India’s largest astronomical spectrograph.


2. It can locate extremely faint light sources from distant quasars and galaxies in a very young
universe, regions around supermassive black holes around the galaxies and cosmic explosions.

Which of the statements given above is/are correct?

(a) 1 only
(b) 2 only
(c) Both 1 and 2
(d) Neither 1 nor 2

EXPLANATION

Indian Scientists have indigenously designed and developed a low-cost optical spectrograph named as
Aries-Devasthal Faint Object Spectrograph & Camera (ADFOSC). The spectroscope, the largest of its
kind among the existing astronomical spectrographs in the country. So, Statement 1 is correct.
It can locate sources of faint light from distant quasars and galaxies in a very young universe, regions
around supermassive black holes around the galaxies, and cosmic explosions. It can locate sources of
light with a photon rate as low as about 1 photon per second. So, Statement 2 is correct.

ADDITIONAL INFORMATION

Aries-Devasthal Faint Object Spectrograph

Spectrograph • A spectrograph is an instrument that separates


incoming light by its wavelength or frequency and
records the resulting spectrum in some kind of
multichannel detector, like a photographic plate.
Spectrograph Working • A spectrograph allows incoming light into the telescope
through a tiny hole in a metal plate to isolate light from
a single area or object.
• This light is bounced off a special grating, which splits
the light into its different wavelengths (just like a prism
makes rainbows).
• The split light lands on a detector, which records the
spectrum that is formed.

Aries-Devasthal Faint Object • The ‘Made in India’ optical spectrograph named


Spectrograph as Aries-Devasthal Faint Object Spectrograph &
Camera (ADFOSC) is the largest of its kind among
the existing astronomical spectrographs in the
country.
Development • It is indigenously designed and developed
by Aryabhatta Research Institute of observational
sciences (ARIES), Nainital, an autonomous institute
of the Department of Science and Technology (DST).
Objective • To study distant quasars and galaxies in a very
young universe, regions around supermassive black-
holes around the galaxies, cosmic explosions like
supernovae, and highly energetic Gamma-ray bursts,
young and massive stars, and faint dwarf galaxies.
Commissioned • It has been successfully commissioned on the 3.6-m
Devasthal Optical Telescope (DOT), the largest in the
country and Asia, near Nainital Uttarakhand.
• It uses a complex arrangement of several lenses made
of special glasses, polished to better than 5-nanometer
smoothness to produce sharp images of the celestial
sky.
• Photons coming from distant celestial sources,
collected by the telescope, are sorted into different
colors by the spectrograph and are finally converted
into electronic recordable signals using an in-house
developed Charge-Coupled Device (CCD) camera cooled
to an extremely low temperature of -120 0C. s

Future Development • ARIES now plans to commission more complex


instruments such as spectro-polarimeter and high
spectral resolution spectrograph on the 3.6-m
Devasthal telescope shortly.

Q.51) The “DC Wadhwa case” in the Indian Judicial history is related to which one of the following
issues?

(a) Misuse of imposing President Rule


(b) Judicial review of Anti Defection Law
(c) Repromulgation of Ordinances
(d) The time limit for Governor to assent the bills

EXPLANATION

DC Wadhwa case challenged the power of the Governor to re-promulgate an ordinance without
changing its content for an indefinite period and not trying to convert it into an act. The exceptional
power of law-making by an ordinance cannot be used as a substitute for the legislative power of the
state legislature. So, Option (c) is correct.

ADDITIONAL INFORMATION

Ordinance

Ordinance • Ordinances have the same force and effect as an act of Parliament but are like
temporary laws.
• To deal with unforeseen and urgent matters
• It can be promulgated by the President and Governor of a State.
President • Article – 123 empowers President to promulgate an ordinance during the recess
Power of Parliament.
• The most important legislative power of the President.
• Not a parallel power of legislation – can be promulgated only ordinances have
the same force and effect as an act of Parliament but are like temporary laws.
• Cooper case – President’s satisfaction of exceptional circumstances is justiciable
on the ground of malafide.
Conditions • An ordinance can be issued only on those subjects on which the Parliament can
make laws.
• An ordinance is subject to the same constitutional limitation as an act of
Parliament. Hence, an ordinance cannot abridge or take away any of the
fundamental rights.
• Ordinance making power of President is not a discretionary power; he has to act
on the aid and advice of the Council of Ministers headed by the Prime Minister.
Parliament • Every ordinance issued by the President during the recess of Parliament must be
approval laid before both the Houses of Parliament when it reassembles.
o If approved becomes an act.
o It can also be disapproved by both houses by passing a resolution.
o If no action is taken then cease to operate on expiry of six weeks from
the reassembly of Parliament.
(Six weeks calculated from reassembling of the later house)
Special • Like other laws, the ordinance can also be retrospective.
features • It can also modify or repeal any act of Parliament or other ordinance.
• It can also amend tax law.
• It cannot be issued to amend Constitution.
Governor • He can promulgate ordinances when the state legislature is not in session.
Power • These ordinances must be approved by the state legislature within six weeks from
its reassembly.
• He can also withdraw an ordinance anytime.
• This is the most important legislative power of the governor.

Q.52) As of now, which one of the following is the single largest source (about 40%) of India’s greenhouse
gas emissions?

(a) Cement and Chemical Industries


(b) Electricity sector
(c) Transportation Sector
(d) Agricultural Sector

EXPLANATION:

According to the Third Biennial Update Report of India to the United Nations Framework Convention
on Climate Change the energy sector in India accounted for about 75 per cent of the total GHG
emissions for the year 2016. Electricity production was the single largest source in this category,
accounting for about 40 per cent of the national total GHG emissions in 2016. The manufacturing
industries and construction together emitted 18.68 per cent of total emissions from the energy sector.
Transport contributed to 13 per cent of emissions from the energy sector with predominant share of 90
per cent arising from road transport followed by civil aviation (6 per cent), railways (3 per cent) and
domestic water-borne navigation (1 per cent).

So, Option (b) is correct.

ADDITIONAL INFORMATION:

Greenhouse gas emissions

India's status
 India is the third-largest emitter of carbon dioxide (CO2) in the world (not
including the European Union [EU]), emitting 2.3 gigatonnes (gt) CO2 in
2015.
 Measured in absolute terms, India’s emissions have been the third-
highest globally since about 2008.
 While current per capita emissions are very low, projections by the
United Nations (UN) suggest that India’s population will continue to
increase and surpass China’s around 2025 resulting in an overall
increase.
Steps taken by
India  India’s new targets comprise of five elements.

1) Reducing Emissions Intensity or emissions per unit of GDP by 45%


in 2030 relative to 2005 levels
2) Cutting absolute emissions by one billion tonnes, presumably from
projected business-as-usual (BAU) 2030 levels. India’s current
annual emissions are around 2.8 billion tonnes and projected to
reach about 4.5 billion tonnes in 2030 on a BAU basis.
3) 500 GW of non-fossil fuel installed power generation capacity by
2030
4) 50% electricity generation from renewable sources by 2030
5) Net-zero emissions by 2070

 International Solar Alliance (ISA)-It is an alliance of about 121 nations


heralded by France and India, lying partly or completely between the
tropics to harness their solar energy potential by collaborative efforts in
the field of solar technologies
 National Adaptation Fund for Climate Change (NAFCC)-To assist State
and UTs particularly vulnerable to the adverse effects of climate change
in meeting the cost of adaptation.
 Government of India approved the National Mission on Electric Mobility
in 2011 and subsequently National Electric Mobility Mission Plan
(NEMMP) 2020 was unveiled in 2013.
 The idea of National Hydrogen Mission was first mooted in the Budget
2021 putting an effort to tap green energy sources for manufacturing
hydrogen.
 The government has already set a vision to make India a gas-based
economy. The goal is to increase the share of gas in the energy mix from
the current 6 per cent to 15 per cent in 2030.
 The Railways’ net-zero 2030 targets cutting 60 million tonnes annually
 Use of LED bulbs targets cutting another 40 million tonnes a year.
 India has reached around 101 GW of solar and wind due to numerous
constraints. If large hydro and nuclear are added,current RE installed
capacity is about 150 GW.
 Infrastructure for Resilient Island States (IRIS) - India has launched an
international climate initiative called IRIS to provide technical knowledge
and financial assistance to small island nations with the help of
developed countries
Global Greenhouse
Gas emissions

Q.53) Consider the following statements about the “Emergency Use Authorisation” (EUA) of vaccines,
recently seen in the news:

1. A EUA does not mean that a vaccine has skipped essential safety trials.
2. Only the World Health Organization’s approved vaccines can be used under the EUA.
3. A new vaccine that has already been approved for emergency use in other countries cannot be
granted the EUA.

Which of the statements given above is/are correct?

(a) 1 only
(b) 1 and 2 only
(c) 2 and 3 only
(d) 1 and 3 only

EXPLANATION:

An EUA does not mean that a vaccine has skipped essential safety trials. The regulators need to satisfy
themselves that the product meets reasonable thresholds for safety and effectiveness before granting
approval.

In the US, for instance, the Food and Drug Administration grants EUA for Covid vaccines only after
-a vaccine-maker has undertaken Phase 1 and Phase 2 trials

i. it is able to provide safety and efficacy data for Phase 3 trials as well, using data
generated from over 3,000 participants
So, Statement 1 is correct.
When there is a declared emergency, the regulator of the country decide whether it is worth releasing a
drug or vaccine that is not fully tested for efficacy and safety. In India’s case, it is the DCGI (Drugs
Controller General of India).If there is evidence to suggest it may benefit patients, then the regulator is
well within its rights to issue an EUA to a medical product. It will then be made widely available for
use. Drug regulators in many countries follow a basic thumb rule. This is to approve a drug or a
vaccine if the known and potential benefits outweigh the known potential risks. Thus it is not only the
WTO approved drugs/vaccines.

So, Statement 2 is not correct.


EUA can be granted to new vaccines that have already been approved for emergency use in other
countries.

In January 2021, the DCGI of India approved the first two vaccines which was already approved in
other countries

1. Covishield, produced by Pune-based Serum Institute of India under licensing agreement from
AstraZeneca
2. Covaxin, manufactured by Bharat Biotech

So, Statement 3 is not correct.

ADDITIONAL INFORMATION:

Authorisation of vaccines

How vaccines are


approved  Vaccines and medicines, and even diagnostic tests and medical devices,
require the approval of a regulatory authority before they can be
administered. In India, the regulatory authority is the Central Drugs
Standard Control Organisation (CDSCO).
 For vaccines and medicines, approval is granted after an assessment of
their safety and effectiveness, based on data from trials. Approval from
the regulator is required at every stage of these trials. This is a long
process, designed to ensure that medicine or vaccine is safe and
effective.
 The fastest approval for any vaccine until now — the mumps vaccine in
the 1960s — took about four-and-a-half years after it was developed

• In Phase 1 trials, a vaccine is given to a limited sample set of healthy


people to assess its safety at higher doses.
• If Phase 1 does not throw up safety concerns, Phase 2 is undertaken on
hundreds of people with different health conditions and from different
population strata.
• This helps assess both the effectiveness and the side-effects.
• Phase 3 involves much larger sample, representative of the actual
population, to assess both safety and efficacy.

EUA process
 In emergencies, like the current one, regulatory authorities around the
world have developed mechanisms to grant interim approvals if there is
sufficient evidence to suggest a medical product is safe and effective.
 Final approval is granted only after completion of the trials and analysis
of full data; until then, emergency use authorisation (EUA) allows the
medicine or the vaccine to be used on the public.
 WHO’s EUL procedure is a risk-based procedure for evaluating and
listing unlicensed vaccines, therapies, and in-vitro diagnostics to
expedite the availability of these products to people affected by a public
health emergency. It also allows countries to expedite their regulatory
approval to import and administer vaccines.

Q.54) With respect to Hydrogen as the Energy in India, which one of the following is not correct?

(a) Hydrogen provides three times more energy than fossil fuels
(b) The production of hydrogen releases pure water as the only by-product
(c) Hydrogen occurs naturally on the Earth in the form of gas only
(d) Hydrogen is the most abundant element in the universe

EXPLANATION

Hydrogen has the highest energy content of any common fuel by weight so it can provide three times
more energy than running on with other fossil fuels. So, Option (a) is not correct.
Hydrogen production takes place from natural gas, oil, coal and electrolysis of water producing by-
products like oxygen, carbon monoxide and carbon dioxide-based on the method and source of
production. When hydrogen is consumed as a fuel it is considered clean fuel because it produces only
water as a by-product. So, Option (b) is correct.
Hydrogen occurs naturally on earth but only in compound form with other elements in liquids, gases,
or solids. Not just as gas. So, Option (c) is not correct.
Hydrogen is found in great quantities on Earth combined with other elements. In the Universe also it is
the most abundant as stars such as the sun consist mostly of hydrogen since it is highly flammable
and sustains continuous burning. So, Option (d) is not correct.

ADDITIONAL INFORMATION

Hydrogen • It is the simplest element.


• It has only one proton.
• It is both abundant in our environment and also in the Universe.
• It occurs naturally on earth but only in compound form with other
elements in liquids, gases, or solids.
• It is considered an energy carrier that can be used to store, move, and
deliver energy produced from other sources.
Types Depending on the types of the production process involved hydrogen can be
classified into
• Green hydrogen – Produced by using renewable sources which electrolyse
water into hydrogen and oxygen, emitting zero carbon dioxide in the
process.
• Blue hydrogen – Produced from natural gas using steam reforming. Output
is hydrogen and carbon dioxide which is captured a stored (Carbon Capture
and Storage).
• Grey hydrogen – Produced from natural gas or methane without capturing
greenhouse gases.
• Black and brown hydrogen – Using black coal or lignite (brown coal) in
hydrogen making. It causes the most damage to the environment.
• Pink hydrogen – Generated through electrolysis powered by nuclear energy.
• Yellow hydrogen – New technology which makes use of solar power for
electrolysis.
Production • Hydrogen can be produced through low carbon technology.
• Some of the technologies used are as follows.
o Fossil Resources – By product solid carbon, carbon monoxide,
carbon dioxide
o Biomass/Waste – By product clean water, electricity and chemicals
o Electrolysis of Water – By-product water
As Fuel • Alternative fuel
• Power fuel cells in electric vehicles
• Clean fuel – when consumed produces only water
Advantage • Readily available in nature
• Doesn’t emit greenhouse gases
• Used in the rocket as a fuel
• Efficient as a fuel
• Renewable source of energy
Disadvantage • Expensive to produce and difficult to store
• Highly flammable
National • Generation of hydrogen from green power sources
Hydrogen • Try to replace fossil fuels and help to achieve climate change goals
Mission • Pilot project on Blue Hydrogen, Hydrogen CNG (H-CNG) and Green
Hydrogen is carried out

Q.55) The “Dandeli Wildlife Sanctuary”, recently seen in the news, is located at which one of the following
locations in India?

(a) It is located on the confluence of the Western Ghats and the Eastern Ghats
(b) It is the largest sanctuary in the North-Western Desertic Plains in India
(c) It is the only known tiger reserve in India to report frequent sightings of the elusive black
panther
(d) It is located in the high ranges of the Central Indian Highlands.

EXPLANATION

Dandeli Wildlife Sanctuary is located in Uttara Kannada district of Karnataka whereas the Western
Ghats and Eastern Ghats confluence at Nilgiris hill in Tamil Nadu. So, Options (a) and (d) are not
correct.
It consists of dense deciduous trees and not a desertic plain and is considered the second largest
wildlife sanctuary in Karnataka. So, Option (b) is not correct.
Black Panther is one of the rarest to sight in India and it can be spotted in six wildlife sanctuaries in
India and they are Dandeli Wildlife Sanctuary, Kabini Wildlife Sanctuary, Bhadra Wildlife Sanctuary,
Tadoba Andhari Tiger Reserve, Manas National Park and Nagarhole National Park. So, Option (c) is
correct.

ADDITIONAL INFORMATION

Dandeli Wildlife Sanctuary


Dandeli Wildlife Sanctuary • In the Uttar Karnataka region of Southern India.
• It is a joint tiger project with the adjoining Anshi National
Park.
• It is also considered as Elephant Reserve under Project
Elephant.
• Located on banks of River Kali.
Flora • Dense deciduous trees and evergreen trees
• Covered with Bamboo and teak
• Also have herbs, climbers and creepers
Fauna • Barking deer
• Bison
• Indian Mongolian
• Black Panther
• Sambhar
• Tiger
• Leopard
• Elephant
• Flying squirrel
• Malabar giant squirrel
Special Features • A unique experience of bird watching
• Includes Blue-throated Barbet, Great Pied Hornbill, Malabar
Pied Hornbill and Peregrine falcon
• Crocodile spotting is the major attraction
• Variety of reptilian and amphibian species
Black Panther • A melanistic colour variant of leopard and jaguar
• Found in tropical and deciduous forests

Q.56) Consider a situation that a trader ‘X’ from Tamil Nadu had sold goods to ‘Y’ from Karnataka worth
Rs. 1,00,000. The GST rate is 18% for this transaction. Now in this regard, which one of the following
statements is correct?

(a) The tax amount will be shared between Tamil Nadu and Karnataka

(b) The Tax amount will be allocated only to the Central Government

(c) The Tax amount will be shared between Tamil Nadu and the Central Government

(d) The Tax amount will be shared between Karnataka and the Central Government

EXPLANATION :
Since, GST is a destination tax and therefore it is not Tamil nadu but Karnataka will receive its
share of SGST.
In the Given Example, For goods worth Rs. 1,00,000 will attract an IGST of Rs.18,000 (18%) since
transaction is done Interstate.
This IGST will be collected by centre and the state’s Share of Rs.9000 will be transferred to the
state government (in this case destination state of karnataka).
Note : IGST is paid by the receiving person, collected by the sender, given to the central government
and distributed between central and state governments.
So, option (d) is correct.
ADDITIONAL INFORMATION :

Model of Working of IGST :

Taxes under
GST

Intra State • When the location of the supplier and the place of supply i.e., location of the
buyer are in the same state. In Intra-State transactions, a seller has to
collect both CGST and SGST from the buyer.
• The CGST gets deposited with the Central Government and SGST gets
deposited with the State Government.
Inter State • When the location of the supplier and the place of supply are in different
states.
• Also, in cases of export or import of goods or services or when the supply of
goods or services is made to or by a SEZ unit, the transaction is assumed to
be Inter-State.
• In an Inter-State transaction, a seller has to collect IGST from the buyer.
Q.57) Consider the following statements:

1. Every cess is collected only after Parliament has authorised its creation

2. Constitution of India allows cess to be excluded from the purview of the divisible pool of taxes.

Which of the statements given above is/are correct?

(a) 1 only
(b) 2 only
(c) Both 1 and 2
(d) Neither 1 nor 2

EXPLANATION :

Every cess is collected after Parliament has authorised its creation through an enabling legislation that
specifies the purpose for which the funds are being raised.
So, statement 1 is correct.

Article 270 of the Constitution allows cess to be excluded from the purview of the divisible pool of taxes
that the Union government must share with the States.
So, statement 2 is correct.

ADDITIONAL INFORMATION :

Cess:

Definition • The Union government is empowered to raise revenue through a gamut of


levies, including taxes (both direct and indirect), surcharges, fees and cess.
• While direct taxes, including income tax, and indirect taxes such as GST are
taxes where the revenue received can be spent by the government for any
public purpose in any manner it deems appropriate for the nation’s good, a
cess is a earmarked tax that is collected for a specific purpose and ought to be
spent only for that.
Timeline of • A report titled Cesses and Surcharges: Concept, Practice and Reforms since
Cess in India 1944, prepared by the Vidhi Centre for Legal Policy in August 2018 and
submitted to the Fifteenth Finance Commission listed 42 cesses that have been
levied at various points in time since 1944.
• The very first cess was levied on matches, according to this study.
• Post-Independence, the cess taxes were linked initially to the development of a
particular industry, including a salt cess and a tea cess in 1953.
• Subsequently, the introduction of a cess was motivated by the aim of ensuring
labour welfare. Some cesses that exemplified this thrust were the iron ore
mines labour welfare cess in 1961, the limestone and dolomite mines labour
welfare cess of 1972 and the cine workers welfare cess introduced in 1981.
• The introduction of the GST in 2017 led to most cesses being done away with
and as of August 2018, there were only seven cesses that continued to be
levied.
• These were Cess on Exports, Cess on Crude Oil, Health and Education Cess,
Road and Infrastructure Cess, Building and Other Construction Workers
Welfare Cess, National Calamity Contingent Duty on Tobacco and Tobacco
Products and the GST Compensation Cess.
• And in February 2021, Central Government introduced a new cess — a Health
Cess of 5% on imported medical devices — in the Finance Bill for 2020-2021.
Criticisms • The CAG’s finding that the Centre retained ₹47,272 crore of GST compensation
cess in the Consolidated Fund instead of crediting it to the GST compensation
fund in the very first two years of the implementation of the new indirect tax
regime has raised several key questions.
• Given that cess does not need to be a part of the divisible pool of resources,
this increasing share of cess in the Union government’s tax receipts has a
direct impact on fiscal devolution.

Q.58) Consider the following statements regarding the Producer Price Index (PPI) inflation:

1. It measures the average change over time in the selling prices received by domestic producers for
their output.
2. It tracks price change in both the goods and the services sector.
3. It measures prices from the perspective of consumers as well as from the viewpoint of industries
that make the products.

Which of the statements given above is/are correct?

(a) 1 only

(b) 1 and 2 only

(c) 2 and 3 only

(d) 1, 2 and 3

EXPLANATION:

The Producer Price Index is a family of indexes that measures the average change over time in the
selling prices received by domestic producers of goods and services. The Producer Price Index hasn't
been in use in India yet, but Niti Aayog has created a roadmap to introduce it soon.
So, Statements 1 and 2 are correct.

The PPI measures price movements from the seller's point of view. Conversely, the consumer price
index (CPI) measures cost changes from the viewpoint of the consumer. In other words, this index
tracks changes to the cost of production. Sellers’ and purchasers’ prices differ due to government
subsidies, sales and excise taxes, and distribution costs
So, Statement 3 is not correct.

ADDITIONAL INFORMATION:
Inflation Index

Inflation  Inflation is the measure of change in the average price of services


and commodities, done at regular intervals. It indicates a decrease in
the purchasing power of a unit of a nation’s currency as the
products and services get more expensive. Inflation is the difference
between aggregate demand and aggregate supply of goods and
services. When aggregate demand exceeds the supply of goods at
current prices, there is a rise in the price level.

 A certain level of inflation is required in the economy to ensure that


expenditure is promoted and money hoarding through savings is
discouraged.

Consumer Price  In India, there are two main sets of inflation indices to measure
Index changes in price levels — Consumer Price Index (CPI) and Wholesale
Price Index (WPI). These indices measure changes at the retail and
wholesale price levels, respectively. CPI tracks any shift in retail
prices of essential and daily goods and services consumed by
households across the country. In short, it captures changes in price
level at the consumer level.
 In April 2014, the RBI had adopted the CPI as its key measure of
inflation.

Wholesale Price  WPI, on the other hand, is the average change in the price of
Index commodities at the wholesale level. It considers the price of goods
traded among corporations, not goods purchased by consumers. The
aim of WPI aims price drifts that reflect demand and supply in
manufacturing, industry and construction
 WPI data is published by the Office of Economic Adviser, Ministry of
Commerce and Industry, while CPI data is published by the National
Statistical Office (NSO), Ministry of Statistics and Programme
Implementation (MoSPI). The base year for WPI is 2011-12 while the
base year for CPI is 2012.

Q.59) Consider the following statements about the “Negative Real Interest Rate”, often seen in the news:

1. The real rate of return on investment is its actual rate of return minus the prevailing inflation rate.
2. Real interest rates can be effectively negative if inflation exceeds the nominal interest rate.
3. The negative real interest rates will only affect the interest income on the investments and not the
capital.

Which of the statements given above are correct?

(a) 1 only

(b) 1 and 2 only

(c) 2 and 3 only

(d) 1, 2 and 3

EXPLANATION :
The real rate of return on an investment is its actual rate of return minus the prevailing inflation
rate.
To get at the real interest rate, we subtract the rate of inflation during the period of investment,
from the return we earn on it.
So, statement 1 is correct.
Eg: If SBI’s one-year FD that is offering an interest of 5 per cent. If inflation in the next one year
turns out to be 5.3 per cent, the real rate of interest would be a negative 0.3 per cent (five minus
the expected inflation rate of 5.3 per cent).
Real interest rates can be effectively negative if inflation (5.3%) exceeds the nominal interest rate
(5%).
So, statement 2 is correct.
Negative real interest rates will not only decrease the interest on the Capital but also reduces the
value of the capital.
For eg, If we save Rs. 10,000 as deposit to buy an LED for Rs. 10,400 after 2 years
Interest rate = 4%; Inflation= 6% ; Effective IR= 2%
After 2years, Returns = Rs. 10,200. But in the meantime due to inflation the cost of the LED has
also risen to around Rs. 10,600 (which can’t be bought with return of Rs. 10,200)
Therefore the value of the amount which we have deposited has also reduced because of the NRIR.
So, statement 3 is not correct.

ADDITIONAL INFORMATION :

Nominal • The nominal interest rate is the stated interest rate of a bond or loan, which
signifies the actual monetary price borrowers pay lenders to use their money.
Interest Rate

Real Interest • The real interest rate is so named, because unlike the nominal rate, it factors
inflation into the equation, to give investors a more accurate measure of their
Rate buying power, after they redeem their positions.
Negative • The term negative interest rate refers to interest paid to borrowers rather than
to lenders.
Interest Rate
• Central banks typically charge commercial banks on their reserves as a form
of non-traditional expansionary monetary policy, rather than crediting them.
• This is a very unusual scenario that generally occurs during a deep economic
recession when monetary efforts and market forces have already pushed
interest rates to their nominal zero bound.
• This tool is meant to encourage lending, spending, and investment rather than
hoarding cash, which will lose value to negative deposit rates.
Significance • Negative interest rates are a form of monetary policy that sees interest rates
fall below 0%.
of Negative IR
• Central banks and regulators use this unusual policy tool when there are
strong signs of deflation.
• Borrowers are credited interest instead of paying interest to lenders in a
negative interest rate environment.
• Central banks charge commercial banks on reserves in an effort to incentivize
them to spend rather than hoard cash positions.
• Although commercial banks are charged interest to keep cash with a nation's
central bank, they are generally reluctant to pass negative rates onto their
customers.

Q.60) Consider the following statements regarding the procedure in the Appointment of Ad-hoc Judges in
the High Courts of India:

1. The Supreme Court can appoint the retired judges of both the Supreme Court and the High Court
as ad-hoc judges to a high court.
2. It is mainly used to deal with the backlog of cases in the High Court.
3. Since the goal was to clear the backlog, the ad-hoc judges can be assigned more than five-year-old
cases.

Which of the statements given above is/are correct?

(a) 1 only
(b) 1 and 2 only
(c) 2 and 3 only
(d) 1, 2 and 3
EXPLANATION:

The Supreme Court has laid down guidelines for the appointment of ad-hoc judges in High Courts
under Article 224A of the Constitution of India. Article 224A enables a Chief Justice of a High
Court, with the previous consent of the President, to request a former High Court judge to "sit and
act as a judge" of the High Court to hear cases.
So, Statement 1 is not correct.

Terming pendency of around 57 lakh cases in High Courts as “docket explosion”, the Supreme
Court activated a “dormant” constitutional provision to pave way for appointment of retired High
Court judges as ad-hoc ones for a period of two to three years to clear the backlog and came out
with guidelines to regulate appointments. Thus it is mainly used to deal with the backlog of cases
in the High Court.
So, Statement 2 is correct.

Their tenure, the Supreme court said, may vary based on the need. Since the goal was to clear the
backlog, the ad-hoc judges can be assigned more than five-year-old cases, it said, adding that this
would not, however, affect the High Court Chief Justice's discretion to allot any other matter.
So, Statement 3 is correct.

ADDITIONAL INFORMATION:
Ad hoc judges in the high court
 Article 224A, used rarely, of the Constitution deals with the
appointment of ad-hoc judges in High Courts and says “the Chief
Justice of a High Court for any State may at any time, with the previous
consent of the President, request any person who has held the office of a
Judge of that Court or of any other High Court to sit and act as a Judge
of the High Court for that State”.
 A bench of Chief Justice S.A. Bobde and Justices S.K. Kaul and Surya
Kant issued a slew of guidelines about issues such as tenure, salary,
perks, etc, and their role in adjudicating cases recent decision of the
Supreme Court of India laid down guidelines for the chief justices of
high courts to invoke Article 224A of the constitution to appoint retired
high court judges on an ad-hoc basis.

Conditions There are two steps to be fulfilled for high courts to make use of this
provision.
1) Step one, the high court concerned should have made recommendations
for more than 20% of its vacancies, without which Article 224A cannot
be invoked. The judgment states that the reason for this requirement is
to ensure that Article 224A is not resorted to as an easy alternative to
judicial appointments made through the regular process.
2) Step two, the judgment states that there can be multiple trigger points
for appointing ad-hoc retired judges and lists the below five events as
triggers (it is important to note here however that the judgment does not
state that this is an exhaustive list):

Q.61) Which one of the following statements is not correct about the Council of Scientific & Industrial
Research?

(a) It is a pioneer of India’s intellectual property movement


(b) It operates as an autonomous body through the Societies Registration Act, 1860
(c) It is the largest Research and Development organisation in India
(d) It is headed by the Union Minister of Science and Technology

EXPLANATION:

CSIR is a Pioneer of India’s intellectual property movement. It has filed about 225 Indian patents
and 250 foreign patents per year during 2015-20. CSIR has a patent portfolio of 1,132 unique
patents in force, out of which 140 patents have been commercialized. CSIR also has 2,587 in force
patents granted abroad in multiple countries. Amongst its peers in publicly funded research
organizations globally, CSIR is a leader in filing and securing patents worldwide.
So, option (a) is correct.

Although it is mainly funded by the Ministry of Science and Technology, it operates as an


autonomous body through the Societies Registration Act, 1860
So, Option (b) is correct.

The Council of Scientific and Industrial Research (IAST: vaigyanik tathā audyogik anusandhāna
pariṣada) abbreviated as CSIR was established by the Government of India in September 1942 as
an autonomous body that has emerged as the largest research and development organisation in
India
So, option (c) is correct

The Prime Minister of India is the chairman of the Council of Scientific and Industrial Research.
So, option (d) is not correct

ADDITIONAL INFORMATION:
Council of Scientific & Industrial research

Features
 The Council of Scientific & Industrial Research (CSIR), known for its
cutting edge R&D knowledge base in diverse S&T areas, is a
contemporary R&D organization.
 CSIR has a dynamic network of 37 national laboratories, 39 outreach
centres, 3 Innovation Complexes, and five units with a pan-India
presence. CSIR’s R&D expertise and experience are embodied in about
3460 active scientists supported by about 4350 scientific and technical
personnel as of June 2021.
 CSIR is ranked 37th among 1587 government institutions worldwide
and is the only Indian organization among the top 100 global
government institutions, according to the Scimago Institutions Ranking
World Report 2021. CSIR holds the 7th rank in Asia and leads the
country at the first position.
Area of work CSIR covers a wide spectrum of science and technology – from
oceanography, geophysics, chemicals, drugs, genomics, biotechnology and
nanotechnology to mining, aeronautics, instrumentation, environmental
engineering and information technology. It provides significant technological
intervention in many areas concerning societal efforts, which include
environment, health, drinking water, food, housing, energy, farm and non-
farm sectors. Further, CSIR’s role in S&T human resource development is
noteworthy.

2. Vice President: Union Minister of Science and Technology (Ex-officio)


Organisation
3. Governing Body: The Director-General is the head of the governing
body.
a. The other ex-officio member is the finance secretary
(expenditures).
b. Other members' terms are of three years

Q.62) Which one of the following represents a ‘progressive tax structure’?

(a) Tax rate is the same across all Incomes

(b) Tax rate increases as income increases

(c) Tax rate decreases as income increases

(d) Higher revenue of taxation on consumption than income

EXPLANATION:

India follows a progressive tax regime. A progressive tax system means that high-income earners
are taxed more than low-income ones. The extent of the progressivity in this tax method depends on
how fast the tax rates rise with the increase in income levels. An example of progressive taxation
is a 10% tax rate for income of Rs 2 lakh, 20% for Rs 5 lakh and 30% for Rs 10 lakh.
So, Option (b) is correct.

ADDITIONAL INFORMATION:
Tax Structures

Proportional  A proportional tax is a taxing mechanism in which the taxing authority


Taxation charges the same rate of tax from each taxpayer, irrespective of income.
This means that lower class, or middle class, or upper-class people pay
the same amount of tax. Since the tax is charged at a flat rate for
everyone, whether earning higher income or lower income, it is also
called a flat tax.
 A proportional tax is based on the theory that since everybody is equal,
taxes should also be charged the same way.
It is unfair to charge more from anybody having a higher income. The
government charges a flat rate of 30% on the income earned by the
companies in India, exclusive of surcharge and educational cess. A
surcharge of 10% and a cess are collected on the tax amount collected.

Regressive
Taxation  Low-income individuals pay a higher amount of taxes compared to
high-income earners under a regressive tax system. That's because the
government assesses tax as a percentage of the value of the asset that a
taxpayer purchases or owns. This type of tax does not correlate with an
individual's earnings
 Such taxes include property taxes, sales taxes on goods, and excise
taxes on consumables, such as gasoline or airfare. Excise taxes are
fixed and they're included in the price of the product or service

Q.63) Consider the following factors:


1. The target growth rate of production
2. Increased banking habits of the people
3. Inflation
4. The excess inflow of foreign currency

Which of the above factors determine the quantity of currency printed in an economy?

(a) 1 and 3 only


(b) 1, 3 and 4 only
(c) 1, 2 and 3 only
(d) 1, 2, 3 and 4

ADDITIONAL INFORMATION:

CURRENCY REGULATIONS IN INDIA • The Reserve Bank of India (RBI) prints and
manages currency in India, whereas the Indian
government regulates what denominations to
circulate.
• The Indian government is solely responsible for
minting coins.
• The RBI is permitted to print currency up to
10,000-rupee notes.
• RBI was granted its role in currency management
based on the Reserve Bank of India Act in 1934.
Specifically, Section 22 of the RBI Act gives the
bank the authority to issue currency notes.
• The Reserve Bank of India has printing
facilities in Dewas, Mysore, and Salboni

Q.64) The term “New umbrella entities”, sometimes seen in the news recently, is related to

(a) Non-performing assets

(b) Retail payment systems

(c) Foreign Direct Investment

(d) Credit rating agencies

EXPLANATION:

To set up pan-India umbrella entity/entities focussing on retail payment systems to manage


and operate new payment system(s) in the retail space comprising of but not limited to ATMs,
White Label PoS; Aadhaar based payments and remittance services; newer payment methods,
standards, and technologies; monitor related issues in the country and internationally; take
care of developmental objectives like enhancement of awareness about the payment systems.
Such entity shall be a Company incorporated in India under the Companies Act, 2013 and
maybe a ‘for-profit’ or a Section 8 Company as may be decided by it. The umbrella entity shall
be a Company authorized by the Reserve Bank of India (RBI) under Section 4 of the PSS Act,
2007.
So, option (b) is correct.
ADDITIONAL INFORMATION:

NEW UMBRELLA ENTITIES

SCOPE OF ACTIVITIES  Operate clearing and settlement systems for


participating banks and non-banks; identify and
manage relevant risks such as settlement, credit,
liquidity, and operational and preserve the integrity of
the system(s); monitor retail payment system
developments and related issues in the country and
internationally to avoid shocks, frauds, and contagions
that may adversely affect the system(s) and/or the
economy in general.
 Fulfill its policy objectives and ensure that principles of
fairness, equity, and competitive neutrality are applied
in determining participation in the system; frame
necessary rules and the related processes to ensure
that the system is safe and sound and that payments
are exchanged efficiently.
 Carry on any other business as suitable to further
strengthen the retail payments ecosystem in the
country. It is expected that the umbrella entity shall
offer innovative payment systems to include hitherto
excluded cross-sections of the society and which
enhance access, customer convenience, and safety and
the same shall be distinct yet interoperable.
 It is also expected to interact and be interoperable, to
the extent possible, with the systems operated by
NPCI.
 The umbrella entity may be permitted to participate in
Reserve Bank’s payment and settlement systems,
including having a current account with Reserve Bank,
if required.

PLAYERS INVOLVED AND  The entities are owned and controlled by Indian
CRITERIA citizens with a minimum of 3 years of experience in the
payment system.
 Foreign investment is allowed as long as they follow the
guidelines.
 The umbrella entity shall have a minimum paid-up
capital of ₹500 crores. No single Promoter / Promoter
Group shall have more than 40% investment in the
capital of the umbrella entity.

NEED FOR NEW UMBRELLA  The volume of digital transactions occurring in India
ENTITIES and the need for their effective monitoring.
 As of now, the NPCI (National payments corporation of
India) handles the whole payment system and acts as a
monopoly.
 To increase the competition in this sector and also for
their effective functioning in dealing with the customer
queries, granting licenses, etc.

Q.65) The Union Ministry of Health and Family Welfare has recently notified the National Policy on Rare
Diseases, 2021. In this context, consider the following statements.

1. A ‘rare disease’ is defined as a health condition of low prevalence that affects a small number of
people when compared with other prevalent diseases in the general population.
2. About 95% of rare diseases have no approved treatment and less than 1 in 10 patients receive
disease-specific treatment.
3. According to the policy, rare diseases include genetic diseases, rare cancers, infectious tropical
diseases and degenerative diseases.

Which of the statements given above is/are correct?

(a) 1 only
(b) 1 and 2 only
(c) 2 and 3 only
(d) 1, 2 and 3

EXPLANATION:

Broadly, a ‘rare disease’ is defined as a health condition of low prevalence that affects a small
number of people when compared with other prevalent diseases in the general population. While
there is no universally accepted definition of rare diseases, countries typically arrive at their
descriptions, taking into consideration disease prevalence, its severity and the existence of
alternative therapeutic options.
So, Statement 1 is correct.

About 95% of rare diseases have no approved treatment and less than 1 in 10 patients receive
disease-specific treatment. These diseases have differing definitions in various countries and range
from those that are prevalent in 1 in 10,000 of the population to 6 per 10,000
In India, roughly 450 rare diseases have been recorded from tertiary hospitals, of which the most
common are Haemophilia, Thalassemia, Sickle-cell anaemia, auto-immune diseases, Gaucher’s
disease, and cystic fibrosis
So, Statement 2 is correct.

India does not have a definition of rare diseases because there is a lack of epidemiological data on
their incidence and prevalence. According to the policy, rare diseases include genetic diseases, rare
cancers, infectious tropical diseases, and degenerative diseases. As per the policy, out of all rare
diseases in the world, less than five per cent have therapies available to treat them.
So, Statement 3 is correct

ADDITIONAL INFORMATION:
National Policy on rare diseases, 2021

Objective
 The Rare Diseases Policy aims to lower the high cost of treatment for rare
diseases with an increased focus on indigenous research with the help of
a National Consortium to be set up with the Department of Health
Research, Ministry of Health & Family Welfare as convenor.
 Increased focus on research and development and local production of
medicines will lower the cost of treatment for rare diseases.
 The policy also envisages the creation of a national hospital based
registry of rare diseases so that adequate data is available for the
definition of rare diseases and research and development related to rare
diseases within the country.

Categories  Under the policy, there are three categories of rare diseases —
requiring one-time curative treatment, diseases that require long-
term treatment but where the cost is low, and those needing long-
term treatments with high cost
Screening  The Policy also focuses on early screening and prevention through
primary and secondary health care infrastructure such as Health and
Wellness Centres and District Early Intervention Centres (DEICs) and
counselling for the high-risk parents. The screening will also be supported
by Nidan Kendras set up by the Department of Biotechnology.

Tertiary healthcare  The policy also aims to strengthen tertiary health care facilities for the
prevention and treatment of rare diseases through designating 8 health
facilities as Centre of Excellence and these CoEs will also be provided one-
time financial support of up to Rs 5 crores for up-gradation of diagnostics
facilities.
Financial Support  A provision for financial support up to Rs. 20 lakhs under the Umbrella
Scheme of Rashtriya Arogya Nidhi is proposed for treatment, of those rare
diseases that require a one-time treatment (diseases listed under Group 1
in the rare disease policy). Beneficiaries for such financial assistance
would not be limited to BPL families, but the benefit will be extended to
about 40% of the population, who are eligible under Pradhan Mantri Jan
Arogya Yojana.

Q.66) In which of the following organizations, both India and Japan have common membership?

1. Quadrilateral Security Dialogue or QUAD


2. East Asia Summit
3. Asia-Africa Growth Corridor
4. G4 Nations

Select the correct answer using the code given below.

(a) 1, 2 and 4 only


(b) 1, 2 and 3 only
(c) 2 and 4 only
(d) 1, 2, 3 and 4

EXPLANATION:

The Quadrilateral Security Dialogue is a strategic dialogue between the United States, India, Japan
and Australia that is maintained by talks between member countries
So, Statement 1 is correct.

The EAS has 18 members - the ten ASEAN countries (Brunei, Cambodia, Indonesia, Laos, Malaysia,
Myanmar, the Philippines, Singapore, Thailand, Vietnam) along with Australia, China, India,
Japan, New Zealand, the Republic of Korea, Russia and the United States
So, Statement 2 is correct

The Asia-Africa Growth Corridor or AAGC is an economic cooperation agreement between the
governments of India, Japan and multiple African countries.
So, Statement 3 is correct.

The G4 nations, comprising Brazil, Germany, India, and Japan, are a group of four countries that
support each other's bids for permanent seats on the United Nations Security Council
So, Statement 4 is correct

ADDITIONAL INFORMATION:
International Groupings

Quad Formation:-
The group met for the first time in 2007 on the sidelines of the Association of
Southeast Asian Nations (ASEAN).
Objective:-
It is considered an alliance of maritime democracies, and the forum is
maintained by meetings, semi-regular summits, information exchanges and
military drills of all the member countries. One of the primary objectives of the
Quad is to work for a free, open, prosperous and inclusive Indo-Pacific region.

East Asia Summit Formation:-


The first summit was held in Kuala Lumpur, Malaysia on 14 December 2005
Objective:-
The East Asia Summit (EAS) is a regional forum held annually by leaders of,
initially, 16 countries in the East Asian, Southeast Asian, South
Asian and Oceanian regions, based on the ASEAN Plus Six mechanism.
Membership expanded to 18 countries including Russia and the United
States at the Sixth EAS in 2011. Since its establishment, ASEAN has held a
central role and leadership in the forum. It plays an important role in the
regional architecture of the Asia-Pacific
Asia-Africa growth Formation:-
corridor To begin with, India on 25 May 2017 launched a vision document for Asia-
Africa Growth Corridor or AAGC at the African Development Bank meeting in
Gujarat.
Objective:-
It aims for Indo-Japanese collaboration to develop quality infrastructure in
Africa, complemented by digital connectivity, which would undertake the
realization of the idea of creating a free and open Indo-Pacific Region

Q.67) Consider the following statements about the “Trend inflation”:

1. It is the underlying component of inflation to which actual inflation converges after a shock.

2. It is consistent with the potential level of output, the natural rate of unemployment and the natural
rate of interest.

3. At present, the estimate of trend inflation for India is 4.1-4.3 per cent.

Which of the statements given above is/are correct?


(a) 1 and 2 only
(b) 2 and 3 only
(c) 1 only
(d) 1, 2 and 3

Q.68) Which one of the following best explains “Capital Adequacy Ratio”?

(a) Banks maintaining some assets in non-cash form with themselves


(b) It is the percentage of total capital to total risk-weighted assets of the bank
(c) It is the ratio between the amount of capital required and the increased output produced
(d) It is the percentage of money allocated for developing Capital assets out of the total Capital
Expenditure

EXPLANATION:

The capital adequacy ratio (CAR) is a measure of how much capital a bank has available, reported as
a percentage of a bank's risk-weighted credit exposures. The purpose is to establish that banks have
enough capital on reserve to handle a certain amount of losses, before being at rof of becoming
insolvent.
So, Option (b) is correct.

ADDITIONAL INFORMATION:
Capital Adequacy Ratio

Formula It is decided by central banks and bank regulators to prevent commercial


banks from taking excess leverage and becoming insolvent in the
process.: It is measured as

Capital Adequacy Ratio = (Tier I + Tier II + Tier III (Capital funds)) /Risk-
weighted assets
Basel III Norms The risk-weighted assets take into account credit risk, market risk and
operational risk.
The Basel III norms stipulated a capital to risk-weighted assets of 8%.
However, as per RBI norms, Indian scheduled commercial banks are
required to maintain a CAR of 9% while Indian public sector banks are
emphasized to maintain a CAR of 12%.

Q.69) Consider the following statements:

1. Banks and other financial institutions borrow money from RBI for the short-term at Repo rate.

2. Banks and other financial institutions purchase government securities from RBI at Reverse repo
rate.

Which of the statements given above is/are correct?

(a) 1 only
(b) 2 only
(c) 1 and 2 only
(d) Neither 1 nor 2
EXPLANATION

The rate at which commercial banks and financial institutions borrow from RBI by mortgaging their
dated government securities and treasury bills is called the Repo rate.

So, Statement 1 is correct.

The rate at which RBI borrows from commercial banks by mortgaging its dated government securities
and treasury bills. It cannot purchase government securities at reverse repo rate.

So, Statement 2 is not correct.

ADDITIONAL INFORMATION

Repo rate  Repo rate in India is controlled by RBI


 Changed made in the Repo rate can directly impact the
economy
 If the Repo rate is increased
1. The bank can reduce the borrowing
2. The bank can borrow at a higher rate from RBI

Reverse Repo rate  The Reverse Repo rate in India is controlled by RBI
 If the Reverse Repo rate is increased
1. The bank can lend to RBI at a higher interest rate
2. The bank can lend to the customer at a higher interest
rate

Q.70) Consider the following statements:

1. Nidhi companies are a type of Non-Banking Financial Companies.

2. The registration and regulation of Nidhi companies are done by State governments.

3. The deposits received by Nidhi companies are not much when compared to the organised banking
sector.

Which of the statements given above is/are correct?

(a) 1 only
(b) 2 only
(c) 1 and 2 only
(d) 1 and 3 only

EXPLANATION

Nidhi Company is a type of Non-Banking Financial Company(NBFC). It is formed to borrow and lend
money to its members. It inculcates the habit of saving among its members and works on the principle
of mutual benefit.
So, Statement 1 is correct.
It is registered as a public company and should have “Nidhi Limited” as the last words of its name. It
does not require registration and regulation by the State governments.
So, Statement 2 is not correct.
Nidhi company should accept deposits from its members only. It shall not take deposits from any Body
corporate. Deposits shall not be accepted from any minor, so the deposits received by Nidhi companies
are not much when compared to the organized banking sector.
So, Statement 3 is correct.

ADDITIONAL INFORMATION

Nidhi Company
A Company which has been incorporated as a Nidhi with the object of cultivating the habit of thrift and
saving amongst its members, receiving deposits from, and lending to, its members only, for their
mutual benefit.
Acceptance of deposits  It should accept deposits from its members only
 It can’t deal with
1. chit funds
2. hire-purchase finance
3. leasing finance
4. insurance or securities business
Requirements  A minimum of seven members is required to start a
Nidhi Company
 A minimum of 5 lakh rupees, is required as the equity
share capital to start a Nidhi Company
Regulations  It is a company registered under the Companies Act,
2013
 It is regulated by the Ministry of Corporate Affairs.

Q.71) Which of the following is/are fully-owned subsidiaries of the Reserve Bank of India?

1. National Housing Bank

2. Deposit Insurance and Credit Guarantee Corporation

3. Bharatiya Reserve Bank Note Mudran Private Limited

Select the correct answer using the code given below.

(a) 2 and 3 only


(b) 1 only
(c) 1 and 3 only
(d) 1, 2 and 3
EXPLANATION

The banking system in India is regulated by the Reserve Bank of India (RBI), the apex financial
institution in India. There are four fully owned subsidiaries of RBI which are controlled and owned by
RBI. They are Deposit Insurance and Credit Guarantee Corporation, Bharatiya Reserve Bank Note
Mudran Private Limited, Reserve Bank Information Technology Private Limited, Indian Financial
Technology, and Allied Services.
So, Statements 2 and 3 are correct.
ADDITIONAL INFORMATION

Deposit Insurance and Credit Guarantee  It was established under the DICGC act 1961,
Corporation on 1978
 It ensures money of the deposit holders in
banks, if the bank fails to pay
 All commercial banks including branches of a
foreign bank in India, local area bank, and
RRB are insured under it except NBFC and
primary cooperative society
Bharatiya Reserve Bank Note Mudran  It was established in 1995
Private Limited  To address the demand and supply for
banknotes
 Its function is to print the banknotes for the
Reserve Bank of India (RBI)
Reserve Bank Information Technology  It was established in July 2016
Private Limited  To improve the cyber resilience of the Indian
banking industry
Indian Financial Technology and Allied  It is established under Section 8 of the Indian
Services Companies Act, 2013
 To design, deploy & provide essential IT-related
services for the Reserve Bank of India and other
financial institutions

Q.72) Consider the following statements:

1. Currently, the Reserve Bank of India acts as a banker to all the State Governments in India except
Sikkim.

2. All the State Governments and the Central Government is required to maintain a minimum cash
balance with the Reserve Bank of India.

Which of the statements given above is/are correct?

(a) 1 only
(b) 2 only
(c) Both 1 and 2
(d) Neither 1 nor 2

EXPLANATION

The main function of the Reserve Bank of India is that it acts as a banker and debt manager to the
government. It acts as a banker to the government both center and state except Sikkim. Previously
Jammu and Kashmir were also exempted under this but later included by the RBI agreement.
So, Statement 1 is correct.
All the State Governments are required to maintain a minimum balance with the Reserve Bank, which
varies from state to state depending on the relative size of the state budget and economic activity.
Under the administrative arrangements, the Central Government is required to maintain a minimum
cash balance with the Reserve Bank.
So, Statement 2 is correct.

ADDITIONAL INFORMATION

Reserve Bank of India


RBI as a banker and debt manager to  Act as a banker and debt manager to both center and
government state
 Exception
1. Sikkim
2. Jammu and Kashmir until recent time
Sikkim  Sikkim was made an associate state and then given
the status of a state later
 Sikkim state enjoys a special status under article
371F
 The Banking Regulation Act 1949, and Companies
Act do not extend to Sikkim.
Jammu and Kashmir  The Reserve bank of India has agreed with the
Jammu and Kashmir government to act as the sole
agent for the investment of government's funds
 To Carry out the general banking business of the
state government.
 The move would take away the special status enjoyed
by J&K Bank previously

Q.73) Why do the forests in the Central Indian regions have frequent forest fires?

(a) Because the monsoon rainfall is not reaching this region


(b) The predominant vegetation in this region is dry deciduous forests
(c) They are manmade caused by those collecting Mahua flowers and Tendu leaves inside the
forests
(d) Due to the practice of slash and burn agriculture in this region, forest fires are frequently occurring
EXPLANATION:

The eastern parts of the central Indian states such as Chhattisgarh and Madhya Pradesh will receive
the monsoonal rainfall of 100-150cms but the western parts will receive less than that due to the
interior of the continents. So, option (a) is not correct.
The forest type which can be seen here is both dry and moist deciduous forests, it is not always prone
to forest fires but fire may happen before the trees shed the leaves. So, option (b) is not correct.
In recent times, in the central Indian states, fire is caused by human beings by the activities such as
disposing of cigar butts or matchsticks or setting fires to collect flowers or leaves. So, option (c) is
correct.
The method of slash and burn agriculture in this region will cause forest fires as this involves the
deforestation of forests and it is predominant in the northeastern region. So, option (d) is not correct.

ADDITIONAL INFORMATION:

FOREST FIRES IN INDIA


VULNERABILITIES • Based on the forest inventory records, 54.40% of forests in
India are exposed to occasional fires, 7.49% to moderately
frequent fires, and 2.405 to high incidence levels while
35.71% of India’s forests have not yet been exposed to fires
of any real significance.
• Vulnerable areas-northeast India and central India.
EXAMPLES • Prolonged fires in Uttarkhand, Himachal Pradesh
• Similipal national park in orissa
• Bandhavgarh national park in Madhya Pradesh
CAUSES • Natural causes:
-Lightning, heat patterns, etc
• Man-made causes:
-Smoking, campfires, fireworks.

MEASURES • National action plan on forest fires and their protection


scheme.
• In 2004, FOREST SURVEY OF INDIA developed the forest
alert system, and recently satellite information was
gathered from NASA and ISRO.

Q.74) Consider the following statements:

1. Under the Constitution, Supreme Court has the power to transfer cases from the High Courts to
itself if cases involve the same questions of law.
2. Only the Supreme Court and not the High Courts play a vital role in the constitutional scheme in
India.

Which of the statements given above is/are correct?

(a) 1 only
(b) 2 only
(c) Both 1 and 2
(d) Neither 1 nor 2

EXPLANATION:

The supreme court has the power to transfer the cases from the high court to itself if the core of the
matter is the same because it is dealt with both the high court and supreme court at the same time in
this case the decree of the order will result in overlapping decisions to follow that’s why under the
constitution it says the Supreme Court is the ultimate interpreter of the Constitution. It can give a
final version to the spirit and content of the provisions of the constitution and the verbiage used in
the constitution. So, Statement 1 is correct.

In the Indian constitution, it is said that both the Supreme Court and the High Court have the
power to interpret the constitutional laws in the country. So, Statement 2 is not correct.

Q.75) Consider the following statements about the “Financial Inclusion Index”:

1. It is a comprehensive index incorporating details of banking, investments, insurance, postal and


pension sector.

2. It has been constructed with 2018 as a base year.

3. It is published annually by the Reserve Bank of India.

Which of the statements given above is/are correct?

(a) 1 and 2 only


(b) 1 and 3 only
(c) 3 only
(d) 1, 2 and 3

EXPLANATION

The Financial Inclusion Index has been conceptualized as a comprehensive index incorporating
details of banking, investments, insurance, postal well as the pension sector in consultation with
the Government and respective sectoral regulators. So, Statement 1 is correct.

The Financial Inclusion Index has been constructed without any ‘base year’ and as such it reflects
cumulative efforts of all stakeholders over the years towards financial inclusion. So, Statement 2
is not correct.

The Financial Inclusion Index will be published annually by the Reserve Bank of India in July every
year. So, Statement 3 is correct.

ADDITIONAL INFORMATION

Financial Inclusion Index

Financial  Financial Inclusion means making financial products and services


Inclusion affordable to individuals and businesses for transactions, payments,
savings, and credit and insurance.
 To enable inclusive and sustainable development.
Financial  RBI of introduced the Financial Inclusion Index(FI-Index) to calculate
how many people in the country have access to banking and financial
Inclusion
Index services and products and the usage and quality of such facilities.
 Scale from 0 to 100
 0 represents complete financial exclusion
 100 represents complete financial inclusion
 Parameters – there are three broad parameters based on 97 indicators
1. Access – 35%
2. Usage – 45%
3. Quality – 20%
 The FI-Index has been constructed without any ‘base year’.
 The FI-Index will be published annually in July every year.
Steps taken for  Nationalization of banks in 1969 and 1980
 Laying down of priority sector requirements
Financial
 Lead bank scheme to the introduction of regional rural banks
Inclusion
Index  Pradhan mantri jan dhan yojana (PMJDY)
 Two insurance schemes – Pradhan Mantri Suraksha Bima Yojana
(PMSBY)and Pradhan Mantri Jeevan Jyoti Bima Yojana (PMJJBY)

Q.76) The provisions of “Priority Sector Lending” shall apply to

1. Regional Rural Bank

2. Small Finance Bank

3. Local Area Bank

4. Urban Co-operative Bank

Select the correct answer using the code given below.

(a) 1 and 3 only


(b) 2 and 4 only
(c) 1, 2 and 4 only
(d) 1, 2, 3 and 4

EXPLANATION

Priority Sector Lending issued by the Reserve Bank of India includes Commercial Banks, Small
Finance Banks, Regional Rural Banks, Urban Co-operative Banks, and Local Area Banks. So,
Option (d) is correct.

ADDITIONAL INFORMATION

Priority Sector Lending

Priority  The objective of the priority sector lending program is to ensure


adequate credit flows into some of the vulnerable sectors of the economy.
Sector
Lending  This may not be attractive for banks as it may not be profitable.

Categories  Agriculture
 Micro, Small and Medium Enterprises
 Export Credit
 Education
 Housing
 Social Infrastructure
 Renewable Energy
 Others

 40% of Adjusted Net Bank Credit (ANBC - outstanding Bank Credit in


Targets
India) for all Domestic commercial banks and foreign banks with 20
branches and above.
 75% of Adjusted Net Bank Credit for Regional Rural Banks and Small
Finance Banks.
 The overall PSL target of only urban co-operative banks will go up in a
phased manner from the existing target of 40 percent to 75 percent by
March-end 2024.
Sub-targets  Agriculture – 18%
 Micro Enterprises – 7.5%
 Weaker Sections – 10%
Non-  Banks having any shortfall in lending to the priority sector shall be
achievement allocated amounts for contribution to the Rural Infrastructure
of targets Development Fund (RIDF) established with NABARD and other funds
with NABARD/NHB/SIDBI/ MUDRA Ltd., as decided by the Reserve
Bank from time to time.

Q.77) Consider the following statements about the “National Electronic Funds Transfer” (NEFT):

1. It is a nationwide centralised payment system owned and operated by the Reserve Bank of India.

2. There is no limit for funds transfer through the NEFT system.

3. The outbound remittance through the NEFT system is completely prohibited.

4. It is available round the clock throughout the year on all days.

Which of the statements given above are correct?

(a) 1 and 2 only


(b) 3 and 4 only
(c) 1, 2 and 4 only
(d) 1, 2, 3 and 4

EXPLANATION

National Electronic Funds Transfer (NEFT) is an electronic payment system to quickly transfer
money between banks throughout India. It is a nationwide centralized payment system owned and
operated by the Reserve Bank of India (RBI). So, Statement 1 is correct.

There is no limit for funds transfer through the NEFT system imposed by the RBI. However, banks
may place amount limits based on their risk perception with the approval of its Board. ). So,
Statement 2 is correct.

The outbound remittances through the NEFT system are permitted only to Nepal under Indo-Nepal
Remittance Scheme. Under this scheme, the remitter can transfer funds from any of the NEFT-
enabled bank branches in India to Nepal, irrespective of whether the beneficiary in Nepal maintains
an account with a bank branch in Nepal or not. The beneficiary would receive funds in Nepalese
Rupees. So, Statement 3 is not correct.

The NEFT system is available round the clock throughout the year on all days, i.e., on a 24x7x365
basis. NEFT presently operates in batches at half-hourly intervals throughout the day. In case of
non-availability of NEFT for any reason, an appropriate message will be broadcast by RBI to all
system participants. So, Statement 4 is correct.

ADDITIONAL INFORMATION

National Electronic Funds Transfer (NEFT)

Money transfer  Technological advancement has made online money transfer from one
account to another account easier.
 Some of the famous methods are
1. National Electronic Funds Transfer (NEFT)
2. Real-Time Gross Settlement (RTGS)
3. Immediate Payment Service (IMPS)
Comparison Means of NEFT RTGS IMPS
comparison
Minimum Re.1 Rs.2 Lakh Re.1
transfer value
Maximum No limit No limit Rs.2 Lakh
transfer value
Payment Online and Online and Online
Options Offline Offline
Settlement Half hourly Real-Time Real-Time
type basis
Inward No charges No charges Decided by
Transaction the individual
charges member
banks

Q.78) Which one of the following statements is not correct about “e-RUPI”?

(a) It does not require the beneficiary to have a bank account

(b) It is a cashless voucher-based mode of payment

(c) It is developed and launched by the Reserve Bank of India

(d) It is provided in the form of an SMS or QR code

EXPLANATION

e-RUPI requires users to have only mobile phone and e-voucher and does not need to have a digital
payment app or a bank account. So, Option (a) is not correct.
e-RUPI is developed to have a cashless and contactless payment collection which is a digital
voucher. So, Option (b) is not correct.

e-RUPI is developed with the support of DFS (Department of Financial Services) and NHA (National
Health Authority) and is powered by the National Payments Corporation of India (NPCI). So, Option
(c) is correct.
e-RUPI is a digital voucher which a beneficiary gets on his phone in the form of an SMS or QR
code. It is a pre-paid voucher, which he/she can go and redeem at any center that accepts it. So,
Option (d) is not correct.

ADDITIONAL INFORMATION

e-RUPI

Digital  The Digital India program is a flagship program of the Government of India
India with a vision to transform India into a digitally empowered society and
knowledge economy. “Faceless, Paperless, Cashless” is one of the professed
roles of Digital India.
 The Vision is to provide the facility of seamless digital payment to all citizens
of India in a convenient, easy, affordable, quick, and secured manner.
 It is under the Ministry of Electronics & Information Technology (MeitY)
 Prime Minister launched e-RUPI, a cashless and contactless instrument for
digital payment.
 e-RUPI is a digital solution launched by the Hon'ble PM via video
e-RUPI
conferencing to allow a cashless payment solution for COVID-19 vaccination.
 It plays a major role in strengthening Direct-Benefit Transfer and making it
more transparent.
Developed  The National Payments Corporation of India (NPCI), which oversees the
by digital payments ecosystem in India, has launched e-RUPI, a voucher-based
payments system to promote cashless transactions.
 It has been developed in collaboration with the Department of Financial
Services, Ministry of Health & Family Welfare, and National Health
Authority.
Working  e-RUPI is a digital voucher which a beneficiary gets on his phone in the form
of an SMS or QR code.
 It is a pre-paid voucher, which he/she can go and redeem at any center that
accepts it.
 e-RUPI is a person-specific, even purpose-specific digital voucher.
Real-time  If the Government wants to cover a particular treatment of an employee in a
Example specified hospital, it can issue an e-RUPI voucher for the determined amount
through a partner bank.
 The employee will receive an SMS or a QR Code on his feature
phone/smartphone.
 He/she can go to the specified hospital, avail of the services, and pay through
the e-RUPI voucher received on his phone.
Advantage  It does not require a bank account.
 It is a contactless two-step redemption process that does not require sharing
of personal details either.
 e-RUPI is operable on basic phones also.
 Therefore it can be used by persons who do not own smartphones or in places
that lack internet connection or without an internet connection.
Issuer  NPCI has partnered with 11 banks for e-RUPI transactions.
 They are Axis Bank, Bank of Baroda, Canara Bank, HDFC Bank, ICICI Bank,
Indian Bank, IndusInd Bank, Kotak Mahindra Bank, Punjab National Bank,
State Bank of India, and Union Bank of India.
Usage  To begin with, NPCI has tied up with more than 1,600 hospitals where e-RUPI
can be redeemed.

Q.79) Consider the following statements:

1. Corporate insolvency and personal insolvency are part of the Insolvency and Bankruptcy Code.

2. Insolvency and Bankruptcy Code does not cover cross-border insolvency.

3. Both National Company Law Tribunal and National Company Law Appellate Tribunal are formed
under the Insolvency and Bankruptcy Code.

Which of the statements given above is/are correct?

(a) 2 and 3 only


(b) 1 and 2 only
(c) 1 only
(d) 1, 2 and 3

EXPLANATION:

The Code provides a time-bound process for resolving insolvency in companies and among individuals.
Insolvency is a situation where individuals or companies are unable to repay their outstanding debt.
So, Statement 1 is correct.

Sections 234 and 235 of the Code which envisages entering into bilateral agreements and issuance of
letters of request to foreign courts by Adjudicating Authorities under the Code resulted in an ad-hoc
framework that was susceptible to delay and uncertainty for creditors and debtors as well as for courts.
The committee was recently formed and the draft bill was under the consideration to give full effect to
the cross-border insolvency. So, Statement 2 is not correct.

The companies act 2013 give effect to the formation of both the National Company Law Tribunal and
Appellate tribunal to deal with companies and limited liabilities and the Debt Recovery Tribunal for the
Individuals and partnership firms. So, Statement 3 is not correct.

ADDITIONAL INFORMATION:

INSOLVENCY AND BANKRUPTCY CODE

OBJECTIVES  To Consolidate all existing Insolvency laws in India.


 To have better and simple insolvency proceedings in
India.
 To protect the interest of creditors and individuals who
are involved in this.
 To set up an Insolvency and Bankruptcy Board of India.
 To promote entrepreneurship.

PARTIES INVOLVED  Insolvency professionals and agencies, information


utilities, and the adjudicating authorities such as
National Company Law Tribunal.

RECENT CHANGES  The bill has been drafted to include more information
regarding cross-border insolvency.
 The Insolvency amendment bill,2021:
-The Ordinance introduces an alternate insolvency
resolution process for micro, small, and medium
enterprises (MSMEs), called the pre-packaged
insolvency resolution process (PIRP).

Q.80) Consider the following terms:

1. Public Issue

2. Rights Issue

3. Private Placement

To which one of the following are terms given above are related?

(a) Ways in which a company raises capital in the secondary market


(b) Ways in which goods are sold in Commodity trading
(c) Ways in which a company raises money in the primary market
(d) Terms related to foreign institutional investment

EXPLANATION:

The secondary market only acts as a medium for buying and selling of securities that were already
issued in the New issue market of the primary market which consists of only the Stock Exchange
and Over-The-Counter Exchange (unlisted securities). So, option (a) is not correct.
A commodity market involves buying, selling, or trading a raw product, such as oil, gold, or coffee.
It consists of derivatives such as futures, options, and forwards. So, option (b) is not correct.
The securities which are issued in the new issue market are called primary market which consists
of a public issue (first time issued to the general public) which is also called as Initial Public
Offering, whereas the Right Issue refers to offering the security to the existing shareholders of the
company, Private Placement means offering shares directly to the financial institutions, mutual
funds, and high worth investors. So, option (c) is correct.

A foreign institutional investor (FII) is an investor or investment fund investing in a country


outside of the one in which it is registered or headquartered. The hedge funds, pension funds,
mutual funds, and participatory notes are some of the forms involved in this. So, option (d) is not
correct.

Q.81) Consider the following statements about the “Ways and Means Advances”:

1. It is a facility available only to the central government.

2. Government can use this facility as a main source of finance to compensate for the budget deficit.

3. Government has to return the amount within a year.

Which of the statements given above is/are not correct?

(a) 1 and 2 only


(b) 2 and 3 only
(c) 1 only
(d) 1, 2 and 3

EXPLANATION :

The Reserve Bank of India (RBI) gives temporary loan facilities to the central and state
governments. This loan facility is called Ways and Means Advances (WMA).
So, statement 1 is not correct.

These borrowings are meant purely to help them to tide over temporary mismatches in cash
flows of their receipts and expenditures. In that sense, they aren’t a source of finance per se.
So, statement 2 is not correct.
Section 17(5) of the RBI Act, 1934 authorises the central bank to lend to the Centre and state
governments subject to their being repayable “not later than three months from the date of the
making of the advance” (and therefore it is 90 days and not one year).
So, statement 3 is not correct.

ADDITIONAL INFORMATION :

Ways and means advances :

About • The Reserve Bank of India (RBI) gives temporary loan facilities to the central and
state governments. This loan facility is called Ways and Means Advances (WMA).
• The Ways and Means Advances scheme was introduced in 1997.
Purpose • The Ways and Means Advances scheme was introduced to meet mismatches in
the receipts and payments of the government.
Working • The government can avail of immediate cash from the RBI, if required. But it
has to return the amount within 90 days. Interest is charged at the existing
repo rate.

• If the WMA exceeds 90 days, it would be treated as an overdraft (interest rate on


overdrafts is 2 percentage points more than the repo rate).

Types • There are two types of Ways and Means Advances — normal and special.
• Special WMA or Special Drawing Facility is provided against the collateral of the
government securities held by the state. After the state has exhausted the limit
of SDF, it gets normal WMA.
• The interest rate for SDF is one percentage point less than the repo rate.
• The number of loans under normal WMA is based on a three-year average of
actual revenue and capital expenditure of the state.

Limits • The limits for Ways and Means Advances are decided by the government and
RBI mutually and revised periodically.
• For the Centre, the WMA limit during the first half of 2020-21 (April-September)
has been fixed at Rs 120,000 crore.
• This is 60% higher than the Rs 75,000 crore limit for the same period of 2019-
20. The limit for the second half of the last fiscal (October-March) was Rs
35,000 crore.
• For the states, the aggregate WMA limit was Rs 32,225 crore till March 31,
2020.
• On April 1, the RBI announced a 30% hike in this limit, which has now been
enhanced to 60%, taking it to Rs 51,560 crore.

Q.82) Consider the following statements about the “National Bank for Financing Infrastructure and
Development” (NaBFID):

1. It is a corporate body with authorized share capital of Rs. 1 Lakh crore.

2. It is placed under the purview of the Department of Economic Affairs.


3. It may raise money in the form of loans or otherwise both in Indian Rupees or foreign currencies.

Which of the statements given above is/are correct?

(a) 1 and 2 only


(b) 1 and 3 only
(c) 2 only
(d) 1, 2 and 3

EXPLANATION :

NBFID is as a corporate body with authorised share capital of one lakh crore rupees.
Shares of NBFID may be held by:
(i) central government,
(ii) multilateral institutions,
(iii) sovereign wealth funds,
(iv) pension funds,
(v) insurers,
(vi) financial institutions,
(vii) banks, and
(viii) any other institution prescribed by the central government.
Initially, the central government will own 100% shares of the institution which may subsequently
be reduced up to 26%.
So, statement 1 is correct.

National Bank for Financing Infrastructure and Development (NABFID) is placed under the
purview of the Department of Financial Services (DFS) in the Finance Ministry for
administration purposes.
So, statement 2 is not correct.

NBFID may raise money in the form of loans or otherwise both in Indian rupees and foreign
currencies, or secure money by the issue and sale of various financial instruments including
bonds and debentures.
So, statement 3 is correct.

ADDITIONAL INFORMATION :
National Bank for Financing Infrastructure and Development

Purpose • National Bank for Financing Infrastructure and Development (NBFID) is the
principal development financial institution (DFIs) for infrastructure financing.
• DFIs are set up for providing long-term finance for such segments of the
economy where the risks involved are beyond the acceptable limits of
commercial banks and other ordinary financial institutions. Unlike banks, DFIs
do not accept deposits from people.
• They source funds from the market, government, as well as multi-lateral
institutions, and are often supported through government guarantees.
Functions • extending loans and advances for infrastructure projects,
• taking over or refinancing such existing loans,
• attracting investment from private sector investors and institutional investors
for infrastructure projects,
• organising and facilitating foreign participation in infrastructure projects,
• facilitating negotiations with various government authorities for dispute
resolution in the field of infrastructure financing, and
• providing consultancy services in infrastructure financing.
Source of • NBFID may borrow money from:
Funds o central government,
o Reserve Bank of India (RBI),
o scheduled commercial banks,
o mutual funds, and
o multilateral institutions such as World Bank and Asian Development
Bank.
Management • NBFID will be governed by a Board of Directors. The members of the Board
include:
o the Chairperson appointed by the central government in consultation
with RBI,
o a Managing Director,
o up to three Deputy Managing Directors,
o two directors nominated by the central government,
o up to three directors elected by shareholders, and
o a few independent directors (as specified).
Role of • The central government will provide grants worth Rs 5,000 crore to NBFID by
Central the end of the first financial year.
Govt. • The government will also provide guarantee at a concessional rate of up to 0.1%
for borrowing from multilateral institutions, sovereign wealth funds, and other
foreign funds.
• Costs towards insulation from fluctuations in foreign exchange (in connection
with borrowing in foreign currency) may be reimbursed by the government in
part or full.
• Upon request by NBFID, the government may guarantee the bonds, debentures,
and loans issued by NBFID.
Role of RBI • RBI may grant a licence for DFI in consultation with the central government.
RBI will also prescribe regulations for these DFIs.

Q.83) Consider the following statements:

1. Liquidity trap is a contradictory economic situation in which interest rates are very high and
savings rates are low.

2. During a liquidity trap, consumers choose to avoid their funds in cash savings and invest in bonds.

Which of the statements given above is/are correct?

(a) 1 only
(b) 2 only
(c) Both 1 and 2
(d) Neither 1 nor 2
EXPLANATION :

A liquidity trap is a contradictory economic situation in which interest rates are very low and savings
rates are high, rendering monetary policy ineffective.

So, statement 1 is not correct.

As described by economist John Maynard Keynes, during a liquidity trap, consumers choose to avoid
bonds and keep their funds in cash savings because of the prevailing belief (Speculation) that interest
rates could soon rise (which would push bond prices down).

Because bonds have an inverse relationship to interest rates, many consumers do not want to hold an
asset with a price that is expected to decline.

At the same time, central bank efforts to spur economic activity are hampered as they are unable to
lower interest rates further to incentivize investors and consumers.

So, statement 2 is not correct.

ADDITIONAL INFORMATION :

Liquidity Trap :

Money Supply Money is the most liquid of all assets in the sense that it is universally
and Demand acceptable and hence can be exchanged for other commodities very easily.
If, instead of holding on to a certain cash balance, you put the money in a fixed
deposits in some bank you can earn interest on that money (opportunity cost).
People desire to hold money balance broadly from two motives.
1. Transaction Motive
2. Speculation Motive
Speculation Speculative demand for money is inversely related to the rate of interest, i.e.,
Motive higher the rate of Interest, smaller wall be speculative demand for money and
vice versa.
(i) If market rate of interest is very high and everyone expects it to fall
in the future (i.e., rise in price of bond) thereby anticipating capital gain
from bond holding, people will convert their money into bonds. Thus,
speculative demand for money is low.
(ii) On the contrary, if the rate of interest is low and people expect it to
rise in future (i.e., fall in price of bond) anticipating capital loss from
bond holding, people convert their bonds into money in order to avoid
future capital loss. They hold up money balance thinking that income
from nonmonetary assets like bond will be low and so the cost of money
holding will also be low.
(iii) Thus, speculative demand for money becomes very high so much so
that when the rate of interest declines to minimum, in such a situation,
speculative demand for money becomes infinite (perfectly elastic).
(iv) This pushes the economy into liquidity trap and the speculative
demand curve becomes flat as shown in the figure.
Q.84) Consider the following statements:

The effect of ‘Quantitative easing’ is that it necessarily

1. increase the demand for bonds and securities


2. inject liquidity in the banking system
3. increase aggregate demand while keeping inflation within target.

Select the correct answer using the code given below.

(a) 1 and 2 only


(b) 2 only
(c) 1 and 3 only
(d) 1, 2 and 3

EXPLANATION :

Quantitative easing is an occasionally used monetary policy, which is adopted by the government to
increase money supply in the economy in order to further increase lending by commercial banks and
spending by consumers.

The Reserve Bank of India infuses a pre-determined quantity of money into the economy by buying
financial assets from commercial banks and private entities (government bonds or other financial
assets (e.g., municipal bonds, corporate bonds, stocks, etc.)). This leads to an increase in banks'
reserves.

Since RBI infuses liquidity through purchase of bonds the demand for bonds increases.

So, statement 1 is correct.


Quantitative easing (QE) is done by central banks in order to inject money into the economy to expand
economic activity.

So, statement 2 is correct.


Quantitative easing is considered to be an unconventional form of monetary policy, which is usually
used when inflation is very low or negative, and when standard monetary policy instruments have
become ineffective.
Infusion of Liquidity helps to increase the aggregate demand in economy and since it is already used in
a low inflation scenario it keeps inflation within target.

So, statement 3 is correct.


ADDITIONAL INFORMATION :

Quantitative Easing :

About • Quantitative easing is an unconventional monetary policy to increase the


money supply.
• Central banks target the supply of money by buying or selling government
bonds.
• When the economy stalls and the central bank wants to encourage economic
growth, it buys government bonds.
• This lowers short-term interest rates and increases the money supply.
• Central banks like the European Central Bank,U.S. Federal Reserve have
adopted this policy to boost their respective economies.
Helicopter • Helicopter money is an unconventional monetary policy tool, which involves
Money printing large sums of money and distributing it to the public, to stimulate the
economy during a recession (decline in general economic activity) or when
interest rates fall to zero.
• The term was coined by American economist Milton Friedman, It basically
denotes a helicopter dropping money from the sky.
• Under such a policy, a central bank "directly increases the money supply and,
via the government, distribute the new cash to the population with the aim of
boosting demand and inflation.
Open market • Open market Operations (OMOs) are the market operations conducted by the
Operations RBI by way of sale and purchase of G-Secs to and from the market.
• OMOs are done with an objective to adjust the rupee liquidity conditions in
the market on a durable basis.
• With this monetary tool the RBI manages and controls the liquidity, rupee
strength and monetary management through purchase and sale of
government securities (G-Secs).
• When the RBI feels that there is excess liquidity in the market, it resorts to
sale of securities thereby sucking out the rupee liquidity.
• Similarly, when the liquidity conditions are tight, the RBI may buy securities
from the market, thereby releasing liquidity into the market.
Operation • Operation Twist is the name given to a US Federal Reserve monetary policy
Twist operation.
• It involves the purchase and sale of government securities to boost the
economy by bringing down long-term interest rates.
• It normally leads to lower longer-term yields, which will help boost the
economy by making loans less expensive for those looking to buy homes, cars
and finance projects.
• But saving becomes less desirable because it doesn’t pay as much interest.
Q.85) The role of the capital market include

1. Mobilisation of savings

2. Acceleration of capital formation

3. Promotion of industrial growth

4. Raising of both short-term and long-term capital

5. Regulation of funds

Select the correct answer using the code given below.

(a) 2 and 3 only


(b) 1, 4 and 5 only
(c) 1, 2, 3 and 5 only
(d) 1, 2, 3, 4 and 5

EXPLANATION :

A financial market facilitates the transfer of savings from savers to investors. It gives savers the
choice of different investments and thus helps to channelize surplus funds into the most productive
use
So, statement 1 is correct.

Investors use money from capital market in further investments and thus accelerating capital
formation.
So, statement 2 is correct.
Capital markets provide the most important factors of production (i.e.) Capital that helps in the
growth of industries.

So, statement 3 is correct.

Capital market encompasses the trade in both stocks and bonds. These are long-term assets bought
by financial institutions, professional brokers, and individual investors.

It is only in the money market the trade is in short-term debt. Where there is a constant flow of cash
between governments, corporations, banks, and financial institutions, borrowing and lending for a
term as short as overnight and no longer than a year.

So, statement 4 is not correct.

Capital markets not only helps in fund mobilisation, but it also helps in proper allocation of these
resources. It can have regulation over the resources so that it can direct funds in a qualitative
manner.
So, statement 5 is correct.

ADDITIONAL INFORMATION :

Financial Markets :

Financial • A business is a part of an economic system that consists of two main sectors –
Market households which save funds and business firms which invest these funds.
• A financial market helps to link the savers and the investors by mobilizing
funds between them.
• In doing so it performs what is known as an allocative function. It allocates or
directs funds available for investment into their most productive investment
opportunity.

• A financial market is a market for the creation and exchange of financial assets.
Classification

Money Treasury Bill:


Market • A Treasury bill is basically an instrument of short-term borrowing by the
Instruments Government of India maturing in less than one year.
• They are also known as Zero Coupon Bonds issued by the Reserve Bank of
India on behalf of the Central Government to meet its short-term requirement of
funds.
• Treasury bills are issued in the form of a promissory note.

Commercial Paper:
• Commercial paper is a short-term unsecured promissory note, negotiable and
transferable by endorsement and delivery with a fixed maturity period.
• It is issued by large and creditworthy companies to raise short-term funds at
lower rates of interest than market rates.
• It usually has a maturity period of 15 days to one year.

Call Money:
• Call money is short term finance repayable on demand, with a maturity period
of one day to fifteen days, used for inter-bank transactions.

Certificate of Deposit:
• Certificates of deposit (CD) are unsecured, negotiable, short-term instruments
in bearer form, issued by commercial banks and development financial
institutions.
• They can be issued to individuals, corporations and companies during periods
of tight liquidity when the deposit growth of banks is slow but the demand for
credit is high.
• They help to mobilise a large amount of money for short periods.

Commercial Bill:
• A commercial bill is a bill of exchange used to finance the working capital
requirements of business firms.
• It is a short-term, negotiable, self-liquidating instrument which is used to
finance the credit sales of firms.

• When goods are sold on credit, the buyer becomes liable to make payment on a
specific date in future.
• The seller could wait till the specified date or make use of a bill of exchange.
• The seller (drawer) of the goods draws the bill and the buyer (drawee) accepts it.
• On being accepted, the bill becomes a marketable instrument and is called a
trade bill.
• These bills can be discounted with a bank if the seller needs funds before the
bill matures.
• When a trade bill is accepted by a commercial bank it is known as a commercial
bill.

Q.86) Which one of the following statements is correct about the “SVAMITVA Scheme”?

(a) It is a centralised database about the migrant labours using the modern technology
(b) It is the scheme for surveying the land parcels in the rural inhabited area using Drone
technology
(c) It is a scheme that aims at creating social infrastructure in the rural regions
(d) It is a self-employment scheme aimed at the rural regions across the country

EXPLANATION:

Launched in April 2020, the SVAMITVA scheme aims at the establishment of clear ownership of
property in rural areas by mapping land parcels using drone technology.

So, Option (b) is correct.

ADDITIONAL INFORMATION:

SVAMITVA SCHEME

Objective

 The SVAMITVA (Survey of Villages and Mapping with Improvised


Technology in Village Areas) scheme was launched on April 24, 2020, as
a central sector scheme to promote a socio-economically empowered and
self-reliant rural India.
 The scheme aims at the establishment of clear ownership of property in
rural areas by mapping land parcels using drone technology and
providing a ‘record of rights’ to eligible households by issuing legal
ownership cards to them.

Implementing  The Ministry of Panchayati Raj (MoPR) is the Nodal Ministry for the
agency & Coverage implementation of the scheme.
 Under the pilot project, the scheme was successfully implemented in
Madhya Pradesh, Uttar Pradesh, Maharashtra, Uttarakhand, Rajasthan,
Haryana and Karnataka. Now, it will be rolled out in the entire country
to provide property cards to people.

Methodology  The plan is to survey all rural properties using drones and prepare GIS-
based maps for each village. The process begins with the signing of an
MoU between the Survey of India (SoI) and the State government
concerned.
 The SoI will use technology for topographical mapping, including
satellite imageries and drone platforms.

Benefits
 Digitisation of personal identity and agriculture land, and now a
residential property in rural areas through SVAMITVA, will facilitate
transparent transactions in land parcels.
 Non-farm related activities will benefit from the clear title and the
removal of land supply constraints.
 Clear title records, accompanied by the legalisation of land leasing,
will improve their access to credit, insurance and support services.
With digital records, banks can lend freely without much
documentation.
 Formal lease markets and digitisation of personal records can lead
to improved implementation of schemes such as PM-KISAN, Fasal
Bima Yojana and Rythu Bandhu. These can be directed towards the
cultivators, instead of absentee landlords. The main challenge for
SVAMITVA is to ensure Centre-State coordination and smooth
working of dispute settlement systems

Q.87) According to the recent report, titled “Climate Vulnerability Assessment for Adaptation Planning in
India Using a Common Framework” released by the Department of Science and Technology, which one
of the following pair of states is the most vulnerable states in India?

(a) All of the Himalayan States from Kashmir to Tripura


(b) All the eight North-Eastern States of India
(c) Chhattisgarh, Jharkhand, Mizoram, Odisha, Assam, Bihar, Arunachal Pradesh and West
Bengal
(d) All the North-Western States and the Coastal States of India

EXPLANATION:

Eight Indian states -- Jharkhand, Mizoram, Odisha, Chhattisgarh, Assam, Bihar, Arunachal Pradesh
and West Bengal -- are highly vulnerable to climate change, according to the report, titled 'Climate
Vulnerability Assessment for Adaptation Planning in India Using a Common Framework' released by
the Department of Science and Technology (DST),
The report identifies the most vulnerable states and districts in India with respect to current climate
risk and key drivers of vulnerability.
So, Option (c) is correct.
ADDITIONAL INFORMATION:

'Climate Vulnerability Assessment for Adaptation Planning in India Using a Common Framework'

The high
Vulnerability States
 These states, mostly in the eastern part of the country, require
prioritisation of adaptation interventions, the report pointed out. Among
all states, Assam, Bihar and Jharkhand have over 60 per cent of
districts in the category of highly vulnerable districts.

 Lack of forest area per 100 rural population was found to be one of the
major drivers of vulnerability for Assam even though the state has a
forest cover of 42 per cent, followed by low road density, the report said.

 In the case of Bihar, the report cited poor health infrastructure to be the
key vulnerability driver in 36 districts, followed by a high percentage of
marginal and small operational holders in 24 districts.

 The lack of implementation of the rural employment scheme MGNREGA,


causing a lack of alternative livelihood opportunities, appeared as a key
driver in 14 districts of Bihar, followed by a lack of women’s
participation in the workforce in 11 districts.

 Lack of crop insurance and rainfed agriculture were key drivers of


vulnerability for Jharkhand.

Low vulnerability Himachal Pradesh, Telangana, Sikkim and Punjab have been categorised as
states lower-middle vulnerable states. Uttarakhand, Haryana, Tamil Nadu, Kerala,
Nagaland, Goa and Maharashtra have been categorised as states with low
vulnerability.

Q.88) Consider the following activities:

1. A foreigner visiting India for tourism.


2. An Indian IT firm providing software solutions to a foreign client.
3. An Indian student getting an education from a foreign university.
4. A foreign resident of India accessing healthcare facilities in a hospital in India.

Which of the above are the examples of ‘Indian export of services’?

(a) 1 and 2 only


(b) 2 only
(c) 1, 2 and 4 only
(d) 1, 2, 3 and 4

EXPLANATION

Export refers to goods and services that are produced in one country and sold to buyers in another
country. A foreigner visiting India for tourism is an export of service, since the foreign nationals will
buy Indian goods and use Indian services, India will get foreign currency in this process.
So, Statement 1 is correct.
An Indian IT firm with a Foreign client is exporting IT software solution service to the Foreign Client
for which India will get foreign currency as it is sold to buyers in another country.
So, Statement 2 is correct.

Indian students getting an education from foreign universities will be included under the import of
services. The student is gaining education service and India is giving Indian Currency to the foreign
nation for rendering the service. The service received is taxable in the form of education tuition fees
it is not an export of service.
So, Statement 3 is not correct.

A foreign resident in India is different from a foreign tourist so the service availed to them will not be
considered as an Export. Because India will not get any foreign currency in this provided service.
So, Statement 4 is not correct.

Q.89) Which of the following is likely to result in an appreciation of the Indian Rupee in comparison to
the US Dollar?

(a) Increase in India's imports


(b) Increase in investments by Indians in the United States of America
(c) Increase in remittances into India from the United States of America
(d) Increase in inter-state trade in India

EXPLANATION

Appreciation of the Indian rupee occurs when the value of the rupee increases compared to US dollars.
An increase in India's imports from America will cause a decrease in the value of rupees due to the
payment of transactions for goods and services imported through the dollar.
So, Option (a) is not correct.
Investment by Indians in America results in an increased outflow of dollars from India in the form of
investment capital. This caused a decrease in the value of the Indian Rupee in comparison to the dollar
So, Option (b) is not correct.
Increased remittances into India from America bring the US dollar into India, hence increasing the
value of the rupee causing appreciation of the rupee.
So, Option (c) is correct.
An increase in inter-state trade doesn't have a significant effect on the appreciation of the Indian rupee.
So, Option (d) is not correct.

ADDITIONAL INFORMATION

Appreciation of Indian Rupee


Appreciation • The rise in the currency value of a country in relation to a
foreign currency is called appreciation in the floating exchange
rate.
• If the demand for Indian currency is high, the Indian rupee
will appreciate, and if demand is low, it will depreciate
Causes • Higher economic growth
• Increase in foreign investment
• Increase in exports
• Increase in the policy interest rate by the central bank
Q.90) Which one of the following is not a part of India's foreign exchange reserves?

(a) Securities issued by foreign governments


(b) Gold
(c) Special Drawing Rights issued by the International Monetary Fund
(d) Securities issued by the Government of India

EXPLANATION

Foreign exchange reserves include Foreign currency Assets, Gold stocks of RBI, Special Drawing Right
holding of the government, Reserve Tranche. Forex should contain foreign banknotes, foreign treasury
bills, foreign bank deposits, and foreign government securities. Hence securities issued by the
government of India do not fall under India's foreign exchange reserves.
So, Option (d) is correct.

ADDITIONAL INFORMATION

Foreign exchange reserves


Definition • It is vital to a nation's economic well-being
• The International Monetary Fund (IMF) defines foreign
reserves as external assets that a country’s monetary
authority can use to meet the balance of payments needs
Forex reserve • It consists of
1. Foreign Currency Assets
a. Assets Valued based on a currency other than the
country's own currency
2. Gold Stock of RBI
a. RBI has gold stock as a backup to issue currency
and to meet unexpected Balance of Payment problem
3. SDR(Special Drawing Rights holding of the government
a. It is a reserve created by IMF
b. The member country contributes in the proportion of
their IMF quota
4. Reserve Tranche
a. A portion of funds kept in reserve
Steps were taken to improve • Aatma Nirbhar Bharat to make India a self-reliant nation
Forex • Niryat Rin Vikas Yojana scheme
Recent News • Recently India become the 4th largest forex reserves holder
globally

Q.91) Which one of the following statements is correct about the “Washington Consensus”?

(a) It is the set of reforms in the United Nations Governance Structure

(b) It usually refers to the level of agreement between the International Monetary Fund and
World Bank

(c) It endorses the adoption of social welfare policies in the poor countries

(d) It is an agreement to carry out reforms in the governance of the World Trade Organization
EXPLANATION:

Washington Consensus is a set of neoliberal economic prescriptions made by the International


Monetary Fund, the World Bank, and the U.S. Treasury to developing countries that faced economic
crises. It recommended structural reforms that increased the role of market forces in exchange for
immediate financial help. The term was coined by British economist John Williamson in 1989.
So, option (b) is correct.

ADDITIONAL INFORMATION:

Washington Consensus

Recommended
The 10 recommended policy reforms of the Washington Consensus of 1989
Policy Reforms are

1. Reduce national budget deficits

2. Redirect spending from politically popular areas toward neglected fields


with high economic returns

3. Reform the tax system

4. Liberalize the financial sector with the goal of market-determined interest


rates

5. Adopt a competitive single exchange rate


.
6. Reduce trade restrictions

7. Abolish barriers to foreign direct investment

8. Privatize state-owned enterprises

9. Abolish policies that restrict competition

10. Provide secure, affordable property rights


While some said that the Washington Consensus was used to impose harsh
conditions that were unhelpful for economic recovery, others have argued
that although not perfect, it was favorable to long-term economic growth in
developing economies

India and
 India was forced into adopting the path of the Washington
Washington Consensus when it turned to the International Monetary Fund (IMF)
for support because its foreign exchange reserves had become
Consensus
precariously low.
 India had to open its economy for more imports and more
investments by foreigners. Free market ideologies, on which the
Washington Consensus was founded, forbade government-supported
industrial policies
Q.92) Consider the following statements regarding the recent remittance data by the World Bank:

1. India is the world's largest recipient of remittances.


2. Gulf counties are the biggest source of these remittances to India, accounting for over 20 per cent
of these funds.

Which of the statements given above is/are correct?

(a) 1 only

(b) 2 only

(c) Both 1 and 2

(d) Neither 1 nor 2

EXPLANATION:

India is the world's largest recipient of remittances, as it received $87 billion in remittances in 2021,
according to a World Bank report.
So, Statement 1 is correct.

For 2021, the United States was the biggest source, accounting for over 20% of these funds, World
Bank, according to a World Bank report.
So, Statement 2 is not correct.

ADDITIONAL INFORMATION:

Recipient of remittances

 India is followed by China, Mexico, the Philippines, and Egypt at the


global level.
 In India, remittances are projected to grow 3% in 2022 to $89.6 billion,
reflecting a drop in overall migrant stock, as a large proportion of
returnees from the Arab countries await the return, Remittances to low
and middle-income countries are projected to have grown a strong 7.3%
to reach $589 billion in 2021
 India had received over $83 billion in remittances in 2020.

Q.93) The term “Special Data Dissemination Standards” is related to which one of the following
organisations?

(a) World Trade Organization

(b) World Bank

(c) International Monetary Fund

(d) Organisation for Economic Co-operation and Development

EXPLANATION:

Special Data Dissemination Standard (SDDS) is an International Monetary Fund standard to guide
member countries in the dissemination of national statistics to the public. It was established in April
1996.
So, Option (c) is correct.

ADDITIONAL INFORMATION:

Special Data Dissemination Standards

Indian Context
 The Reserve Bank of India is one of the earliest central bank signatories of
SDDS.
 Under the Special Data Dissemination Standards (SDDS) of the International
Monetary Fund (IMF), central banks undertake the responsibility of
disseminating information under certain data categories, such as analytical
accounts of the banking sector, analytical accounts of the central bank,
balance of payments, international reserves and exchange rates. The IMF
requires that these data should be available at regular intervals in the public
domain.
 The IMF as well as central banks also provide a National Summary Data Page
(NSDP) on their websites to provide quick access to a single comprehensive
source of economic and financial data consistent with the data categories and
components described in the subscriber's metadata. Additionally, many of the
NSDPs include further links to additional data or information on other
national Internet data sites.

Q.94) Which one of the following ministries is the nodal ministry to facilitate the implementation of the
“SAARC Social Charter” in India?

(a) Union Ministry of Statistics and Programme Implementation

(b) Union Ministry of Social Justice and Empowerment

(c) Union Ministry of Personnel, Public Grievances and Pensions

(d) Union Ministry of Labour and Employment

EXPLANATION:

The Ministry of Statistics and Programme Implementation(MoSPI) has been designated as the nodal
Ministry to facilitate the implementation of the SAARC Social Charter in India.

So, Option (a) is correct.

ADDITIONAL INFORMATION:

SAARC Social Charter

SAARC The South Asian Association for Regional Cooperation (SAARC) was
established when its Charter was formally adopted on December 8,
1985, by the Heads of State or Government of Bangladesh, Bhutan, India,
Maldives, Nepal, Pakistan, and Sri Lanka at the First Summit meeting in
Dhaka.

The Social Establishment


Charter The SAARC Heads of State/ Government signed the SAARC
Social Charter at the 12th SAARC summit on 4th January 2004.
Objective
 The Charter envisages action in the areas of poverty alleviation,
health, education, human resource development, the status of
women, rights and well-being of children, population stabilization,
drug addiction, rehabilitation & reintegration
 The objective of the Charter. is to place people at the centre of
development and to direct the economy to meet human needs more
effectively.
Initiatives
 The SAARC countries in 2004 agreed to set up a National
Committee to facilitate the implementation of the SAARC Social
Charter and monitor performance in achieving its goals to exchange
ideas and information on best practices, apart from promoting
collaborative poverty and alleviation projects.

India's role  A National Coordination Committee under the chairpersonship of


Secretary, Ministry of Statistics and Programme Implementation
was constituted on 12 th September 2005 with members from
Planning Commission, Ministries of External Affairs, Rural
Development, Panchayati Raj, Social Justice and Empowerment,
Housing and Urban Poverty Alleviation, Health and Family
Welfare, Youth Affairs and Sports, Human Resource
Development, Women and Child Development and Finance.

Q.95) Consider the following statements:

1. India’s exports were relatively stagnant during the first three five-year plan periods.

2. India’s share in the world’s total exports has been increasing steadily since 1950.

Which of the statements given above is/are correct?

(a) 1 only
(b) 2 only
(c) Both 1 and 2
(d) Neither 1 nor 2

EXPLANATION:

India's exports were virtually stagnant during the first decade 1951-60. Total exports averaged Rs. 606
crores a year during the First Plan period and Rs. 609 crores a year during the Second Plan. In
contrast with this stagnation, exports showed a striking expansion during the first three years of the
Third Plan. There was, however, a slowing down in the growth of exports in the fourth year. In the last
year exports recorded a small fall of Rs. 10 crores as compared to the previous year, due mainly to the
bad harvest and Pakistan-India hostilities. Thus India’s exports were relatively stagnant during the first
three five-year plan periods.

So, Statement 1 is correct.


ADDITIONAL INFORMATION:

India’s Foreign Trade

Highlights  Since India opened its markets starting 1990-91, there has been an
exponential rise in the country’s foreign trade exposure – exports
have increased more than 16 times and imports more than 19 times.
In FY 2020-21, India’s imports and exports stood at US$394.43
billion and US$291.80 billion, respectively.

 The top exports of India are Refined Petroleum ($39.2B), Diamonds


($22.5B), Packaged Medicaments ($15.8B), Jewellery ($14.1B), and
Cars ($7.15B), exporting mostly to United States ($55.3B), United
Arab Emirates ($28.6B), China ($17.4B), Hong Kong ($11.5B), and
Singapore ($9.53B)

 India main imports are: mineral fuels, oils and waxes, and
bituminous substances (27 percent of total imports); pearls,
precious and semi-precious stones and jewelry (14 percent);
electrical machinery and equipment (10 percent); nuclear reactors,
boilers, machinery and mechanical appliances (8 percent)

 India's major import partners are China (16 percent of total


imports), the United States (6 percent), the United Arab Emirates (6
percent), Saudi Arabia (5 percent), and Switzerland (5 percent).

Q.96) Consider the following statements:

1. Commercial borrowings are the largest component of India’s external debt

2. More than 50 per cent of India’s external debt is denominated in US dollars.

3. India's external debt statistics for the quarters ending March and June are released by the
Department of Economic Affairs.

Which of the statements given above is/are correct?

(a) 1 and 2 only


(b) 2 and 3 only
(c) 1 only
(d) 1, 2 and 3

EXPLANATION:

Commercial borrowings remained the largest component of external debt, with a share of 37.4 percent,
followed by non-resident deposits (24.9 percent) and short-term trade credit (17.1 percent).
So, Statement 1 is correct.
US dollar-denominated debt remained the largest component of India’s external debt, with a share of
52.1 percent at the end-March 2021, followed by the Indian rupee (33.3 percent), yen (5.8 percent),
SDR (4.4 percent), and the euro (3.5 percent).
So, Statement 2 is correct.
India's external debt statistics for the quarters ending March and June are released by the Reserve
Bank of India
So, Statement 3 is not correct.

ADDITIONAL INFORMATION:

India's external debt

 At the end-March 2021, India’s external debt was placed at US$


Highlights
570.0 billion, recording an increase of US$ 11.5 billion over its level
at the end of March 2020.
 The external debt to GDP ratio increased to 21.1 percent at the end-
March 2021 from 20.6 percent at end-March 2020.
 The share of outstanding debt of non-financial corporations in total
external debt was the highest at 40.4 percent, followed by deposit-
taking corporations (except the central bank) (28.2 percent), general
government (18.8 percent), and other financial corporations (8.1
percent).
 The instrument-wise classification shows that the loans were the
largest component of external debt, with a share of 34.8 percent,
followed by currency and deposits (25.2 percent), trade credit and
advances (17.6 percent), and debt securities (17.0 percent)

Q.97) Which one of the following statements is not correct about the Non-Banking Financial Companies
(NBFCs)?

(a) Every NBFC must be registered with the Reserve Bank of India
(b) It should have a minimum net owned fund of Rs. 200 lakh
(c) It should be a company registered under the Companies Act, 1956
(d) NBFCs whose asset size is Rs. 500 crores or more as per the last audited balance sheet are
considered as systemically important NBFCs

EXPLANATION :

In terms of Section 45-IA of the RBI Act, 1934, no Non-banking Financial company can commence or
carry on business of a non-banking financial institution without a) obtaining a certificate of registration
from RBI.

However, in terms of the powers given to the Bank, to obviate dual regulation, certain categories of
NBFCs which are regulated by other regulators are exempted from the requirement of registration with
RBI viz.
• Venture Capital Fund/Merchant Banking companies/Stock broking companies registered with
SEBI,
• Insurance Company holding a valid Certificate of Registration issued by IRDA,
• Nidhi companies as notified under Section 620A of the Companies Act, 1956,
• Chit companies as defined in clause (b) of Section 2 of the Chit Funds Act, 1982,
• Housing Finance Companies regulated by National Housing Bank, Stock Exchange or a Mutual
Benefit company.
So, statement 1 is not correct.

A company incorporated under the Companies Act, 1956 and desirous of commencing business of non-
banking financial institution as defined under Section 45 I(a) of the RBI Act, 1934 should comply with
the following:
i. it should be a company registered under Section 3 of the companies Act, 1956
ii. It should have a minimum net owned fund of ₹ 200 lakh.
So, statement 2 is correct.
It should be a company registered under the Companies Act, 1956
So, statement 3 is correct.

NBFCs whose asset size is of ₹ 500 cr or more as per last audited balance sheet are considered as
systemically important NBFCs.

The rationale for such classification is that the activities of such NBFCs will have a bearing on the
financial stability of the overall economy.
So, statement 4 is correct.
ADDITIONAL INFORMATION :

Non-Banking Financial Companies (NBFCs):

Definition • A Non-Banking Financial Company (NBFC) is a company registered under the


Companies Act, 1956
o engaged in the business of loans and advances,
o acquisition of shares/stocks/bonds/debentures/securities issued by
Government or local authority
o or other marketable securities of a like nature, leasing, hire-purchase,
insurance business,
o chit business
• But does not include any institution whose principal business is that of
agriculture activity, industrial activity, purchase or sale of any goods (other than
securities) or providing any services and sale/purchase/construction of
immovable property.
• A non-banking institution which is a company and has principal business of
receiving deposits under any scheme or arrangement in one lump sum or in
installments by way of contributions or in any other manner, is also a non-
banking financial company (Residuary non-banking company).
Difference NBFCs lend and make investments and hence their activities are akin to that of banks;
with banks however there are a few differences as given below:

i. NBFC cannot accept demand deposits;


ii. NBFCs do not form part of the payment and settlement system and cannot issue
cheques drawn on itself;
iii. deposit insurance facility of Deposit Insurance and Credit Guarantee Corporation is
not available to depositors of NBFCs, unlike in case of banks.
Types NBFCs are categorized

a) in terms of the type of liabilities into Deposit and Non-Deposit accepting NBFCs,
b) non deposit taking NBFCs by their size into systemically important and other
non-deposit holding companies (NBFC-NDSI and NBFC-ND) and
c) by the kind of activity they conduct.

Within this broad categorization the different types of NBFCs are as follows:

I. Asset Finance Company (AFC) :

An AFC is a company which is a financial institution carrying on as its principal business


the financing of physical assets supporting productive/economic activity, such as
automobiles, tractors, lathe machines, generator sets, earth moving and material
handling equipments, moving on own power and general purpose industrial machines.
Principal business for this purpose is defined as aggregate of financing real/physical
assets supporting economic activity and income arising therefrom is not less than 60% of
its total assets and total income respectively.

II. Investment Company (IC) :

IC means any company which is a financial institution carrying on as its principal


business the acquisition of securities,

III. Loan Company (LC):

LC means any company which is a financial institution carrying on as its principal


business the providing of finance whether by making loans or advances or otherwise for
any activity other than its own but does not include an Asset Finance Company.

IV. Infrastructure Finance Company (IFC):

IFC is a non-banking finance company a) which deploys at least 75 per cent of its total
assets in infrastructure loans, b) has a minimum Net Owned Funds of ₹ 300 crore, c) has
a minimum credit rating of ‘A ‘or equivalent d) and a CRAR of 15%.

V. Systemically Important Core Investment Company (CIC-ND-SI): CIC-ND-SI is an


NBFC carrying on the business of acquisition of shares and securities

Q.98) As per the standard practice, India’s “International Investment Position” is published on a
quarterly basis by
(a) Reserve Bank of India
(b) Ministry of Finance
(c) Department of Investment and Public Asset Management
(d) NITI Aayog
EXPLANATION:

After January 2007, the Reserve Bank of India published IIP every quarter. Before that, it was
brought about on an annual basis.
So, Option (a) is correct.
ADDITIONAL INFORMATION:

India’s International Investment Position

International
The International investment position (IIP) of a country is a financial
Investment
statement of the value and composition of its external financial assets and
Position
liabilities. A positive NIIP value indicates that a nation is a creditor nation,
while a negative value indicates that it is a debtor nation.
Key Features of The Reserve Bank released data relating to India’s International Investment
India’s IIP in June Position in end-June 2021.
2021
 Net claims of non-residents in India declined by US$ 24.3 billion
during April-June 2021 to US$ 327.0 billion as of end-June
2021.
 Reserve assets accounted for 95 percent of the increase of US$
35.9 billion in Indian residents’ overseas financial assets during
the quarter.
 Foreign-owned assets in India recorded an increase of US$ 11.6
billion, corresponding to an equal rise in inward foreign direct
investment.
 Variation in the exchange rate of rupee vis-a-vis other currencies
affected change in liabilities when valued in US$ terms.
 Reserve assets continued to have the dominant share (68.3
percent) in India’s international financial assets.
 The share of non-debt liabilities in total international liabilities
edged up to 52.7 percent in June 2021.
 The ratio of international assets to international liabilities has
gradually improved to 73.2 percent in June 2021 from 68.5
percent a year ago

Q.99) Which one of the following statements is not correct regarding the Right to Freedom of Movement,
as per the Indian Constitution?

(a) It gives every citizen the right to move freely throughout the country
(b) It is protected against only state action and not private individuals
(c) It is available only to the citizens and to shareholders of a company but not to foreigners or legal
persons
(d) Making FASTag mandatory for all vehicles plying on the National Highways could be
considered as a breach of this right

EXPLANATION:

Article 19 of the Indian Constitution consists of the clause Right to Freedom of Movement in that it
gives rights to every citizen of India to move freely throughout the territory of the country. It consists of
two dimensions as Internal (right to move inside the country) and External (right to move in and go out
of the country) but the latter one is dealt with the Article 21. So, option (a) is correct.
The right to move freely is protected only against state action and not private individuals. But this right
is not absolute, even though this protects the citizen’s movement from the state action, has some
reasonable restrictions mentioned are the interests of the general public and the protection of
scheduled tribes and their habitat. So, option (b) is correct.
This right is available only to the citizens and to shareholders of a company but not to foreigners or
legal persons like companies or corporations etc, and articles 15,16,19,29, and 30 are confined only to
Indian citizens. So, option (c) is correct.
Recently the Bombay high court has said that making FASTAG mandatory for all vehicles will not
breach the fundamental rights in any way. So, option (d) is not correct.

ADDITIONAL INFORMATION:

IMPORTANT CASES OR JUDGEMENTS RELATED TO THE RIGHT TO MOVEMENT


KHARAK SINGH VS STATE OF UP (1963) • The right to move connotes the fact wherever one
CASE likes and however one likes.
STATE OF UP VS KAUSHALYA CASE • In this regard, the right of movement of
(1964) prostitutes has restrictions under the interests of
public health and morals.
RECENT SUPREME COURT JUDGEMENT • Externment orders will be given by the state
which prevents the movement under certain
conditions.
• But the court said that orders have to be given
only in exceptional cases such as to maintain law
and order in a locality or to prevent a breach of
any public peace and not on flimsy grounds.

Q.100) Consider the following statements about the “Central Bank Digital Currency”:

1. It uses an electronic record or digital token to represent the virtual form of a fiat currency of a
particular nation.
2. It is centralized and is issued and regulated by the competent monetary authority of the country.
3. Bitcoin is one of these kinds of digital currencies in the World in recent times.

Which of the statements given above is/are correct?

(a) 1 only
(b) 1 and 2 only
(c) 2 and 3 only
(d) 1, 2 and 3

You might also like